Currently Empty: ₹0.00
Respiratory Module Apr 2022 MRCP Part-1
Time limit: 0
Quiz Summary
0 of 123 Questions completed
Questions:
Information
You have already completed the quiz before. Hence you can not start it again.
Quiz is loading…
You must sign in or sign up to start the quiz.
You must first complete the following:
Results
Quiz complete. Results are being recorded.
Results
0 of 123 Questions answered correctly
Your time:
Time has elapsed
You have reached 0 of 0 point(s), (0)
Earned Point(s): 0 of 0, (0)
0 Essay(s) Pending (Possible Point(s): 0)
Categories
- Respiratory Module Apr 2022 MRCP Part-1 0%
- 1
- 2
- 3
- 4
- 5
- 6
- 7
- 8
- 9
- 10
- 11
- 12
- 13
- 14
- 15
- 16
- 17
- 18
- 19
- 20
- 21
- 22
- 23
- 24
- 25
- 26
- 27
- 28
- 29
- 30
- 31
- 32
- 33
- 34
- 35
- 36
- 37
- 38
- 39
- 40
- 41
- 42
- 43
- 44
- 45
- 46
- 47
- 48
- 49
- 50
- 51
- 52
- 53
- 54
- 55
- 56
- 57
- 58
- 59
- 60
- 61
- 62
- 63
- 64
- 65
- 66
- 67
- 68
- 69
- 70
- 71
- 72
- 73
- 74
- 75
- 76
- 77
- 78
- 79
- 80
- 81
- 82
- 83
- 84
- 85
- 86
- 87
- 88
- 89
- 90
- 91
- 92
- 93
- 94
- 95
- 96
- 97
- 98
- 99
- 100
- 101
- 102
- 103
- 104
- 105
- 106
- 107
- 108
- 109
- 110
- 111
- 112
- 113
- 114
- 115
- 116
- 117
- 118
- 119
- 120
- 121
- 122
- 123
- Current
- Review
- Answered
- Correct
- Incorrect
-
Question 1 of 123
1. Question
Which of the following statement is true of infections with Mycobacterium tuberculosis?
CorrectIncorrectHint
Commonly, untreated Smear-Positive Pulmonary Tuberculosis [TB] is likely to be infectious. Active disease may be indicated by Grade III / IV response to Tuberculin. Eighty percent of individuals with a history of BCG Vaccination have Grade I / II response._x000D_
All forms of Pulmonary Tuberculosis [TB] may be treated equally except Tuberculous Pleural Effusion which may require drainage (with large effusions causing breathlessness) and adjunct corticosteroids to delay reaccumulation._x000D_
The length of treatment for other forms are:_x000D_
? Bone Tuberculosis [Bone TB] ? 9 months_x000D_
? Meningitis ? 1 year_x000D_
? Drug Resistance ? 2 years._x000D_
Streptomycin has high activity against extracellular organisms whilst Pyrazinamide have high activity against intracellular organisms. -
Question 2 of 123
2. Question
Which of the following statements regarding Idiopathic Pulmonary Fibrosis is correct?
CorrectIncorrectHint
About 50% of patients have an improvement in their symptoms with steroids and 25% have improved lung function. A recent study has suggested this treatment may actually be associated with an increased mortality, but further studies are required before this is widely accepted._x000D_
High Resolution CT Scans [HRCTs] can be used to determine the Stage of Idiopathic Pulmonary Fibrosis. When Ground Glass Attenuation predominates, it usually represents an Inflammatory Cell Infiltrate which has been shown to respond well to treatment. As the disease progresses it develops a Reticular Appearance which represents Irreversible Fibrotic Changes. The presence of a predominantly Ground Glass Appearance is also an independent predictor of survival._x000D_
Both Total Lung Capacity (TLC) and Residual Volume (RV) reduce with restrictive disorders like Idiopathic Pulmonary Fibrosis, so the ratio is maintained. A raised RV / TLC Ratio suggests a combination of Airways Obstruction and Restrictive Defect not just Idiopathic Pulmonary Fibrosis as mentioned in this question._x000D_
Peak Flow measures Airway Obstruction. Idiopathic Pulmonary Fibrosis is characterised by a Restrictive Defect on Lung Function Testing._x000D_
Peak Incidence is in the Sixth Decade. -
Question 3 of 123
3. Question
Which one of the following statements regarding idiopathic pulmonary fibrosis is correct?
CorrectIncorrectHint
Active inflammation may be suggested by a computed tomography (CT) scan (option C) is the correct answer. A high-resolution computed tomography (HRCT) scan of the chest can show the characteristic features of idiopathic pulmonary fibrosis, such as honeycombing, traction bronchiectasis, and subpleural reticular opacities. However, these findings do not indicate active inflammation, but rather chronic fibrosis and architectural distortion of the lung parenchyma. In some cases, the high-resolution computed tomography (HRCT) scan may also show areas of ground glass attenuation, which are regions of increased lung density that do not obscure the underlying structures. Ground glass attenuation may represent an inflammatory cell infiltrate, which has been shown to respond well to corticosteroid treatment and have a better prognosis than fibrotic lesions. Therefore, the presence of ground glass attenuation on high-resolution computed tomography (HRCT) scan may suggest active inflammation in idiopathic pulmonary fibrosis.
_x000D_
The other options are incorrect for the following reasons:
_x000D_
- _x000D_
- A. Lung volumes show a raised residual volume / total lung capacity ratio: This is incorrect. Lung volumes are measurements of the amount of air that the lungs can hold at different phases of the respiratory cycle. Residual volume (RV) is the amount of air that remains in the lungs after a full exhalation, and total lung capacity (TLC) is the maximum amount of air that the lungs can hold. The ratio of residual volume to total lung capacity reflects the degree of air trapping in the lungs, and a high ratio indicates reduced lung function. Idiopathic pulmonary fibrosis causes scarring and stiffening of the lung tissue, which reduces the lung’s ability to expand and contract normally. This leads to a decrease in both residual volume and total lung capacity, so the ratio is maintained or slightly reduced. A raised residual volume / total lung capacity ratio suggests a combination of airways obstruction and restrictive defect, not just idiopathic pulmonary fibrosis as mentioned in this question.
- B. Peak incidence seen in the fourth decade: This is incorrect. The peak incidence of idiopathic pulmonary fibrosis is seen in the sixth or seventh decade of life, not the fourth. The average age of diagnosis is around 66 years, and the disease is more common in men than in women.
- D. 80% of patients initially respond well to immunosuppression: This is incorrect. There is no evidence that immunosuppressive therapy is effective or beneficial for idiopathic pulmonary fibrosis. In fact, some studies have shown that immunosuppression may be harmful and increase the risk of mortality and infection in these patients. The only approved medications for idiopathic pulmonary fibrosis are pirfenidone and nintedanib, which can slow down the progression of the disease, but not reverse it.
- E. Peak flow rate is a good guide to severity: This is incorrect. Peak flow rate is a measure of the maximum speed of air that can be exhaled from the lungs. It is mainly used to monitor and manage asthma, which is an obstructive lung disease that causes narrowing of the airways. Idiopathic pulmonary fibrosis, on the other hand, is a restrictive lung disease that causes reduced lung volume and compliance. Therefore, peak flow rate is not a good indicator of the severity or prognosis of idiopathic pulmonary fibrosis. A better measure is the forced vital capacity (FVC), which is the amount of air that can be forcibly exhaled from the lungs after a full inhalation.
_x000D_
_x000D_
_x000D_
_x000D_
-
Question 4 of 123
4. Question
According to the latest National Institute for Health and Care Excellence (NICE) guidance, which one of the below combinations of post-bronchodilator results is now classed as severe airflow obstruction in chronic obstructive pulmonary disease (COPD)?
CorrectIncorrectHint
According to the latest National Institute for Health and Care Excellence (NICE) guidance, the combination of post-bronchodilator results that is now classed as severe airflow obstruction in chronic obstructive pulmonary disease (COPD) is option C: FEV1/FVC <0.7 & FEV1 predicted <30%. This is a change from the previous National Institute for Health and Care Excellence (NICE) guideline, which used the GOLD criteria and defined severe airflow obstruction as FEV1/FVC <0.7 & FEV1 predicted 30-49% (option A). The new National Institute for Health and Care Excellence (NICE) guidance has adopted the lower limit of normal (LLN) approach, which defines airflow obstruction as FEV1/FVC below the fifth percentile of the normal population. This means that some people who were previously classified as having severe airflow obstruction may now be reclassified as having very severe airflow obstruction, which has implications for their management and prognosis.
_x000D_
What Is GOLD Criteria:
_x000D_
The GOLD criteria are a set of guidelines for the diagnosis, management, and prevention of chronic obstructive pulmonary disease (COPD). They are developed by the Global Initiative for Chronic Obstructive Lung Disease (GOLD), collaboration between the National Heart, Lung, and Blood Institute (NHLBI), the World Health Organization (WHO), and other international health organizations.
_x000D_
The GOLD criteria use a combination of spirometry results, symptom assessment, exacerbation history, and comorbidity evaluation to classify COPD into four grades of severity and four groups of treatment recommendations. The spirometry results measure the degree of airflow obstruction, based on the ratio of forced expiratory volume in one second (FEV1) to forced vital capacity (FVC) and the percentage of predicted FEV1. The symptom assessment uses either the COPD Assessment Test (CAT) or the Modified Medical Research Council (mMRC) questionnaire to evaluate the impact of dyspnoea, cough, and sputum on the patient’s quality of life. The exacerbation history counts the number and severity of chronic obstructive pulmonary disease (COPD) flare-ups in the past year. The comorbidity evaluation considers the presence of other chronic diseases that may affect the patient’s prognosis and treatment.
_x000D_
The GOLD criteria are updated periodically to reflect the latest evidence and best practices in chronic obstructive pulmonary disease (COPD) care. The most recent version is the 2023 GOLD Report, which introduced some changes from the previous versions, such as a new definition of chronic obstructive pulmonary disease (COPD), a lower limit of normal (LLN) approach for spirometry, and a simplified treatment algorithm.
_x000D_
NOTE:
_x000D_
There have been some alterations to previous classification of airflow obstruction, based on the American Thoracic Society, GOLD and European Respiratory Society Standards.
_x000D_
Below are the classifications:
_x000D_
_x000D_
FEV1/FVC ratio is less than 0.7 in chronic obstructive pulmonary disease (COPD).
-
Question 5 of 123
5. Question
Which of the following statements is true of Psittacosis (Ornithosis)?
CorrectIncorrectHint
Chlamydia psittaci is endemic in birds including psittacine birds, canaries, finches, pigeons and poultry._x000D_
Pet owners, vets and zoo keepers are most at risk. It is rare in children._x000D_
Person to person transmission occurs though it is very rare._x000D_
Sputum Gram stain reveals a few leucocytes and no predominant bacteria. There are few signs and few laboratory/x ray findings._x000D_
Positive serology is with complement-fixing antibodies._x000D_
It is treated with tetracycline. -
Question 6 of 123
6. Question
A 17-year-old boy presented with a strongly positive Mantoux test.
_x000D_
Which one of the following statements regarding his immune reaction is correct?
CorrectIncorrectHint
The correct answer to the above question is B. It is a cell-mediated immune response. This is the correct statement regarding this boy’s immune reaction to the Mantoux test.
_x000D_
The Mantoux test, also known as the tuberculin skin test (TST), is a diagnostic tool used to detect exposure to Mycobacterium tuberculosis, the bacterium that causes tuberculosis (TB). Mantoux test has replaced Heaf test in 2005 in United Kingdom. It involves injecting a small amount of purified protein derivative (PPD), a substance derived from the tuberculosis (TB) bacterium, just under the surface of the skin on the inside of the forearm.
_x000D_
The immune reaction to the Mantoux test is a type of delayed hypersensitivity reaction, which is mediated by T lymphocytes, a type of white blood cells that are involved in cell-mediated immunity.
_x000D_
Cell-mediated immunity is a branch of the adaptive immune system that protects the body from intracellular pathogens, such as viruses, bacteria, fungi, and parasites, by activating macrophages, natural killer cells, and cytotoxic T cells.
_x000D_
If the person has been exposed to the tuberculosis (TB) bacterium in the past, either by infection or vaccination, their T lymphocytes will recognize the purified protein derivative (PPD) as a foreign antigen and mount an immune response. This will cause inflammation and swelling at the site of injection, which is measured as the area of induration (hardening) after 48 to 72 hours. The size of the induration reflects the degree of the immune reaction and the likelihood of tuberculosis (TB) infection.
_x000D_
The other statements are incorrect regarding his immune reaction to the Mantoux test, as follows:
_x000D_
- _x000D_
- A. The response is mediated by B lymphocytes: This is incorrect, as B lymphocytes are a type of white blood cells that are involved in humoral immunity, not cell-mediated immunity. Humoral immunity is a branch of the adaptive immune system that protects the body from extracellular pathogens, such as bacteria, toxins, and venoms, by producing antibodies. Antibodies are proteins that bind to specific antigens and neutralize or eliminate them. The Mantoux test does not measure the antibody response to the tuberculosis (TB) bacterium, but the T cell response.
- C. The area of induration will be less than 10 mm in diameter: This is incorrect, as the area of induration may vary depending on the person’s risk factors for tuberculosis (TB) infection. In general, an induration of 5 mm or more is considered positive for people who have a high risk of tuberculosis (TB) infection, such as those who have HIV infection, close contact with tuberculosis (TB) case, or chest x-ray findings suggestive of tuberculosis (TB). An induration of 10 mm or more is considered positive for people who have a moderate risk of tuberculosis (TB) infection, such as those who have certain medical conditions, work or live in high-risk settings, or come from countries with high tuberculosis (TB) prevalence. An induration of 15 mm or more is considered positive for people who have a low risk of tuberculosis (TB) infection, such as those who have no known risk factors.
- D. The reaction typically develops within 24 hours: This is incorrect, as the reaction to the Mantoux test is a delayed hypersensitivity reaction, which takes 48 to 72 hours to develop. This is because it takes time for the T lymphocytes to migrate to the site of injection and release inflammatory mediators. If the reaction develops within 24 hours, it may indicate an immediate hypersensitivity reaction, which is mediated by IgE antibodies and mast cells, not T lymphocytes. This type of reaction is usually seen in people who have a history of atopy, such as allergic rhinitis, asthma, or eczema.
- E. If a skin biopsy were taken, immunohistochemistry would show immune complex deposition: This is incorrect, as immune complex deposition is a feature of type III hypersensitivity reaction, not type IV hypersensitivity reaction. Type III hypersensitivity reaction is mediated by IgG or IgM antibodies and complement, not T lymphocytes. Immune complex deposition occurs when the antibodies bind to soluble antigens and form insoluble aggregates that deposit in the tissues and blood vessels, causing inflammation and damage. This type of reaction is seen in some autoimmune diseases, such as systemic lupus erythematosus, rheumatoid arthritis, or vasculitis. If a skin biopsy were taken from the Mantoux test site, immunohistochemistry would show T lymphocyte infiltration, not immune complex deposition.
_x000D_
_x000D_
_x000D_
_x000D_
-
Question 7 of 123
7. Question
A 16-year-old girl presents at the respiratory clinic with shortness of breath and insomnia prior to an examination.
_x000D_
Clinical examination is normal. Chest x-ray and peak expiratory flow rate (PEFR) are normal.
_x000D_
Which one of the following investigations is most suggestive of asthma?
CorrectIncorrectHint
The most suggestive investigation of asthma in this case would be option D: Diurnal variation in peak expiratory flow rate (PEFR) >20%.
_x000D_
While options A, C, and E may indicate a potential allergic predisposition, they are not specific to asthma and can be present in other conditions as well. Option B, resolution of symptoms the day after the examination, could be attributed to various factors and does not necessarily point towards asthma.
_x000D_
Diurnal variation in peak expiratory flow rate (PEFR) >20% is a characteristic feature of asthma. Asthma symptoms tend to worsen during the night and early morning due to increased airway inflammation and bronchoconstriction. This results in decreased airflow and lower peak expiratory flow rate (PEFR) readings. In contrast, as the day progresses and the airways relax, the peak expiratory flow rate (PEFR) readings tend to improve. A diurnal variation of >20% in peak expiratory flow rate (PEFR) is suggestive of bronchial hyperresponsiveness, a hallmark feature of asthma.
_x000D_
Therefore, option D, diurnal variation in peak expiratory flow rate (PEFR) >20%, is the most indicative investigation suggesting asthma in this case.
-
Question 8 of 123
8. Question
A 16-year-old boy presents to the Emergency Department with Acute Severe Asthma.
_x000D_
On examination, his Peripheral Pulse Volume fell during Inspiration.
_x000D_
Which one of the following is the most likely explanation for this clinical sign?
CorrectIncorrectHint
This patient is demonstrating Pulsus Paradoxus._x000D_
The right heart responds directly to changes in intrathoracic pressure, while the filling of the left heart depends on the pulmonary vascular volume._x000D_
At high respiratory rates, with severe air flow limitation (for example, acute asthma) there is an increased and sudden negative intrathoracic pressure on inspiration and this will enhance the normal fall in blood pressure. -
Question 9 of 123
9. Question
A 16-year-old boy is brought to the Emergency Department with acute severe asthma.
_x000D_
On examination, his peripheral pulse volume fell during inspiration.
_x000D_
Which one of the following is the most likely explanation for this clinical sign?
CorrectIncorrectHint
The most likely explanation for this clinical sign is C. Reduced left atrial filling pressure on inspiration. This is because this boy has pulsus paradoxus, an abnormally large decrease in systolic blood pressure and pulse wave amplitude during inspiration. Pulsus paradoxus is a sign of severe asthma, as well as other conditions that affect the heart and lungs.
_x000D_
Pulsus paradoxus occurs when the negative intra-thoracic pressure during inspiration causes an increased right venous return, which expands the right atrium and reduces the compliance of the left atrium due to their shared septum. This leads to a reduced left atrial filling pressure and a reduced left ventricular stroke volume, which manifests as a decrease in systolic blood pressure and pulse volume.
_x000D_
The other options are incorrect because they do not explain the pulsus paradoxus phenomenon. Option A is incorrect because myocardial depression due to hypoxia would cause a global reduction in cardiac output, not a variation with the respiratory cycle. Option B is incorrect because beta-agonist bronchodilator drugs would increase the heart rate and blood pressure, not decrease them. Option D is incorrect because the heart rate usually increases slightly during inspiration, not decreases. Option E is incorrect because peripheral vasodilatation would cause a decrease in diastolic blood pressure, not systolic blood pressure.
-
Question 10 of 123
10. Question
A 15-year-old girl is brought to the pulmonology clinic with an acute exacerbation of asthma.
_x000D_
On examination, her respiratory rate was 31 breaths per minute, her heart rate was 120 beats per minute and a peak expiratory flow rate (PEFR) was 30% of the predicted value.
_x000D_
Her blood gas analysis (on air) shows:
_x000D_
_x000D_ _x000D_
_x000D_ _x000D_ PaO2
_x000D_
_x000D_
_x000D_ 8.7 kPa
_x000D_
_x000D_
_x000D_ (11.3-12.6)
_x000D_
_x000D_
_x000D_
_x000D_ _x000D_ PaCO2
_x000D_
_x000D_
_x000D_ 3.3 kPa
_x000D_
_x000D_
_x000D_ (4.7-6.0)
_x000D_
_x000D_
_x000D_
_x000D_
_x000D_
After administering oxygen and corticosteroids, which one of the following is the most appropriate next step to be incorporated into the management?
CorrectIncorrectHint
The most appropriate next step to be incorporated into the management of this girl is B. Salbutamol via oxygen-driven nebuliser. This is because salbutamol is a short-acting beta 2 agonist that can rapidly relieve the bronchospasm and airflow obstruction in acute asthma exacerbations. Oxygen-driven nebulisation can deliver a higher dose of salbutamol than a metered-dose inhaler and can also improve the oxygen saturation of the patient. Salbutamol via oxygen-driven nebuliser is recommended for patients with moderate to severe asthma exacerbations who have a PEF of less than 50% of the predicted value.
_x000D_
The other options are less appropriate for the following reasons:
_x000D_
- _x000D_
- A. Intravenous salbutamol is not routinely used for acute asthma exacerbations, as it has no proven benefit over inhaled salbutamol and may cause more adverse effects, such as tachycardia, hypokalaemia, and hyperglycaemia.
- C. Ipratropium bromide is an anticholinergic agent that can be added to salbutamol for patients with severe asthma exacerbations who do not respond adequately to salbutamol alone. However, it is not a substitute for salbutamol and should not be used as monotherapy.
- D. Salmeterol is a long-acting beta 2 agonist that can prevent asthma symptoms and reduce the risk of exacerbations when used as a maintenance therapy in combination with inhaled corticosteroids. However, it is not indicated for the treatment of acute asthma exacerbations, as it has a slower onset of action and a longer duration of action than salbutamol.
_x000D_
_x000D_
_x000D_
_x000D_
- _x000D_
- Intravenous aminophylline is a methylxanthine that can have bronchodilator and anti-inflammatory effects in asthma. However, it is not recommended for the routine management of acute asthma exacerbations, as it has a narrow therapeutic window and a high risk of toxicity, such as nausea, vomiting, arrhythmias, and seizures. It may be considered as a rescue therapy for patients with life-threatening asthma exacerbations who do not respond to other treatments, but only under close monitoring and with serum level measurement.
_x000D_
_x000D_
Another Approach:
_x000D_
This young girl meets the criteria for acute severe asthma. In such cases, it is important to administer β2-agonists as soon as possible, preferably through nebulization with oxygen. These doses should be repeated at 15-30 minute intervals, or continuous nebulization can be used if there is an inadequate response to bolus therapy. For patients with acute severe or life-threatening asthma, or those with a poor initial response, it is recommended to add nebulized ipratropium bromide, as it provides significantly greater bronchodilation when combined with a β2-agonist.
_x000D_
To maintain oxygen saturation levels between 94-98%, oxygen should be given. If a patient’s saturation falls below 92% on air, an arterial blood gas (ABG) should be performed to rule out hypercapnia. However, it is important not to delay treatment for the arterial blood gas (ABG). Initially, high-flow oxygen is used and then gradually reduced to maintain adequate saturation levels. Unless the clinician suspects chronic obstructive pulmonary disease (COPD), there is no need to be cautious with oxygen therapy.
_x000D_
Steroids play a crucial role in reducing mortality, relapses, subsequent hospital admissions, and the need for β2-agonists. Therefore, once oxygen and nebulizer therapy are established, a dose of 40-50 mg of steroids should be given. This treatment should be continued for 5 days or until recovery, and can be stopped abruptly unless the patient has been taking long-term oral corticosteroids.
_x000D_
If the patient fails to respond to the above treatment steps, the use of intravenous magnesium sulphate and aminophylline may be considered. However, it is important to have a discussion with senior colleagues of the treating physician before administering these medications.
_x000D_
Intensive care is necessary for patients with severe acute or life-threatening asthma who are not responding to therapy. It should be considered for patients with deteriorating peak flow, persistent or worsening hypoxia, hypercapnia, acidosis, exhaustion, or altered consciousness. When transferring patients to an Intensive Care Unit (ICU), they should be accompanied by a doctor who is capable of intubation if necessary.
_x000D_
As a side note, chest radiographs are not typically necessary unless the clinician suspects pneumothorax or consolidation, or if there is life-threatening asthma, a failure to respond to treatment, or a need for ventilation.
-
Question 11 of 123
11. Question
A 15-year-old boy with cystic fibrosis presents to the General Clinic with abdominal pain. There is no associated nausea and vomiting.
_x000D_
Which of the following is most likely to be the cause?
CorrectIncorrectHint
Distal Intestinal Obstruction Syndrome occurs in 10-20% of patients with Cystic Fibrosis and incidence increases with age. About 80% of cases present for the first time in adults._x000D_
The pathogenesis is partially due to loss of CFTR Function in the Intestine which results in deregulation of chloride secretion from the crypts, bicarbonate secretion from Brunner’s glands and sodium transport. This leads to the accumulation of viscous mucus and faecal material in the terminal ileum, caecum, and ascending colon._x000D_
Investigation should include a plain abdominal radiograph which classically shows faecal loading in the right iliac fossa, dilatation of the ileum and an empty distal colon._x000D_
Ultrasound may be helpful in identifying an obstruction mass but cannot be relied upon to exclude other causes of pain and bowel obstruction._x000D_
CT can help with diagnosis and shows dilated small bowel and proximal colon with or without intestinal wall swelling._x000D_
Treatment for mild and moderate episodes is initially with hydration and full dietetic review to ensure that the pancreatic enzyme dose is titrated to fat intake. Regular laxatives should be given, for example, Senna and Lactulose._x000D_
In addition, N-Acetylcysteine can be used in moderate episodes. This loosens and softens the plugs, presumably by ‘opening’ the disulphide bonds in the abnormal intestinal mucus and maintains luminal patency._x000D_
Severe episodes can be treated with Gastrograffin or Klean-Prep, taken regularly._x000D_
If there are signs of peritoneal irritation or complete bowel obstruction, surgical review should be obtained. Surgeons will often treat initially with Intravenous Fluids and an NG Tube whilst keeping the patient nil by mouth. N-Acetylcysteine can be put down the NG Tube._x000D_
In resistant cases, Phosphate or Gastrograffin Enemas can be used, or Colonoscopy with installation of Gastrograffin. -
Question 12 of 123
12. Question
A 12-year-old girl presents to the Pulmonology Clinic with an Acute Exacerbation Of Asthma.
_x000D_
On examination, her Respiratory rate was 35/Minute, her heart rate was 110 Beats Per Minute and a Peak Expiratory Flow Rate (PEFR) was 30% of the predicted value.
_x000D_
Her Blood Gas Analysis on Air shows:
_x000D_
_x000D_ _x000D_
_x000D_ _x000D_ PaO2
_x000D_
_x000D_
_x000D_ 8.5 kPa
_x000D_
_x000D_
_x000D_ (11.3-12.6)
_x000D_
_x000D_
_x000D_
_x000D_ _x000D_ PaCO2
_x000D_
_x000D_
_x000D_ 3.1 kPa
_x000D_
_x000D_
_x000D_ (4.7-6.0)
_x000D_
_x000D_
_x000D_
_x000D_
_x000D_
After the administration of Oxygen and Corticosteroids what is the most appropriate next step to be incorporated in the management?
CorrectIncorrectHint
This patient fits the criteria for Acute Severe Asthma. In such cases ?2-Agonists should be administered as soon as possible, preferably nebulised driven by oxygen. Repeat doses should be given at 15-30 minute intervals, or continuous nebulisation can be used where there is inadequate response to bolus therapy. Nebulised ipratropium bromide should be added for patients with acute severe or life threatening asthma, or those with a poor initial response. It’s addition produces significantly greater bronchodilation than a ?2-agonist alone._x000D_
Oxygen should be given to maintain saturations at 94-98%. Patients with saturations less than 92% on air should have an ABG to exclude hypercapnia. However, starting treatment should not be delayed to do the ABG. Initially high-flow oxygen is used, and then weaned to maintain adequate saturations. Unless you suspect COPD there is no need to be cautious with oxygen therapy._x000D_
Steroids reduce mortality, relapses, subsequent hospital admission and requirement for ?2-agonists1. The earlier they are given in the attack, the better the outcome. A dose of 40-50 mg should therefore be given once oxygen and nebuliser therapy has been established. This should be continued for 5 days, or until recovery, and can then be stopped abruptly unless the patient has taken long-term oral corticosteroids._x000D_
Failure to respond to the above treatment steps may warrant the use of intravenous magnesium sulphate and aminophylline. However, these should not be used without discussion with your senior colleagues._x000D_
Intensive care is indicated for patients with severe acute or life threatening asthma who are failing to respond to therapy. Consider it in patients with deteriorating peak flow, persisting or worsening hypoxia, hypercapnia, acidosis, exhaustion or altered conscious state. All patients who are transferred to an intensive care unit should be accompanied by a doctor who can intubate if necessary._x000D_
As an aside, chest radiographs are not indicated unless you suspect pneumothorax or consolidation, or there is life-threatening asthma, a failure to respond to treatment or a need for ventilation. -
Question 13 of 123
13. Question
Which one of the following recognised associations is correct?
CorrectIncorrectHint
Option E: Bronchopulmonary aspergillosis and wheezing is the correct answer. Bronchopulmonary aspergillosis is a spectrum of lung diseases caused by an allergic or invasive reaction to the fungus Aspergillus fumigatus, which is commonly found in soil, decaying vegetation, and dust. Wheezing is a high-pitched sound produced by the air passing through narrowed airways, which can be caused by various conditions, such as asthma, bronchitis, or chronic obstructive pulmonary disease (COPD). Bronchopulmonary aspergillosis and wheezing are often associated, especially in the form of allergic bronchopulmonary aspergillosis (ABPA), which occurs in some people with asthma or cystic fibrosis who have a hypersensitivity reaction to the fungus. Allergic bronchopulmonary aspergillosis (ABPA) can cause symptoms such as coughing, fever, mucus production, and worsening of asthma.
_x000D_
The other options are incorrect for the following reasons:
_x000D_
- _x000D_
- A. Pulmonary fibrosis and hypercapnia: Pulmonary fibrosis is a condition that causes scarring and stiffening of the lung tissue, impairing gas exchange. Hypercapnia is a state of elevated carbon dioxide levels in the blood, which can result from hypoventilation or ventilation-perfusion mismatch. Pulmonary fibrosis can cause hypercapnia, but it is not a specific or exclusive association, as many other lung diseases can also cause hypercapnia.
- B. Pulmonary embolus and left bundle branch block (LBBB): Pulmonary embolus is a blockage of a pulmonary artery by a blood clot or other material, which can cause chest pain, shortness of breath, and haemoptysis. Left bundle branch block (LBBB) is a cardiac conduction abnormality that causes a delay in the electrical activation of the left ventricle, which can be seen on an electrocardiogram (ECG). Pulmonary embolus and left bundle branch block (LBBB) are not directly related, although both can affect the right ventricular function and cause pulmonary hypertension.
- C. Lung carcinoids and pleural effusion: Lung carcinoids are rare tumours that arise from the neuroendocrine cells of the bronchial epithelium. They are usually slow-growing and have a good prognosis. Pleural effusion is an accumulation of fluid in the pleural space, which can be caused by various conditions, such as infection, malignancy, heart failure, or trauma. Lung carcinoids and pleural effusion are not commonly associated, as lung carcinoids rarely metastasize or invade the pleura.
- D. Pneumoconiosis and clubbing: Pneumoconiosis is a group of lung diseases caused by inhalation of dust, such as coal, asbestos, or silica. Clubbing is a deformity of the fingers and nails that results from chronic hypoxia and inflammation in the lungs. Pneumoconiosis and clubbing are often associated, especially in advanced stages of the disease, but this association is not as strong as the one between bronchopulmonary aspergillosis and wheezing, as clubbing can also occur in other lung diseases, such as lung cancer, cystic fibrosis, or bronchiectasis.
_x000D_
_x000D_
_x000D_
_x000D_
-
Question 14 of 123
14. Question
According to current guidelines, which one of the below is the recommended duration of warfarin therapy in a patient recently diagnosed with his / her first pulmonary embolism (PE) in the presence of temporary risk factors?
CorrectIncorrectHint
The correct answer is E. 3 months. This is because the patient has a provoked pulmonary embolism (PE), which means that the pulmonary embolism (PE) occurred in the setting of a transient risk factor, such as surgery, immobilization, or hormonal therapy. According to current guidelines, patients with provoked pulmonary embolism (PE) should be treated with anticoagulation for a duration of 3 months, as long as they return to their pre-pulmonary embolism (PE) baseline. The risk of recurrent venous thromboembolism (VTE) in these patients is low, about 1% in the first year after stopping anticoagulation and 0.5% per year after.
_x000D_
The other answer options are incorrect for the following reasons:
_x000D_
- _x000D_
- A. Lifelong: Lifelong anticoagulation is recommended for patients with unprovoked pulmonary embolism (PE) or persistent risk factors, such as cancer, antiphospholipid syndrome, or thrombophilia. The risk of recurrent venous thromboembolism (VTE) in these patients is high, about 10% in the first year after stopping anticoagulation and 5% per year after. However, lifelong anticoagulation also increases the risk of bleeding, so the decision should be individualized and reassessed periodically.
- B. 6 months: 6 months of anticoagulation is an alternative option for patients with unprovoked pulmonary embolism (PE) who have a low or moderate risk of bleeding. However, it is not superior to 3 months of anticoagulation for patients with provoked pulmonary embolism (PE), and it may increase the risk of bleeding complications.
- C. 12 months: 12 months of anticoagulation is another alternative option for patients with unprovoked pulmonary embolism (PE) who have a low or moderate risk of bleeding. However, it is not superior to 3 months of anticoagulation for patients with provoked pulmonary embolism (PE), and it may increase the risk of bleeding complications.
- D. 4 – 6 weeks: 4 – 6 weeks of anticoagulation is insufficient for patients with PE, regardless of the provoking factors. It increases the risk of recurrent venous thromboembolism (VTE) by approximately 50%.
_x000D_
_x000D_
_x000D_
_x000D_
-
Question 15 of 123
15. Question
_x000D_
A 60-year-old lady presents to her primary care physician with complaints of night time sweats, nocturia, poor concentration and daytime somnolence.
To which one of the following conditions does this diagnosis predispose?CorrectIncorrectHint
The correct answer is D. Stroke. This is because the history of this lady is suggestive of sleep apnoea, a condition that causes repeated interruptions of breathing during sleep due to upper airway obstruction. Sleep apnoea can increase the risk of stroke by several mechanisms, such as:
_x000D_
- _x000D_
- Reducing the oxygen levels in the blood and tissues, which can damage the brain cells and blood vessels
- Increasing the blood pressure and heart rate, which can strain the heart and increase the risk of blood clots
- Causing insulin resistance and impaired glucose tolerance, which can worsen the metabolic control and increase the risk of diabetes and its complications
- Disrupting the sleep quality and quantity, which can affect the brain function and mood
_x000D_
_x000D_
_x000D_
_x000D_
_x000D_
The other options are incorrect for the following reasons:
_x000D_
- _x000D_
- A. Osteoporosis is a condition that causes the bones to become weak and brittle, increasing the risk of fractures. It is more common in women and people assigned female at birth (AFAB) after menopause, when the levels of oestrogen decrease. Oestrogen is a hormone that helps regulate the body temperature, the production of urine, and the cognitive functions. Low oestrogen levels can cause hot flashes, night sweats, nocturia, mood swings, memory problems, and fatigue. However, these symptoms are not specific to osteoporosis and can also occur in other conditions, such as sleep apnoea. Moreover, osteoporosis does not directly predispose to stroke, although it can increase the risk of falls and fractures, which can lead to disability and reduced mobility.
- B. Insulin sensitivity is the ability of the body to respond to insulin, a hormone that regulates the blood glucose levels. Insulin resistance is the opposite of insulin sensitivity, meaning that the body does not respond well to insulin and requires more of it to maintain normal glucose levels. Insulin resistance can lead to impaired glucose tolerance, diabetes, and its complications, such as cardiovascular disease and stroke. However, insulin sensitivity itself is not a condition, but a physiological parameter that can vary depending on several factors, such as diet, exercise, weight, age, and hormones. Therefore, it is not correct to say that insulin sensitivity predisposes to stroke, but rather that insulin resistance increases the risk of stroke.
- C. Hypotension is a condition that causes low blood pressure, which can lead to symptoms such as dizziness, fainting, blurred vision, nausea, and fatigue. Hypotension can have various causes, such as dehydration, blood loss, medications, heart problems, or autonomic nervous system disorders. Hypotension can also occur as a result of sleep apnoea, due to the reduced oxygen levels and increased carbon dioxide levels in the blood, which can cause the blood vessels to dilate and lower the blood pressure. However, hypotension itself does not predispose to stroke, but rather is a consequence of sleep apnoea. In fact, stroke is more likely to occur when the blood pressure is too high, as in hypertension, as this can damage the blood vessels and increase the risk of blood clots.
- E. Hypoglycaemia is a condition that causes low blood glucose levels, which can lead to symptoms such as sweating, trembling, hunger, confusion, seizures, and coma. Hypoglycaemia can have various causes, such as excessive insulin or oral hypoglycaemic agents, fasting, alcohol, liver disease, or hormonal deficiencies. Hypoglycaemia can also occur as a result of sleep apnoea, due to the increased insulin resistance and impaired glucose tolerance that sleep apnoea can cause. However, hypoglycaemia itself is not a risk factor for stroke, but rather a consequence of sleep apnoea. In fact, hyperglycaemia, or high blood glucose levels, is a more established risk factor for stroke, as it can damage the blood vessels and increase the risk of blood clots.
_x000D_
_x000D_
_x000D_
_x000D_
-
Question 16 of 123
16. Question
A 60-year-old gentleman presents to his Primary Care Physician with complaints of night time sweats, nocturia, poor concentration and daytime somnolence.
To which of the following conditions does this diagnosis predispose?
CorrectIncorrectHint
This history is typical of sleep apnoea._x000D_
Sleep apnoea is an independent risk factor for stroke (and death from all causes), and is associated with hypertension, impaired glucose tolerance (IGT) and insulin resistance._x000D_
Severe Sleep Apnea and Risk of Ischemic Stroke in the Elderly. Stroke 2006. 37. 2317-2321 -
Question 17 of 123
17. Question
A 20-year-old young lady developed pleural effusions, ascites and ankle swelling. Her Blood Pressure was 110/70 mmHg.
_x000D_
Investigations Revealed:
_x000D_
_x000D_ _x000D_
_x000D_ _x000D_ Serum Alanine Aminotransferase [ALT]
_x000D_
_x000D_
_x000D_ 19 U/L
_x000D_
_x000D_
_x000D_ (5 – 15)
_x000D_
_x000D_
_x000D_
_x000D_ _x000D_ Serum Total Bilirubin
_x000D_
_x000D_
_x000D_ 18 µmol/L
_x000D_
_x000D_
_x000D_ (1 – 22)
_x000D_
_x000D_
_x000D_
_x000D_ _x000D_ Serum Albumin
_x000D_
_x000D_
_x000D_ 20 g/L
_x000D_
_x000D_
_x000D_ (34 – 94)
_x000D_
_x000D_
_x000D_
_x000D_ _x000D_ Serum Total Cholesterol
_x000D_
_x000D_
_x000D_ 9.9 mmol/L
_x000D_
_x000D_
_x000D_ (<5.2)
_x000D_
_x000D_
_x000D_
_x000D_
_x000D_
What is the next most appropriate investigation to be done?
CorrectIncorrectHint
Low Serum Albumin and Elevated Serum Cholesterol would suggest Nephrotic Syndrome. This is defined as more than 3 gm of Proteinuria per day with Serum Albumin less than 25 g/L, Oedema, and Hypercholesterolaemia._x000D_
Other Complications Of Nephrotic Syndrome Include:_x000D_
? Susceptibility To Infection._x000D_
? Thromboses._x000D_
? Renal Failure, and_x000D_
? Protein Malnutrition._x000D_
The normal Blood Pressure makes Pre-Eclampsia unlikely. Besides, the Hypercholesterolaemia is the big clue. -
Question 18 of 123
18. Question
A 21-year-old gentleman presents with sudden onset left-sided pleuritic chest pain and dyspnoea. He has no past medical history of note and takes no regular medication.
_x000D_
On examination he looks in pain. He has a Respiratory Rate of 35/Minute and his Blood Pressure is 110/70 mmHg, with a Pulse Rate 95/Minute. His chest sounds appear normal.
_x000D_
Investigations Show:
_x000D_
_x000D_ _x000D_
_x000D_ _x000D_ Haemoglobin
_x000D_
_x000D_
_x000D_ 150 g/L
_x000D_
_x000D_
_x000D_ (135-180)
_x000D_
_x000D_
_x000D_
_x000D_ _x000D_ Total Leucocyte Count
_x000D_
_x000D_
_x000D_ 5.1 ×109/L
_x000D_
_x000D_
_x000D_ (4-10)
_x000D_
_x000D_
_x000D_
_x000D_ _x000D_ Platelet Count
_x000D_
_x000D_
_x000D_ 203 ×109/L
_x000D_
_x000D_
_x000D_ (150-400)
_x000D_
_x000D_
_x000D_
_x000D_ _x000D_ Serum Sodium
_x000D_
_x000D_
_x000D_ 142 mmol/L
_x000D_
_x000D_
_x000D_ (134-143)
_x000D_
_x000D_
_x000D_
_x000D_ _x000D_ Serum Potassium
_x000D_
_x000D_
_x000D_ 4.7 mmol/L
_x000D_
_x000D_
_x000D_ (3.5-5)
_x000D_
_x000D_
_x000D_
_x000D_ _x000D_ Serum Creatinine
_x000D_
_x000D_
_x000D_ 95 μmol/L
_x000D_
_x000D_
_x000D_ (60-120)
_x000D_
_x000D_
_x000D_
_x000D_ _x000D_ Chest X-Ray
_x000D_
_x000D_
_x000D_ Small Rim Of Air <2 cm On The Left Hand Side.
_x000D_
_x000D_
_x000D_
_x000D_
_x000D_
Which of the following is the most appropriate management?
CorrectIncorrectHint
Only Primary Pneumothoraces (less than 2 cm) which are not associated with dyspnoea should be managed with discharge and instructions to return if they become dyspnoeic._x000D_
In this case, aspiration should be considered._x000D_
If air aspiration is unsuccessful, a repeat can be attempted. If still unsuccessful then small bore chest drain insertion is the treatment of choice._x000D_
If there is a previous history of chest disease, the pneumothorax is considered to be secondary. If the patient is younger than 50 and the rim of air is less than 2 cm, aspiration can be attempted. If the patient is older, the rim is larger, or initial aspiration is unsuccessful then a chest drain should be inserted. -
Question 19 of 123
19. Question
A 21-year-old smoker presents to the Emergency Department with Right Sided Pleuritic Chest Pain and Dyspnoea. He has no previous medical history.
_x000D_
His Blood Pressure is 120/80 mmHg. A Chest X-Ray is done and confirms a Right Sided Pneumothorax with a Rim of 2.5 cm.
_x000D_
Aspiration was done and was successful. He is about to go on holiday abroad in three days time.
_x000D_
Which of the following advice would be given?
CorrectIncorrectHint
Atmospheric air drops during flights and increases in deep sea diving can cause recurrence of pneumothorax. This is due to expansion and rupture of pulmonary blebs. Many Commercial Airlines previously advised a 6-week interval between Pneumothorax and Air Travel, but this has now been amended to 1 week following full resolution._x000D_
However, the British Thoracic Society emphasises that the recurrence risk only significantly falls after 1 year, and therefore in the absence of a definitive surgical procedure patients might wish to defer travel until then. -
Question 20 of 123
20. Question
A 21-year-old smoker presents to the Emergency Department with right sided pleuritic chest pain and dyspnoea. He has no previous medical history.
_x000D_
His blood pressure is 120/70 mmHg. A chest x-ray is done and it confirms a right sided pneumothorax with a rim of 2.5 cm.
_x000D_
Which one of the following is the pathogenesis of pneumothorax?
CorrectIncorrectHint
The most likely pathogenesis of pneumothorax in the younger population is B. Ruptured apical bullae on the lung surface. A bulla is a thin-walled air-filled space in the lung that can be congenital or acquired. Smoking is a major risk factor for developing bullae, especially in young adults. When a bulla ruptures, it creates a communication between the alveolar space and the pleural space, allowing air to escape into the pleural cavity and cause a pneumothorax. This gentleman should be advised for smoking cessation.
_x000D_
The other options are less likely because they are either uncommon or have different mechanisms. Underlying emphysema is a chronic lung disease that causes destruction of the alveolar walls and enlargement of the air spaces. Emphysema can cause pneumothorax, but it is usually seen in older patients with a long history of smoking. Oesophageal rupture is a rare condition that occurs when the oesophagus tears due to trauma, vomiting, or instrumentation. Oesophageal rupture can cause pneumothorax, but it is usually associated with severe chest pain, fever, and subcutaneous emphysema. Leak from pulmonary alveoli is a general term that can describe various causes of pneumothorax, such as trauma, infection, or mechanical ventilation. However, it is not a specific pathogenesis of pneumothorax in the younger population. Underlying asthma is a chronic inflammatory disease of the airways that causes bronchoconstriction and mucus production. Asthma can cause pneumothorax, but it is usually due to a severe asthma attack that causes high intrathoracic pressure and alveolar rupture.
-
Question 21 of 123
21. Question
A 21-year-old young lady is brought to the Emergency Room by her boyfriend. She has been suffering from a heavy cold over the past few days and now has worsening wheeze and a dry cough.
_x000D_
She has a past history of asthma for which she takes high dose seretide and salbutamol as and when required.
_x000D_
On Examination, her blood pressure is 130/80 mmHg, and pulse rate is 98 beats per minute and regular. She has severe wheeze on auscultation of the chest and her respiratory rate is 34 breaths per minute.
_x000D_
Investigations Show:
_x000D_
_x000D_ _x000D_
_x000D_ _x000D_ pH
_x000D_
_x000D_
_x000D_ 7.44
_x000D_
_x000D_
_x000D_ (7.35-7.45)
_x000D_
_x000D_
_x000D_
_x000D_ _x000D_ PCO2
_x000D_
_x000D_
_x000D_ 4.6 kPa
_x000D_
_x000D_
_x000D_ (4.8-6.1)
_x000D_
_x000D_
_x000D_
_x000D_ _x000D_ pO2
_x000D_
_x000D_
_x000D_ 13.1 kPa
_x000D_
_x000D_
_x000D_ (10-13.3)
_x000D_
_x000D_
_x000D_
_x000D_ _x000D_ Peak Expiratory Flow Rate [PEFR]
_x000D_
_x000D_
_x000D_ 169 ml/min (32% Of Predicted)
_x000D_
_x000D_
_x000D_
_x000D_
_x000D_
Which one of the following features in her presentation is consistent with life-threatening asthma?
CorrectIncorrectHint
According to the British Thoracic Society (BTS) guidelines, the feature in the presentation of this lady that is consistent with life-threatening asthma is C. Peak flow 169 ml/minute. This is because a peak flow less than 33% of predicted or best is one of the criteria for life-threatening asthma. The other features of life-threatening asthma listed in the British Thoracic Society (BTS) asthma guidelines include:
_x000D_
- _x000D_
- Silent chest, cyanosis, or feeble respiratory effort
- Bradycardia, hypotension, dysrhythmia, or cardiac arrest
- Exhaustion, confusion, or coma
- Hypoxia (SpO2 <92% or PaO2 <8 kPa)
- Hypercapnia (PaCO2 >6 kPa)
- Normal PaCO2 in the presence of severe asthma (4.6-6 kPa)
_x000D_
_x000D_
_x000D_
_x000D_
_x000D_
_x000D_
_x000D_
This lady’s respiratory rate, pO2, pH, and pCO2 are not indicative of life-threatening asthma, although they may suggest moderate to severe asthma exacerbation.
_x000D_
Let us discuss all of the incorrect options one by one as below:
_x000D_
- _x000D_
- B. pO2 13.1 kPa: This is incorrect because a pO2 of 13.1 kPa is within the normal range (10-13.3 kPa) and does not indicate life-threatening asthma. In fact, hypoxia (pO2 <8 kPa) is one of the criteria for life-threatening asthma.
- D. pH 7.44: This is incorrect because a pH of 7.44 is within the normal range (7.35-7.45) and does not indicate life-threatening asthma. In fact, acidosis (pH <7.3) is one of the criteria for life-threatening asthma.
- E. pCO2 4.6 kPa: This is incorrect because a pCO2 of 4.6 kPa is slightly below the normal range (4.8-6.1 kPa) and does not indicate life-threatening asthma. In fact, hypercapnia (pCO2 >6 kPa) or normal pCO2 in the presence of severe asthma (4.6-6 kPa) are some of the criteria for life-threatening asthma.
- A. Respiratory rate 34 breaths per minute: This is incorrect because a respiratory rate of 34 breaths per minute is not a specific criterion for life-threatening asthma, although it may suggest moderate to severe asthma exacerbation. The British Thoracic Society (BTS) guidelines do not provide a cut-off value for respiratory rate, but they state that tachypnoea (increased respiratory rate) is a sign of respiratory distress and should be monitored closely.
_x000D_
_x000D_
_x000D_
_x000D_
-
Question 22 of 123
22. Question
A 22-year-old lady attends the Pulmonology Clinic for asthma review.
_x000D_
She remains significantly short of breath and has wheeze and coughing in the early hours of the morning. At her last appointment the Pulmonologist instigated treatment with Salmeterol as she was already on a stable dose of 400 mcg per day inhaled Beclomethasone.
_x000D_
On examination in the clinic her Blood Pressure: 100/70 mmHg; Pulse Rate: 70/Minute and Regular. She has scattered wheeze on auscultation of the chest and her PEFR (Peak Expiratory Flow Rate) is 380 (Predicted: 550). This is similar to prior to starting the Salmeterol from which she perceives she has gained no benefit.
_x000D_
According to Asthma Guidelines, which of the following is the most appropriate next step to be taken?
CorrectIncorrectHint
2011 Guidelines have since been updated in 2017 which state that patients should switch to a MART regime if not deriving benefit from a LABA and ICS rather than stopping the LABA and increasing the ICS as previously stated in 2011. _x000D_
New guidelines can be found here as well as more information on MART Regimes._x000D_
Given Asthma is a Chronic Condition, Low Dose Oral Steroids are considered a treatment of last resort at Step Five._x000D_
Guidance has recently been updated in 2020 on the management of asthma in children and young people though this does not apply here. -
Question 23 of 123
23. Question
A 22-year-old lady attends the pulmonology clinic for asthma review.
_x000D_
She remains significantly short of breath and has wheeze and coughing in the early hours of the morning. At her last appointment the pulmonologist instigated treatment with salmeterol as she was already on a stable dose of 400 mcg per day inhaled beclomethasone.
_x000D_
On examination in the clinic, her blood pressure is 100/70 mmHg, pulse rate is 70 beats per minute and regular. She has scattered wheeze on auscultation of the chest and her peak expiratory flow rate (PEFR) is 380 (predicted: 550). This is similar to prior to starting the salmeterol from which she perceives she has gained no benefit.
_x000D_
According to asthma guidelines, which one of the following is the most appropriate next step to be taken?
CorrectIncorrectHint
According to the latest Global Initiative for Asthma (GINA) guidelines, which were updated in 2023, the most appropriate next step for this lady is to switch to maintenance and reliever therapy (MART) regime with low dose inhaled corticosteroids (ICS) (option A). Maintenance and reliever therapy (MART) regime is an asthma treatment plan where the patient uses one combination inhaler that contains both inhaled corticosteroids (ICS) and a fast-acting long-acting beta2-agonist (LABA) such as formoterol for both daily maintenance therapy and the relief of symptoms as required. Maintenance and reliever therapy (MART) regime can improve asthma control, reduce exacerbations, and lower the overall steroid load compared to using separate preventer and reliever inhalers.
_x000D_
The other options are not recommended by the Global Initiative for Asthma (GINA) guidelines for this lady. Adding montelukast (option B) is only suggested for patients who have allergic rhinitis or exercise-induced bronchoconstriction, or who cannot tolerate long-acting beta2-agonist (LABA). Adding theophylline (option C) is not preferred due to its narrow therapeutic window and potential side effects. Adding low dose oral steroids (option D) is only indicated for patients with severe asthma who have frequent or serious exacerbations despite high dose inhaled corticosteroids (ICS) and long-acting beta2-agonist (LABA). Stopping salmeterol and increasing inhaled corticosteroids (ICS) to 800 mcg/day (option E) is not advisable as it may increase the risk of exacerbations and adverse effects without improving asthma control.
_x000D_
_x000D_
The 2023 update of Global Initiative for Asthma (GINA) guidelines focused on the management of asthma in children and young people, and did not change the recommendations for adults with asthma.
-
Question 24 of 123
24. Question
A 22-year-old young gentleman presents to the Emergency Department with sudden onset left sided pleuritic chest pain. He has had a cough over the past few days and says the pain came on after a coughing fit.
_x000D_
On examination, his blood pressure is 150/80 mmHg, pulse rate is 82 beats per minute and regular, his saturations are 95% on room air. Chest sounds appear normal.
_x000D_
Investigations Show:
_x000D_
_x000D_ _x000D_
_x000D_ _x000D_ pH
_x000D_
_x000D_
_x000D_ 7.41
_x000D_
_x000D_
_x000D_ (7.35-7.45)
_x000D_
_x000D_
_x000D_
_x000D_ _x000D_ pCO2
_x000D_
_x000D_
_x000D_ 4.9 kPa
_x000D_
_x000D_
_x000D_ (4.8-6.1)
_x000D_
_x000D_
_x000D_
_x000D_ _x000D_ pO2
_x000D_
_x000D_
_x000D_ 10.4 kPa
_x000D_
_x000D_
_x000D_ (10-13.3)
_x000D_
_x000D_
_x000D_
_x000D_ _x000D_ Chest X-Ray
_x000D_
_x000D_
_x000D_ Small Left Sided Pneumothorax (<5%)
_x000D_
_x000D_
_x000D_
_x000D_
_x000D_
Which one of the following is the most appropriate management for this gentleman?
CorrectIncorrectHint
The correct answer is D. Discharge and review in the clinic in two to three weeks. This is the most appropriate management for a small primary spontaneous pneumothorax (PSP), which is a condition where air leaks into the pleural space and causes partial collapse of the lung. Primary spontaneous pneumothorax (PSP) usually occurs in young, healthy, and tall individuals, and is often triggered by coughing, sneezing, or exercise. Primary spontaneous pneumothorax (PSP) can cause chest pain, shortness of breath, and reduced oxygen levels.
_x000D_
The other answer options are not optimal because:
_x000D_
- _x000D_
- A. Pleural aspiration is a procedure that involves inserting a needle or a catheter between the ribs and removing the air from the pleural space. This can help the lung re-expand faster and prevent recurrence. However, pleural aspiration is not necessary for a small primary spontaneous pneumothorax (<15%), as the air can be absorbed naturally over time. Pleural aspiration is only recommended for larger or more symptomatic primary spontaneous pneumothoraces (PSPs), or if the patient prefers a more active intervention.
- B. Discharge and review in 24 hours is not a sufficient follow-up for a primary spontaneous pneumothorax (PSP), as there is a risk of recurrence or worsening of the condition. This gentleman should be monitored for at least two to three weeks until the primary spontaneous pneumothorax (PSP) is resolved. He should also be advised to avoid smoking, flying, diving, or high-altitude activities until his primary spontaneous pneumothorax (PSP) is resolved. He should also be instructed to seek medical attention if he develops worsening chest pain, breathlessness, or cyanosis.
- C. Admit for overnight oxygen therapy is not indicated for a small primary spontaneous pneumothorax (PSP), as there is insufficient evidence to support its effectiveness and safety. Oxygen therapy may increase the rate of air reabsorption and lung expansion, but it may also cause oxygen toxicity, hypercapnia, or fire hazard. Oxygen therapy may be considered for larger or more symptomatic primary spontaneous pneumothoraces (PSPs), or if the patient has hypoxia.
- E. Chest drain is a procedure that involves inserting a flexible tube into the pleural space and attaching it to a device that continuously removes air from the chest cavity. This can help the lung re-expand and heal. However, chest drain is more invasive and costly than pleural aspiration, and is associated with more pain, complications, and longer hospital stay. Chest drain is only indicated if pleural aspiration fails in a large or symptomatic primary spontaneous pneumothorax (PSP), or if the patient has a secondary spontaneous pneumothorax (SSP), which is a complication of underlying lung disease.
_x000D_
_x000D_
_x000D_
_x000D_
_x000D_
Therefore, based on the clinical presentation and the guidelines, the most appropriate management for this gentleman is to discharge him and review him in the clinic in two to three weeks. He should be advised to avoid smoking, flying, diving, or high-altitude activities until the primary spontaneous pneumothorax (PSP) is resolved. He should also be instructed to seek medical attention if he develops worsening chest pain, breathlessness, or cyanosis.
-
Question 25 of 123
25. Question
A 23-year-old lady presents to the Emergency Department with a two-hour history of chest tightness, dyspnoea, tingling in her hands and light-headedness. She has neither past medical history nor family history of note.
_x000D_
Examination is unremarkable aside from an Elevated Respiratory Rate. Her Pulse Oximetry shows Saturations of 96% on Room Air, which do fall when she walks across the room. A Chest X-Ray is also normal. An Arterial Blood Gas sample (On Air and At Rest) is obtained and the results are as follows:
_x000D_
_x000D_ _x000D_
_x000D_ _x000D_ pH
_x000D_
_x000D_
_x000D_ 7.55
_x000D_
_x000D_
_x000D_ (7.36 – 7.44)
_x000D_
_x000D_
_x000D_
_x000D_ _x000D_ pCO2
_x000D_
_x000D_
_x000D_ 2.3kPa
_x000D_
_x000D_
_x000D_ (4.7 – 6.0)
_x000D_
_x000D_
_x000D_
_x000D_ _x000D_ pO2
_x000D_
_x000D_
_x000D_ 8.8kPa
_x000D_
_x000D_
_x000D_ (11.3 – 12.6)
_x000D_
_x000D_
_x000D_
_x000D_ _x000D_ pHCO3
_x000D_
_x000D_
_x000D_ 27 mmol/L
_x000D_
_x000D_
_x000D_ (20 – 28)
_x000D_
_x000D_
_x000D_
_x000D_
_x000D_
What is the most likely diagnosis?
CorrectIncorrectHint
This lady’s Blood Gas Sample shows Respiratory Alkalosis, as demonstrated by the raised pH, low pCO2 and normal Bicarbonate. Her pO2 is over 8kPa, and she is therefore not technically in Type 1 Respiratory Failure, but it is important to realise she is working hard to maintain this Oxygenation. The most likely cause is an Acute Pulmonary Embolism given the relative Hypoxia, and saturation dropping on exertion. Given her age she may be taking the Oral Contraceptive Pill, which would be a risk factor._x000D_
In Acute Asthma the pCO2 may be low, as the patient has an elevated Respiratory Rate, however in this instance there should be a history of the condition, clinical signs or a reduced pO2._x000D_
Hyperventilation often demonstrates a picture of Respiratory Alkalosis on analysis, however she should have a normal pO2 and her saturations should not fall on exertion._x000D_
Respiratory Muscle Disease typically results in a Respiratory Acidosis._x000D_
Volume Depletion (from a number of causes) would typically cause a Metabolic Alkalosis. -
Question 26 of 123
26. Question
A 23-year-old lady presents to the Emergency Department with a two-hour history of chest tightness, dyspnoea, tingling in her hands and light-headedness. She has neither past medical history nor family history of note.
_x000D_
Examination is unremarkable aside from an elevated respiratory rate. Her pulse oximetry shows saturations of 96% on room air, which do fall when she walks across the room. A chest x-ray is also normal. An arterial blood gas sample (on air and at rest) is obtained and the results are as follows:
_x000D_
_x000D_ _x000D_
_x000D_ _x000D_ pH
_x000D_
_x000D_
_x000D_ 7.55
_x000D_
_x000D_
_x000D_ (7.36 – 7.44)
_x000D_
_x000D_
_x000D_
_x000D_ _x000D_ pCO2
_x000D_
_x000D_
_x000D_ 2.3kPa
_x000D_
_x000D_
_x000D_ (4.7 – 6.0)
_x000D_
_x000D_
_x000D_
_x000D_ _x000D_ pO2
_x000D_
_x000D_
_x000D_ 8.8kPa
_x000D_
_x000D_
_x000D_ (11.3 – 12.6)
_x000D_
_x000D_
_x000D_
_x000D_ _x000D_ pHCO3
_x000D_
_x000D_
_x000D_ 27 mmol/L
_x000D_
_x000D_
_x000D_ (20 – 28)
_x000D_
_x000D_
_x000D_
_x000D_
_x000D_
Which one of the following is the most likely diagnosis?
CorrectIncorrectHint
Pulmonary embolism (PE) is a condition in which a blood clot travels to the lungs and blocks the blood flow, causing impaired gas exchange and hypoxia. Pulmonary embolism (PE) can present with chest tightness, dyspnoea, tingling, and light-headedness, especially if the clot is large or affects the right ventricle. Pulmonary embolism (PE) can also cause a drop in oxygen saturation on exertion, as the ventilation-perfusion mismatch worsens with increased demand. Pulmonary embolism (PE) can be triggered by risk factors such as oral contraceptive use, immobilization, surgery, or cancer.
_x000D_
The arterial blood gas (ABG) analysis of pulmonary embolism (PE) typically shows a respiratory alkalosis, which is a condition in which the blood becomes too alkaline (basic) due to a loss of carbon dioxide (CO2) from hyperventilation. Hyperventilation is a compensatory mechanism to increase oxygen delivery and decrease CO2 levels in the blood. The arterial blood gas (ABG) of pulmonary embolism (PE) may show a high pH, a low pCO2, and normal bicarbonate. The pO2 may be low or normal, depending on the severity of the pulmonary embolism (PE) and the degree of compensation.
_x000D_
The other options are not consistent with this lady’s presentation and arterial blood gas (ABG) for the following reasons:
_x000D_
- _x000D_
- A. Hyperventilation (psychogenic) is a condition that occurs when a person breathes faster or deeper than normal, causing a decrease in CO2 levels in the blood. Hyperventilation can be caused by psychological factors, such as anxiety, panic, stress, or fear. Hyperventilation can cause chest tightness, dyspnoea, tingling, and light-headedness, similar to pulmonary embolism (PE). However, hyperventilation does not cause hypoxia or a drop in saturation on exertion, as the oxygen delivery is not impaired. The pO2 and the alveolar-arterial gradient are usually normal in hyperventilation.
- B. Volume depletion is a condition that occurs when a person loses fluid from the body, such as from vomiting, diarrhoea, or bleeding. Volume depletion can cause hypotension, tachycardia, and decreased urine output. Volume depletion does not cause hyperventilation or respiratory alkalosis, but rather a metabolic alkalosis, which is a condition in which the blood becomes too alkaline due to a loss of acid or a gain of base. The arterial blood gas (ABG) of volume depletion may show a high pH, a normal or high pCO2, and high bicarbonate.
- C. Respiratory muscle disease is a condition that affects the muscles that control breathing, such as the diaphragm, intercostals, or accessory muscles. Respiratory muscle disease can cause dyspnoea, fatigue, and respiratory failure. Respiratory muscle disease does not cause hyperventilation or respiratory alkalosis, but rather hypoventilation or respiratory acidosis, which is a condition in which the blood becomes too acidic due to a build-up of CO2 from inadequate ventilation. The arterial blood gas (ABG) of respiratory muscle disease may show a low pH, a high pCO2, and high bicarbonate.
- D. Acute asthma attack is a condition that occurs when the airways become inflamed, narrowed, and filled with mucus, causing difficulty breathing. Acute asthma attack can cause wheezing, coughing, chest tightness, and hypoxia. Acute asthma attack does not cause hyperventilation or respiratory alkalosis, but rather hyperventilation or respiratory acidosis in severe cases, when the ventilation-perfusion mismatch and the air trapping become significant. The arterial blood gas (ABG) of acute asthma attack may show a low or normal pH, a low or high pCO2, and normal or high bicarbonate.
_x000D_
_x000D_
_x000D_
_x000D_
-
Question 27 of 123
27. Question
A 24-year-old University Student presents to the Respiratory Clinic with increased shortness of breath, a dry cough, left sided pleuritic chest pain and extreme lethargy over three weeks.
_x000D_
There is significant medical history but says a few other people have been ill in his work place over the past few weeks.
_x000D_
On examination, he is Pyrexial: 38.6°C, Blood Pressure: 100/60 mmHg; Pulse Rate: 85/minute and regular. There is scattered wheeze, more marked on the right than the left.
_x000D_
Investigations Reveal:
_x000D_
_x000D_ _x000D_
_x000D_ _x000D_ Haemoglobin
_x000D_
_x000D_
_x000D_ 88 g/L
_x000D_
_x000D_
_x000D_ (115-160)
_x000D_
_x000D_
_x000D_
_x000D_ _x000D_ Total Leucocyte Count
_x000D_
_x000D_
_x000D_ 12.2 ×109/L
_x000D_
_x000D_
_x000D_ (4-11)
_x000D_
_x000D_
_x000D_
_x000D_ _x000D_ Platelet Count
_x000D_
_x000D_
_x000D_ 172 ×109/L
_x000D_
_x000D_
_x000D_ (150-400)
_x000D_
_x000D_
_x000D_
_x000D_ _x000D_ Serum Sodium
_x000D_
_x000D_
_x000D_ 140 mmol/L
_x000D_
_x000D_
_x000D_ (135-146)
_x000D_
_x000D_
_x000D_
_x000D_ _x000D_ Serum Potassium
_x000D_
_x000D_
_x000D_ 4.1 mmol/L
_x000D_
_x000D_
_x000D_ (3.5-5)
_x000D_
_x000D_
_x000D_
_x000D_ _x000D_ Serum Creatinine
_x000D_
_x000D_
_x000D_ 125 µmol/L
_x000D_
_x000D_
_x000D_ (79-118)
_x000D_
_x000D_
_x000D_
_x000D_ _x000D_ ESR
_x000D_
_x000D_
_x000D_ 81 mm/hr
_x000D_
_x000D_
_x000D_ (<10)
_x000D_
_x000D_
_x000D_
_x000D_
_x000D_
Chest X-Ray – left Lower Lobe Pneumonia.
_x000D_
Blood Film – Schistocytes.
_x000D_
Which of the following is the most likely causative organism in this case?
CorrectIncorrectHint
Mycoplasma pneumonia usually affects people younger than
-
Question 28 of 123
28. Question
A 24-year-old University Student, who recently returned from an Educational Tour in Valletta was admitted with a two-day history of fever, generalised lymphadenopathy and a macular rash over the trunk and legs.
_x000D_
Which of the following is the most likely diagnosis?
CorrectIncorrectHint
Infectious Mononucleosis occurs most commonly in adolescents and young adults._x000D_
Clinical features occur after a two to five week incubation period and include:_x000D_
? Fever_x000D_
? Malaise_x000D_
? Pharyngitis_x000D_
? Lymphadenopathy._x000D_
Rashes occur more commonly in patients who have received penicillin or ampicillin. -
Question 29 of 123
29. Question
A 27-year-old lady is referred to the Respiratory Clinic as two of the children in her Kindergarten Class have recently been diagnosed with Tuberculosis.
_x000D_
Clinical Examination reveals a Blood Pressure of 120/70 mmHg, Pulse Rate is 70/Minute and Regular. Her chest is clear. A Mantoux Test results in a reaction measured at 17 mm.
_x000D_
Which of the following is the correct way to manage her?
CorrectIncorrectHint
The Mantoux Test replaced the Heaf Test in 2005 in the United Kingdom. One of its uses in for patients who have had close contact with a person known to have Tuberculosis._x000D_
The injection site should be reviewed 48 ? 72 hours following Intradermal Inoculation Of Tuberculin. The left forearm is typically used. Only the induration, not surrounding erythema, is used in the measurement and the longest diameter is measured in millimetres:_x000D_
? Less than 6 mm – negative test, previously unvaccinated individuals can be given the BCG (within three months) provided there are no contraindications._x000D_
? More than 6 mm but less than 15 mm – hypersensitive to tuberculin protein (may be due to previous TB infection, BCG, or atypical mycobacteria). Patients are not given the BCG if part of an immunisation programme. However, in other contexts (e.g. immigrant screening and contact tracing), further investigation should and follow-up may be indicated._x000D_
? More than 15 mm – strongly hypersensitive to tuberculin, suggestive of TB infection. Patients should be referred for further investigation and treatment._x000D_
The reaction to tuberculin protein may be suppressed by viral infections, live viral vaccines, sarcoidosis, corticosteroids, immunosuppression, severe tuberculous disease and poor nutrition._x000D_
Bronchoscopy would not usually be considered unless other investigations have proved inconclusive._x000D_
Anti-tuberculous therapy is commenced only after infection is confirmed, and patients with a positive Mantoux do not require BCG vaccination. -
Question 30 of 123
30. Question
A 27-year-old lady is referred to the respiratory clinic as two of the children in her kindergarten class have recently been diagnosed with tuberculosis.
_x000D_
Clinical examination reveals a blood pressure of 120/70 mmHg, and a pulse rate of 70 beats per minute and regular. Her chest is clear. A Mantoux test results in a reaction measured at 17 mm.
_x000D_
Which one of the following is the correct way to manage her?
CorrectIncorrectHint
The correct way to manage this lady is to arrange for sputum samples to be collected, which is option C. This is because she has a positive Mantoux test result of 17 mm, which suggests that she has latent tuberculosis infection (LTBI) and may have been exposed to tuberculosis (TB) through close contact with the children in her kindergarten class (see the NOTE below). Sputum samples are used to confirm the diagnosis of tuberculosis (TB) by detecting the presence of Mycobacterium tuberculosis, the bacteria that causes tuberculosis (TB). If the sputum samples are positive, she will need to start anti-tuberculosis chemotherapy to prevent the progression of latent tuberculosis infection (LTBI) to active tuberculosis (TB) disease.
_x000D_
The incorrect options can be ruled out with the following explanations:
_x000D_
- _x000D_
- Option A is incorrect because she is not immune to tuberculosis (TB) and requires further investigation and treatment. A positive Mantoux test result indicates that she has been infected with tuberculosis (TB) bacteria at some point in her life, but it does not mean that she has developed immunity to tuberculosis (TB). She may still develop active tuberculosis (TB) disease in the future, especially if her immune system is weakened by other factors. Therefore, she should not be reassured and that no further action is necessary is not an option.
- Option B is incorrect because she does not need a bronchoscopy. A bronchoscopy is a procedure that involves inserting a flexible tube with a camera and a light into the airways to examine the lungs and collect samples of lung tissue or fluid. It is usually done for patients who have signs or symptoms of pulmonary tuberculosis (TB), such as cough, fever, weight loss, or chest pain, or for patients who have abnormal chest x-rays. However, she has none of these signs or symptoms and her chest is clear. Therefore, a bronchoscopy is unnecessary and invasive for her condition.
- Option D is incorrect because she does not need to be vaccinated with Bacillus Calmette–Guérin (BCG). Bacillus Calmette–Guérin (BCG) is a vaccine that contains a weakened strain of Mycobacterium bovis, a bacterium related to tuberculosis (TB). It is used to prevent severe forms of tuberculosis (TB) in children, especially in countries where tuberculosis (TB) is endemic. However, it is not recommended for adults with latent tuberculosis infection (LTBI), as it has no proven benefit and may cause adverse reactions. Moreover, she has already been infected with tuberculosis (TB) bacteria, so Bacillus Calmette–Guérin (BCG) vaccination will not protect her from tuberculosis (TB).
- Option E is incorrect because she does not need to start anti-tuberculosis chemotherapy without confirmation of tuberculosis (TB) infection. Anti-tuberculosis chemotherapy is a combination of drugs that are used to treat tuberculosis (TB) disease. It is usually given for at least six months and requires close monitoring and adherence. It can also cause side effects such as nausea, vomiting, liver damage, or nerve damage. Therefore, it should not be started without a definitive diagnosis of tuberculosis (TB), which can be obtained from sputum samples or other tests. Starting anti-tuberculosis chemotherapy without confirmation of tuberculosis (TB) infection may lead to unnecessary exposure to drugs, drug resistance, or toxicity.
_x000D_
_x000D_
_x000D_
_x000D_
_x000D_
NOTE:
_x000D_
In the United Kingdom, the Mantoux test took over from the Heaf test in 2005. One of its primary applications is for individuals who have been in close proximity to someone known to have tuberculosis.
_x000D_
After the intradermal inoculation of tuberculin, the injection site should be assessed between 48 to 72 hours. Typically, the left forearm is utilized for this purpose. During measurement, only the induration (not the surrounding erythema) is considered, and the longest diameter is measured in millimeters. The interpretation of the test results is as follows:
_x000D_
– A measurement of less than 6 mm indicates a negative test. Previously unvaccinated individuals can receive the Bacillus Calmette–Guérin (BCG) vaccine within three months, provided there are no contraindications.
_x000D_
– A measurement of more than 6 mm but less than 15 mm suggests hypersensitivity to tuberculin protein. This hypersensitivity may be due to previous tuberculosis infection, BCG vaccination, or exposure to atypical mycobacteria. If the individual is part of an immunization program, they should not receive the BCG vaccine. However, in other situations such as immigrant screening or contact tracing, further investigation and possible follow-up may be necessary.
_x000D_
– A measurement of more than 15 mm strongly indicates hypersensitivity to tuberculin, which is highly suggestive of tuberculosis (TB) infection. Patients with this result should be referred for further investigation and appropriate treatment.
_x000D_
It is important to note that the reaction to tuberculin protein can be suppressed by various factors including viral infections, live viral vaccines, sarcoidosis, corticosteroids, immunosuppression, severe tuberculous disease, and poor nutrition. These factors should be considered when interpreting the results of the Mantoux test.
-
Question 31 of 123
31. Question
A 26-year-old lady presents to the pulmonology clinic some ten months after the birth of her first child.
_x000D_
She has suffered increasing shortness of breath over the past few weeks and months, so much so that she can barely walk up stairs or to the bus stop at the end of her street.
_x000D_
On examination, she looks short of breath at rest. Her jugular venous pressure (JVP) is elevated and there is mild bilateral ankle swelling. Her lung fields are clear.
_x000D_
Investigations Show:
_x000D_
_x000D_ _x000D_
_x000D_ _x000D_ Haemoglobin
_x000D_
_x000D_
_x000D_ 130 g/L
_x000D_
_x000D_
_x000D_ (115-160)
_x000D_
_x000D_
_x000D_
_x000D_ _x000D_ White Blood Cell Count
_x000D_
_x000D_
_x000D_ 7.5 ×109/L
_x000D_
_x000D_
_x000D_ (4-11)
_x000D_
_x000D_
_x000D_
_x000D_ _x000D_ Platelet Count
_x000D_
_x000D_
_x000D_ 210 ×109/L
_x000D_
_x000D_
_x000D_ (150-400)
_x000D_
_x000D_
_x000D_
_x000D_ _x000D_ Serum Sodium
_x000D_
_x000D_
_x000D_ 139 mmol/L
_x000D_
_x000D_
_x000D_ (135-146)
_x000D_
_x000D_
_x000D_
_x000D_ _x000D_ Serum Potassium
_x000D_
_x000D_
_x000D_ 4.4 mmol/L
_x000D_
_x000D_
_x000D_ (3.5-5)
_x000D_
_x000D_
_x000D_
_x000D_ _x000D_ Serum Creatinine
_x000D_
_x000D_
_x000D_ 117 µmol/L
_x000D_
_x000D_
_x000D_ (79-118)
_x000D_
_x000D_
_x000D_
_x000D_
_x000D_
Echocardiogram shows evidence of pulmonary hypertension. VQ scan shows no evidence of pulmonary embolism.
_x000D_
Which one of the following is the most appropriate initial management for this lady?
CorrectIncorrectHint
This lady is presenting with primary pulmonary hypertension, a condition that often manifests in women after giving birth to their first child. In the initial stages of treatment, calcium antagonists are considered as the first-line therapy, but only for patients who do not exhibit signs of right-sided heart failure. However, when selecting the initial therapy, an endothelin receptor antagonist would be the preferred choice. Examples of endothelin receptor antagonists include bosentan and ambrisentan, both of which have shown significant efficacy in reducing pulmonary artery pressure (PAP). It is important to note that these medications can have adverse effects such as peripheral oedema, and regular monitoring of liver function tests (LFTs) is recommended.
_x000D_
Beta blockers, despite their ability to lower peripheral blood pressure, are not recommended for the management of primary pulmonary hypertension. Their main effect is not particularly beneficial in this condition.
_x000D_
It is worth mentioning that calcium antagonists are effective in only a small percentage of patients, around 10-15%, and their use is only recommended if there is no evidence of right-sided heart failure.
_x000D_
Phosphodiesterase type 5 (PDE-5) inhibitors are typically used in cases of mild disease or in individuals with stage III or IV NYHA functional status, often in combination with endothelin receptor antagonists.
_x000D_
Nebulized or subcutaneous prostacyclin analogues, while effective, are less convenient treatment options compared to oral medications. Therefore, they are usually reserved for combination therapy with oral drugs.
_x000D_
In summary, this lady’s presentation suggests primary pulmonary hypertension, which commonly occurs in women after giving birth to their first child. The initial therapy choice depends on the absence of right-sided heart failure, with endothelin receptor antagonists being the preferred option. Calcium antagonists may be considered if right-sided heart failure is not present, although their efficacy is limited to a small percentage of patients. Beta blockers are not recommended. PDE-5 inhibitors are utilized in specific stages of the disease, and nebulized or subcutaneous prostacyclin analogues are usually reserved for combination therapy with oral medications due to their inconvenience.
-
Question 32 of 123
32. Question
A 27-year-old gentleman presents to the Emergency Department with complaint of shortness of breath.
_x000D_
One week ago he developed influenza and has become more short of breath and fatigued in the last 24 hours. His temperature is 101.3°F, his SaO2 is 90% on 2 litres of oxygen, his blood pressure is 100/60 mmHg, and heart rate is 120 beats per minute. A chest x-ray shows patchy consolidation.
_x000D_
Which one of the following antibiotic therapies should be selected for this gentleman?
CorrectIncorrectHint
This gentleman has been diagnosed with community-acquired pneumonia (CAP). Considering his recent history of influenza, Staphylococcus aureus might be considered as the underlying organism, although it is an uncommon cause of community-acquired pneumonia (CAP) in the United Kingdom. Staphylococcus aureus is more prevalent during the winter months, and approximately 40% of cases coincide with influenza-like symptoms. About 3% of influenza cases develop pneumonia, and of those admitted, 10% are confirmed to be caused by Staphylococcus aureus.
_x000D_
In most cases of community-acquired pneumonia (CAP), confirmation through chest radiography is recommended before initiating antibiotic treatment. However, if patients are critically ill, they should be treated based on presumptive diagnosis. Antibiotic therapy should always be initiated within 4 hours of presentation.
_x000D_
Community-acquired pneumonia caused by Staphylococcus aureus is more likely to present with multilobar shadowing, cavitation, pneumatoceles, and spontaneous pneumothorax compared to other organisms. However, there are no specific radiographic features that can definitively predict the likely pathogen. Therefore, the general treatment guidelines for community-acquired pneumonia (CAP) should be followed until the organism is identified. Staphylococcus aureus is associated with high mortality, so if suspected, initial treatment should be for severe community-acquired pneumonia (CAP) (see details below).
_x000D_
Low severity community-acquired pneumonia (CAP) (CURB 0-1) can be treated with oral amoxicillin 500 mg three times daily. CURB 2 community-acquired pneumonia (CAP) should be treated with oral amoxicillin 500 mg – 1 gm three times daily and oral clarithromycin 500 mg twice daily. Alternative options are available for patients allergic to any of the above combinations. High severity community-acquired pneumonia (CAP) (CURB 3-5) should be promptly treated with intravenous co-amoxiclav 1.2 gm three times daily and oral clarithromycin 500 mg twice daily.
_x000D_
For patients with low and moderate severity community-acquired pneumonia (CAP), the oral route is recommended. Patients initially treated with parenteral antibiotics should be switched to an oral regimen once clinical improvement is observed and they have been afebrile for at least 24 hours. In most cases of uncomplicated community-acquired pneumonia (CAP), a 7-day course of antibiotics is recommended. For high severity pneumonia cases where the organism is unidentified, 7-10 days of treatment is indicated, and the duration may be extended to 14-21 days if clinically necessary.
_x000D_
If Staphylococcus aureus is identified, the treatment plan should be adjusted accordingly. Non-methicillin-resistant Staphylococcus aureus (MRSA) organisms should be treated with flucloxacillin and/or rifampicin. For penicillin-allergic patients, teicoplanin and rifampicin can be considered as an alternative. Methicillin-resistant Staphylococcus aureus (MRSA) should be treated with vancomycin. Prolonged antibiotic therapy is recommended in these cases.
-
Question 33 of 123
33. Question
A 26-year-old asthmatic lady is admitted to the Emergency Department with acute severe asthma.
_x000D_
Which one of the following statements regarding acute severe asthma is correct?
CorrectIncorrectHint
The correct answer is C. A high inspired oxygen concentration should be used routinely. This is the correct statement regarding acute severe asthma according to National Institute for Health and Care Excellence (NICE) guidelines.
_x000D_
The reason for this is that people with acute severe asthma are at risk of hypoxia (low oxygen level in the blood) and respiratory failure, which can be life-threatening. Therefore, they should receive high-flow oxygen therapy (at least 15 litres per minute) via a non-rebreather mask to maintain their oxygen saturation above 94%.
_x000D_
The other statements are incorrect according to National Institute for Health and Care Excellence (NICE) guidelines, as follows:
_x000D_
- _x000D_
- A. Inhaled salmeterol is indicated as first-line therapy: This is incorrect, as salmeterol is a long-acting beta-2 agonist (LABA) that is used for maintenance therapy of asthma, not for acute exacerbations. The first-line therapy for acute severe asthma is inhaled short-acting beta-2 agonist (SABA) such as salbutamol, which provides rapid bronchodilation and relief of symptoms.
- B. Pulsus paradoxus is a reliable sign of severity: This is incorrect, as pulsus paradoxus is a sign of increased intrathoracic pressure due to severe airway obstruction, but it is not a reliable indicator of the severity of asthma exacerbations. Pulsus paradoxus is defined as a drop in systolic blood pressure of more than 10 mmHg during inspiration, and it can be difficult to measure accurately in clinical practice. The severity of asthma exacerbations should be assessed based on the person’s symptoms, peak expiratory flow rate, oxygen saturation, and arterial blood gases.
- D. Normal arterial pCO2 is reassuring: This is incorrect, as normal arterial pCO2 (partial pressure of carbon dioxide in the blood) in a person with acute severe asthma may indicate impending respiratory failure, rather than adequate ventilation. This is because the person may have exhausted their respiratory reserve and may not be able to increase their minute ventilation to compensate for the increased carbon dioxide production due to hypoxia and acidosis. A normal or high pCO2 in a person with acute severe asthma should prompt urgent admission to an intensive care unit for possible mechanical ventilation.
- E. Agitation should be managed with a benzodiazepine: This is incorrect, as benzodiazepines are sedatives that can depress the respiratory drive and worsen the hypoxia and hypercapnia in a person with acute severe asthma. Agitation in a person with acute severe asthma may be a sign of hypoxia, acidosis, or anxiety, and it should be managed by correcting the underlying cause, such as increasing the oxygen therapy, administering bronchodilators, or providing reassurance. Benzodiazepines should be avoided unless there is a clear indication, such as status epilepticus or alcohol withdrawal
_x000D_
_x000D_
_x000D_
_x000D_
-
Question 34 of 123
34. Question
A 24-year-old university student, who recently returned from an educational tour in Valletta, was admitted with a two-day history of fever, generalised lymphadenopathy and a macular rash over the trunk and legs.
_x000D_
Which one of the following is the most likely diagnosis?
CorrectIncorrectHint
The most likely diagnosis for this student is A. Infectious mononucleosis. This is a viral infection caused by the Epstein-Barr virus, which is transmitted by saliva. It typically affects young adults and causes fever, sore throat, swollen lymph nodes, and a rash that may appear on the trunk and limbs. The rash may be more prominent in people who take amoxicillin or other antibiotics.
_x000D_
The other options are less likely because they do not match the clinical features of the case. Tuberculosis is a bacterial infection that causes chronic cough, weight loss, night sweats, and may affect various organs. It is usually associated with granulomatous inflammation and caseating necrosis. Sarcoidosis is an inflammatory disorder of unknown cause that also forms granulomas in various tissues, especially the lungs and lymph nodes. It may cause fever, fatigue, dyspnoea, and skin lesions such as erythema nodosum or lupus pernio. Actinomycosis is a chronic bacterial infection that causes abscesses and fistulas in the mouth, neck, chest, abdomen, or pelvis. It is often associated with poor oral hygiene or trauma. Familial Mediterranean fever is a genetic disorder that causes recurrent episodes of fever, abdominal pain, chest pain, and joint pain. It is more common in people of Mediterranean origin and may lead to amyloidosis.
-
Question 35 of 123
35. Question
A 24-year-old young gentleman is admitted with wheezing with a respiratory rate of 35 breaths per minute, a pulse rate of 120 beats per minute, blood pressure of 110/70 mmHg, and peak expiratory flow rate (PEFR) <50% predicted.
_x000D_
He has been given back-to-back nebulisers of salbutamol 5 mg and ipratropium 0.5 mg for the last 45 minutes and is on face mask oxygen. He has been given hydrocortisone 100 mg IV. The Intensive Care Team are aware of the gentleman.
_x000D_
His Arterial Blood Gas (done on high-flow oxygen) reveals:
_x000D_
_x000D_ _x000D_
_x000D_ _x000D_ pH
_x000D_
_x000D_
_x000D_ 7.41
_x000D_
_x000D_
_x000D_ (7.36-7.44)
_x000D_
_x000D_
_x000D_
_x000D_ _x000D_ PaCO2
_x000D_
_x000D_
_x000D_ 5.1 kPa
_x000D_
_x000D_
_x000D_ (4.7-6.0)
_x000D_
_x000D_
_x000D_
_x000D_ _x000D_ PaO2
_x000D_
_x000D_
_x000D_ 24kPa
_x000D_
_x000D_
_x000D_ (11.3-12.6)
_x000D_
_x000D_
_x000D_
_x000D_ _x000D_ Base Excess
_x000D_
_x000D_
_x000D_ −2 mmol/L
_x000D_
_x000D_
_x000D_ (+/-2)
_x000D_
_x000D_
_x000D_
_x000D_ _x000D_ SpO2
_x000D_
_x000D_
_x000D_ 98%
_x000D_
_x000D_
_x000D_
_x000D_
_x000D_
Which one of the following therapies should be implemented next?
CorrectIncorrectHint
The correct answer is D. Magnesium 1-2 gm IV. This is because this gentleman has severe acute asthma that is not responding to standard treatment with oxygen, nebulised bronchodilators, and systemic corticosteroids. This gentleman’s arterial blood gas shows a normal pH, a normal PaCO2, and a high PaO2, indicating that he is receiving adequate oxygen therapy but has severe airflow obstruction. This gentleman’s peak expiratory flow rate is less than 50% predicted, which is another sign of severe asthma.
_x000D_
Magnesium is a smooth muscle relaxant that can reduce bronchospasm and improve lung function in patients with severe acute asthma. Magnesium can be given intravenously as a single dose of 1.2 to 2 g over 20 minutes, preferably under the supervision of a senior clinician or an Intensive Care Team. Magnesium can also reduce the risk of hospital admission and the need for mechanical ventilation.
_x000D_
The other answer options are incorrect for the following reasons:
_x000D_
- _x000D_
- A. Oral aminophylline: Aminophylline is a methylxanthine drug that has bronchodilator and anti-inflammatory effects. However, aminophylline has a narrow therapeutic window and can cause serious side effects such as arrhythmias, seizures, and gastrointestinal bleeding. Aminophylline is not recommended for the routine management of acute asthma, and should only be considered as a last resort in life-threatening cases, under the guidance of a specialist.
- B. Non-invasive ventilation (NIV): Non-invasive ventilation (NIV) is a form of mechanical ventilation that delivers positive pressure to the airways through a mask or a helmet, without the need for intubation. Non-invasive ventilation (NIV) can improve oxygenation and reduce the work of breathing in patients with respiratory failure due to various causes. However, non-invasive ventilation (NIV) is not indicated for patients with acute asthma, as it can worsen airflow obstruction, increase the risk of pneumothorax, and delay intubation if needed.
- C. Oral prednisolone: Prednisolone is a corticosteroid that reduces inflammation and mucus production in the airways. Prednisolone is an essential component of the treatment of acute asthma, and should be given as soon as possible to all patients with moderate, severe, or life-threatening asthma. However, prednisolone alone is not sufficient to treat severe acute asthma, and should be combined with other therapies such as oxygen, bronchodilators, and magnesium. This gentleman in the question has already received hydrocortisone, which is a parenteral form of corticosteroid, so oral prednisolone is not necessary.
- E. Intubation and ventilation: Intubation and ventilation is a life-saving procedure that involves inserting a tube into the trachea and connecting it to a machine that delivers oxygen and pressure to the lungs. Intubation and ventilation is indicated for patients with life-threatening asthma who have signs of respiratory failure, such as hypoxia, hypercapnia, acidosis, altered mental status, or exhaustion. However, intubation and ventilation is not without risks, such as barotrauma, infection, and ventilator-associated lung injury. Therefore, intubation and ventilation should be avoided if possible, and reserved for patients who do not respond to other treatments or who have imminent respiratory arrest. The gentleman in this question does not have any life-threatening features of asthma, and may benefit from magnesium therapy before considering intubation and ventilation.
_x000D_
_x000D_
_x000D_
_x000D_
-
Question 36 of 123
36. Question
A 24-year-old university student presents to the respiratory clinic with increased shortness of breath, a dry cough, right sided pleuritic chest pain and extreme lethargy over three weeks.
_x000D_
He has no significant medical history of note but says a few other people have been ill in his class at the university over the past few weeks.
_x000D_
On examination, he is pyrexial with a temperature of 100.8°F, his blood pressure is 110/70 mmHg, pulse rate is 85 beats per minute and regular. There is scattered wheeze, more marked on the right than the left.
_x000D_
Investigations Reveal:
_x000D_
_x000D_ _x000D_
_x000D_ _x000D_ Haemoglobin
_x000D_
_x000D_
_x000D_ 88 g/L
_x000D_
_x000D_
_x000D_ (115-160)
_x000D_
_x000D_
_x000D_
_x000D_ _x000D_ Total Leucocyte Count
_x000D_
_x000D_
_x000D_ 12.2 ×109/L
_x000D_
_x000D_
_x000D_ (4-11)
_x000D_
_x000D_
_x000D_
_x000D_ _x000D_ Platelet Count
_x000D_
_x000D_
_x000D_ 172 ×109/L
_x000D_
_x000D_
_x000D_ (150-400)
_x000D_
_x000D_
_x000D_
_x000D_ _x000D_ Serum Sodium
_x000D_
_x000D_
_x000D_ 140 mmol/L
_x000D_
_x000D_
_x000D_ (135-146)
_x000D_
_x000D_
_x000D_
_x000D_ _x000D_ Serum Potassium
_x000D_
_x000D_
_x000D_ 4.1 mmol/L
_x000D_
_x000D_
_x000D_ (3.5-5)
_x000D_
_x000D_
_x000D_
_x000D_ _x000D_ Serum Creatinine
_x000D_
_x000D_
_x000D_ 125 µmol/L
_x000D_
_x000D_
_x000D_ (79-118)
_x000D_
_x000D_
_x000D_
_x000D_ _x000D_ Erythrocyte Sedimentation Rate (ESR)
_x000D_
_x000D_
_x000D_ 81 mm/hr
_x000D_
_x000D_
_x000D_ (<10)
_x000D_
_x000D_
_x000D_
_x000D_
_x000D_
Chest X-Ray – Right lower lobe pneumonia.
_x000D_
Blood Film – Schistocytes.
_x000D_
Which one of the following is the most likely causative organism in this case?
CorrectIncorrectHint
The most likely causative organism in this case is option C. Mycoplasma pneumonia. This is because the clinical presentation and the laboratory findings are consistent with a diagnosis of atypical pneumonia, which is often caused by Mycoplasma pneumoniae, a type of bacteria that does not have a cell wall and can infect the respiratory tract. Mycoplasma pneumoniae is also known to cause outbreaks of respiratory illness in young adults, especially in crowded settings such as universities. Mycoplasma pneumoniae can also cause haemolytic anaemia, which is a condition where the red blood cells are destroyed prematurely and can result in schistocytes, or fragmented red blood cells, on the blood film.
_x000D_
The other options are less likely for the following reasons:
_x000D_
- _x000D_
- Option A is incorrect because Pseudomonas aeruginosa is a gram-negative bacterium that usually causes pneumonia in patients with cystic fibrosis, bronchiectasis, or immunosuppression. The student in this question has none of these risk factors and his chest x-ray shows right lower lobe pneumonia, which is not typical for Pseudomonas aeruginosa infection.
- Option B is incorrect because Legionella pneumophila is a gram-negative bacterium that causes Legionnaires’ disease, a severe form of pneumonia that is associated with exposure to contaminated water sources. The student in this question has no history of such exposure and his laboratory findings do not show hyponatraemia, which is a common feature of Legionella pneumophila infection.
- Option D is incorrect because Streptococcus pneumoniae is a gram-positive bacterium that causes typical pneumonia, which is characterized by a lobar or segmental consolidation on the chest x-ray, a productive cough, and a high white blood cell count. This student in the question has a dry cough, a scattered wheeze, and a normal white blood cell count, which are not consistent with typical pneumonia.
- Option E is incorrect because Staphylococcus aureus is a gram-positive bacterium that causes pneumonia in patients with influenza, intravenous drug use, or prior lung damage. The student in the question has no history of these risk factors and his chest x-ray does not show multiple cavities or abscesses, which are common complications of Staphylococcus aureus infection.
_x000D_
_x000D_
_x000D_
_x000D_
-
Question 37 of 123
37. Question
A 32-year-old gentleman had been treated for pulmonary tuberculosis with rifampicin, isoniazid, pyrazinamide and ethambutol for four weeks.
_x000D_
Pre-treatment liver function tests (LFTs) were normal but his most recent investigations revealed:
_x000D_
_x000D_ _x000D_
_x000D_ _x000D_ Serum Total Bilirubin
_x000D_
_x000D_
_x000D_ 100 µmol/l
_x000D_
_x000D_
_x000D_ (0-18)
_x000D_
_x000D_
_x000D_
_x000D_ _x000D_ Serum Alanine Aminotransferase (ALT)
_x000D_
_x000D_
_x000D_ 631 U/L
_x000D_
_x000D_
_x000D_ (5-45)
_x000D_
_x000D_
_x000D_
_x000D_ _x000D_ Serum Aspartate Aminotransferase (AST)
_x000D_
_x000D_
_x000D_ 455 U/L
_x000D_
_x000D_
_x000D_ (5-45)
_x000D_
_x000D_
_x000D_
_x000D_ _x000D_ Serum Alkaline Phosphatase (ALP)
_x000D_
_x000D_
_x000D_ 718 U/L
_x000D_
_x000D_
_x000D_ (40-110)
_x000D_
_x000D_
_x000D_
_x000D_
_x000D_
Which one of the following is the most appropriate next step?
CorrectIncorrectHint
The most appropriate next step is E. Stop all treatments. This is because the gentleman has developed drug-induced liver injury (DILI) from antituberculous treatment (ATT), which is a serious and potentially fatal complication. This gentleman’s liver function tests (LFTs) are markedly elevated, indicating severe hepatocellular damage. The gentleman may also have symptoms such as nausea, dark urine, jaundice, or abdominal pain.
_x000D_
The recommended management of antituberculous treatment (ATT)-associated drug-induced liver injury (DILI) is to stop all anti-tuberculosis drugs immediately and monitor the gentleman closely until the liver function tests (LFTs) normalize or improve significantly. After recovery, the same drugs may be safely reintroduced in most cases, following a stepwise approach. This is possible because of a phenomenon known as hepatic adaptation, which reduces the risk of drug-induced liver injury (DILI) recurrence. However, some patients may require alternative drugs or regimens, depending on the severity of drug-induced liver injury (DILI) and the availability of other options.
_x000D_
The drug that may have caused drug-induced liver injury (DILI) in the above case is pyrazinamide. This is because pyrazinamide is the most hepatotoxic drug among the first-line anti-tuberculosis drugs, and it is usually the first drug to be stopped in cases of drug-induced liver injury (DILI). Pyrazinamide is metabolized to pyrazinoic acid and 5-hydroxy-pyrazinoic acid, which can cause oxidative stress and inflammation in the liver.
_x000D_
The other answer options can be ruled out with the following explanations:
_x000D_
- _x000D_
- Rifampicin: Rifampicin can also cause drug-induced liver injury (DILI), but it is less likely than pyrazinamide. Rifampicin is usually the second drug to be stopped in cases of drug-induced liver injury (DILI), after pyrazinamide. Rifampicin induces the expression of cytochrome P450 enzymes, which can increase the metabolism and toxicity of other drugs.
_x000D_
_x000D_
- _x000D_
- Isoniazid: Isoniazid can also cause drug-induced liver injury (DILI), but it is less likely than pyrazinamide and rifampicin. Isoniazid is usually the third drug to be stopped in cases of drug-induced liver injury (DILI), after pyrazinamide and rifampicin. Isoniazid is metabolized to acetyl hydrazine and hydralazine, which can form reactive metabolites and bind to liver proteins.
_x000D_
_x000D_
Ethambutol: Ethambutol is the least hepatotoxic drug among the first-line anti-tuberculosis drugs, and it is usually the first drug to be restarted in cases of drug-induced liver injury (DILI). Ethambutol is mainly excreted unchanged in the urine, and its metabolism is not affected by cytochrome P450 enzymes
-
Question 38 of 123
38. Question
A 30-year-old young car racer becomes confused and dyspnoeic on the orthopaedic ward, 24 hours after fracturing his left femur in an accident.
_x000D_
Which one of the following skin lesions may be found on examination?
CorrectIncorrectHint
The most likely complication for the case that has been described above is fat embolism syndrome (FES). This is a condition where fat droplets from the bone marrow enter the bloodstream and block the blood vessels in the lungs, brain, and other organs. Fat embolism syndrome (FES) usually occurs 12 to 72 hours after a long bone fracture, especially of the femur. Fat embolism syndrome (FES) can cause shortness of breath, chest pain, mental confusion, coma, and a pinpoint rash (called a petechial rash) that often appears on the chest, head, and neck area.
_x000D_
Therefore, the correct answer to this question is C. Multiple petechiae in both axillae. This is a sign of a petechial rash, which is a common skin manifestation of fat embolism syndrome (FES). The other options are not relevant for fat embolism syndrome (FES), as they are associated with different conditions, such as:
_x000D_
- _x000D_
- Target lesions on the chest: These are circular lesions with a central dark spot and a lighter ring around it. They are seen in erythema multiforme, a hypersensitivity reaction to infections or drugs.
- Vesicular lesions on the torso: These are fluid-filled blisters that can be caused by various infections, such as chickenpox, herpes zoster, or impetigo.
- Palpable purpura on buttocks and legs: These are raised purple spots that indicate bleeding under the skin. They are seen in vasculitis, an inflammation of the blood vessels due to autoimmune or infectious causes.
- Tender red nodules on the shins: These are painful lumps that occur on the front of the lower legs. They are seen in erythema nodosum, an inflammatory reaction of the fat tissue under the skin, often related to infections, drugs, or inflammatory diseases.
_x000D_
_x000D_
_x000D_
_x000D_
-
Question 39 of 123
39. Question
A 30-year-old young Car Racer becomes confused and dyspnoeic on the Orthopaedic Ward, 24 hours after fracturing his left femur in an accident.
_x000D_
Which of the following Skin Lesions may be found on examination?
CorrectIncorrectHint
The appearance of Multiple Petechiae in the distribution of the Axilla or Upper Body is characteristic of a Fat Embolism._x000D_
Unlike emboli that arise from a thrombus, fat emboli are small and multiple producing widespread effects. They may occur one to three days following a fracture and are more common in closed fractures on the long bones or pelvis._x000D_
The clinical features of fat emboli are predominately:_x000D_
? Pulmonary (shortness of breath, hypoxia)_x000D_
? Neurological (confusion and agitation)_x000D_
? Dermatological (petechiae), and_x000D_
? Haematological (thrombocytopenia, anaemia)._x000D_
The petechial rash is pathognomonic of this syndrome, but occurs in only 30-50% of cases. -
Question 40 of 123
40. Question
A 30-year-old lady with acute respiratory distress syndrome (ARDS) is ventilated on Intensive Care Unit.
_x000D_
Her inspired oxygen is 100%, positive end expiratory pressure (PEEP) is 15 cmH2O and peak airway pressure is 40 cmH2O.
_x000D_
Arterial Blood Gases (ABG) show:
_x000D_
_x000D_ _x000D_
_x000D_ _x000D_ PaO2
_x000D_
_x000D_
_x000D_ 6.1 kPa
_x000D_
_x000D_
_x000D_ (11.3-12.6)
_x000D_
_x000D_
_x000D_
_x000D_ _x000D_ PaCO2
_x000D_
_x000D_
_x000D_ 6.7 kPa
_x000D_
_x000D_
_x000D_ (4.7-6.0)
_x000D_
_x000D_
_x000D_
_x000D_ _x000D_ SpO2
_x000D_
_x000D_
_x000D_ 86%
_x000D_
_x000D_
_x000D_ (>92%)
_x000D_
_x000D_
_x000D_
_x000D_
_x000D_
Which one of the following treatments is most likely to decrease mortality in this patient group?
CorrectIncorrectHint
Acute respiratory distress syndrome (ARDS) is a serious condition that causes fluid in the lungs and low oxygen levels in the blood. The goal of treatment for acute respiratory distress syndrome (ARDS) is to improve oxygen levels and treat the underlying cause. Oxygen therapy is the main treatment for acute respiratory distress syndrome (ARDS), and most people with acute respiratory distress syndrome (ARDS) need a mechanical ventilator and/or oxygen therapy to improve blood oxygen levels. Other treatments aim to prevent complications and make the patient comfortable.
_x000D_
The lady in this question is currently receiving the highest level of ventilatory support available but is still experiencing hypoxia. The elevated level of carbon dioxide (CO2) in her blood is a deliberate strategy known as permissive hypercapnia, which is employed to prevent excessive lung distension caused by high tidal volumes.
_x000D_
Treating patients in this condition used to present significant challenges, as no technique showed substantial improvements in prognosis. However, in 2010, the CESAR trial provided a breakthrough. This trial demonstrated that the use of extracorporeal membrane oxygenation (ECMO) significantly increased survival rates without resulting in significant disability.
_x000D_
Based on the above discussion, the most likely treatment to decrease mortality in this patient group is extracorporeal membrane oxygenation (ECMO). Extracorporeal membrane oxygenation (ECMO) is a life support technique that uses a machine to pump blood outside the body, add oxygen to it, and then return it to the body. Extracorporeal membrane oxygenation (ECMO) can improve oxygenation and reduce the risk of ventilator-induced lung injury in patients with severe acute respiratory distress syndrome (ARDS). However, extracorporeal membrane oxygenation (ECMO) is not widely available and has some serious risks, such as bleeding, infection, and blood clots. Therefore, it should be used only when other treatments have failed or are not available.
_x000D_
The other options are less likely to decrease mortality in this patient group, and some may even be harmful. Here is a brief summary of each option:
_x000D_
- _x000D_
- Inhaled nitric oxide therapy: Nitric oxide is a gas that can relax the blood vessels in the lungs and improve blood flow. However, there is no clear evidence that inhaled nitric oxide therapy reduces mortality or improves outcomes in patients with acute respiratory distress syndrome (ARDS). It may also cause side effects such as low blood pressure, bleeding, and kidney problems.
- High frequency oscillatory ventilation (HFOV): High frequency oscillatory ventilation (HFOV) is a type of mechanical ventilation that delivers very small breaths at a very high rate. High frequency oscillatory ventilation (HFOV) can reduce the risk of overstretching the lungs and causing more damage. However, high frequency oscillatory ventilation (HFOV) has not been shown to reduce mortality or improve outcomes in patients with acute respiratory distress syndrome (ARDS). It may also increase the risk of low blood pressure, low cardiac output, and air leaks in the lungs.
- Increasing tidal volume and respiratory rate on the ventilator: Tidal volume is the amount of air that moves in and out of the lungs with each breath, and respiratory rate is the number of breaths per minute. Increasing tidal volume and respiratory rate on the ventilator may seem to improve oxygenation, but it can also cause more lung injury and inflammation. Studies have shown that lower tidal volumes (6 mL per kg) and higher positive end-expiratory pressure (PEEP) values (12 cm H2O) improve outcomes and reduce mortality in patients with acute respiratory distress syndrome (ARDS).
- Prone position: Prone position means lying on the stomach instead of the back. Prone position can improve oxygenation and lung mechanics in patients with acute respiratory distress syndrome (ARDS) by redistributing the lung volume and reducing the pressure on the lungs. Prone position has been shown to reduce mortality in patients with severe acute respiratory distress syndrome (ARDS), but not in patients with mild or moderate acute respiratory distress syndrome (ARDS). Prone position also requires careful monitoring and frequent repositioning to prevent complications such as pressure ulcers, facial oedema, and nerve injuries.
_x000D_
_x000D_
_x000D_
_x000D_
-
Question 41 of 123
41. Question
A 30-year-old lady with Adult Respiratory Distress Syndrome (ARDS) is Ventilated on Intensive Care.
_x000D_
Her ventilator setting is like that Fio2- 100%, Positive End Expiratory Pressure [PEEP] is 15 cmH2O and Peak Airway Pressure is 40 cmH2O.
_x000D_
Her Arterial Blood Gas [ABG] shows:
_x000D_
_x000D_ _x000D_
_x000D_ _x000D_ PaO2
_x000D_
_x000D_
_x000D_ 5.8 kPa
_x000D_
_x000D_
_x000D_ (11.3-12.6)
_x000D_
_x000D_
_x000D_
_x000D_ _x000D_ PaCO2
_x000D_
_x000D_
_x000D_ 7.2 kPa
_x000D_
_x000D_
_x000D_ (4.7-6.0)
_x000D_
_x000D_
_x000D_
_x000D_ _x000D_ SpO2
_x000D_
_x000D_
_x000D_ 82%
_x000D_
_x000D_
_x000D_ (>92%)
_x000D_
_x000D_
_x000D_
_x000D_
_x000D_
Which treatment is most likely to decrease mortality in this patient group?
CorrectIncorrectHint
This lady is on Maximal Ventilatory Therapy but is still Hypoxic. Her High CO2 is a reflection of permissive Hypercapnia to prevent Overdistension of the Lungs with High Tidal Volumes._x000D_
Treatment of these patients used to be extremely difficult, with no significant improvement in prognosis seen will any technique. However, in 2010 the CESAR trial demonstrated a significant increase in survival without significant disability with the use of Extracorporeal Membrane Oxygenation (ECMO)._x000D_
Extracorporeal Membrane Oxygenation [ECMO] involves connecting a patient’s circulation to an external oxygenator and pump, via a catheter placed in the right side of the heart. It requires the continuous infusion of anticoagulant, and as such bleeding is the most commonly associated complication. Infection and haemolysis are also a risk. Extracorporeal Membrane Oxygenation (ECMO) is a treatment option considered for Acute Respiratory Distress Syndrome (ARDS) patients who are refractory to conventional treatments. However, treatment with Extracorporeal Membrane Oxygenation [ECMO] has not shown significant reduction of mortality which may be due to inappropriate selection criteria._x000D_
A recent study1 published in the NEJM concluded that prone positioning improved outcome in severe Acute Respiratory Distress Syndrome [ARDS]. However, it is only one study in a controversial area. There are a number of studies, including one by the same group, which have not shown a mortality benefit (and others which show a detrimental effect on mortality). Prone positioning is therefore not yet widely accepted in clinical practice, although this may change in the future. -
Question 42 of 123
42. Question
A 30-year-old lady is referred to the Respiratory Clinic with increasing shortness of breath and episodes of syncope. On further questioning it transpires that her mother suffered from a lung / circulatory condition and died a few years after a heart lung transplant, and her aunt died at a young age, from a condition which they were told was heart failure.
_x000D_
Her Blood Pressure is 150/80 mmHg, her Pulse Rate is 85/Minute and Regular, and her BMI is 22. There is a murmur of tricuspid regurgitation, and bilateral pitting oedema of the ankles.
_x000D_
Investigations Show:
_x000D_
_x000D_ _x000D_
_x000D_ _x000D_ Haemoglobin
_x000D_
_x000D_
_x000D_ 130 g/L
_x000D_
_x000D_
_x000D_ (115-160)
_x000D_
_x000D_
_x000D_
_x000D_ _x000D_ White Blood Cell Count
_x000D_
_x000D_
_x000D_ 6.7 ×109/L
_x000D_
_x000D_
_x000D_ (4-11)
_x000D_
_x000D_
_x000D_
_x000D_ _x000D_ Platelet Count
_x000D_
_x000D_
_x000D_ 210 ×109/L
_x000D_
_x000D_
_x000D_ (150-400)
_x000D_
_x000D_
_x000D_
_x000D_ _x000D_ Serum Sodium
_x000D_
_x000D_
_x000D_ 139 mmol/L
_x000D_
_x000D_
_x000D_ (135-146)
_x000D_
_x000D_
_x000D_
_x000D_ _x000D_ Serum Potassium
_x000D_
_x000D_
_x000D_ 4.40 mmol/L
_x000D_
_x000D_
_x000D_ (3.5-5)
_x000D_
_x000D_
_x000D_
_x000D_ _x000D_ Serum Creatinine
_x000D_
_x000D_
_x000D_ 120 µmol/L
_x000D_
_x000D_
_x000D_ (79-118)
_x000D_
_x000D_
_x000D_
_x000D_ _x000D_ Echocardiogram
_x000D_
_x000D_
_x000D_ Evidence Of Increased Right Sided Pressures.
_x000D_
_x000D_
_x000D_
_x000D_
_x000D_
Which of the following is the most likely mode of inheritance?
CorrectIncorrectHint
This woman has familial Primary Pulmonary Hypertension (PPH). This is evidenced by her age, and particularly symptoms such as increased shortness of breath and recurrent syncope. The increased right sided pressures are also a give away with respect to the diagnosis._x000D_
The fact her aunt and mother suffered from a similar disease points to an inherited component._x000D_
The condition is recognised to account for 15-20% of cases of Primary Pulmonary Hypertension (PPH) and is due to an inherited mutation in the BMPR2 Gene which codes for a receptor in the TGF Beta family. Hereditary Haemorrhagic Telangiectasia, another cause of Inherited Pulmonary Hypertension, also carries an Autosomal Dominant Inheritance pattern._x000D_
The occurrence of Pulmonary Hypertension in a number of family members makes a de novo mutation highly unlikely._x000D_
Of the other modes of inheritance listed, only autosomal dominant is correct. -
Question 43 of 123
43. Question
A 30-year-old gentleman who works on a local Mushroom Farm comes to the Respiratory Clinic with increased shortness of breath. In addition to feeling increasingly short of breath, he has suffered intermittent fevers and sweats over the past few months.
_x000D_
On examination, his Blood Pressure is 130/70 mmHg, Pulse Rate is 72/Minute and Regular. There are fine inspiratory crackles at both bases on auscultation of his chest.
_x000D_
Investigations show:
_x000D_
_x000D_ _x000D_
_x000D_ _x000D_ Haemoglobin
_x000D_
_x000D_
_x000D_ 137 g/L
_x000D_
_x000D_
_x000D_ (135-177)
_x000D_
_x000D_
_x000D_
_x000D_ _x000D_ White Blood Cell Count
_x000D_
_x000D_
_x000D_ 9.5 ×109/L
_x000D_
_x000D_
_x000D_ (4-11)
_x000D_
_x000D_
_x000D_
_x000D_ _x000D_ Platelet Count
_x000D_
_x000D_
_x000D_ 210 ×109/L
_x000D_
_x000D_
_x000D_ (150-400)
_x000D_
_x000D_
_x000D_
_x000D_ _x000D_ Serum Sodium
_x000D_
_x000D_
_x000D_ 140 mmol/L
_x000D_
_x000D_
_x000D_ (135-146)
_x000D_
_x000D_
_x000D_
_x000D_ _x000D_ Serum Potassium
_x000D_
_x000D_
_x000D_ 4.4 mmol/L
_x000D_
_x000D_
_x000D_ (3.5-5)
_x000D_
_x000D_
_x000D_
_x000D_ _x000D_ Serum Creatinine
_x000D_
_x000D_
_x000D_ 117 µmol/L
_x000D_
_x000D_
_x000D_ (79-118)
_x000D_
_x000D_
_x000D_
_x000D_
_x000D_
Chest X-Ray shows diffuse interstitial shadowing. Spirometry shows Mixed Restrictive / Obstructive Picture.
_x000D_
Which of the following is the most appropriate course of action for the longer term?
CorrectIncorrectHint
This gentleman’s occupation as a mushroom picker raises the possibility of Mushroom Picker’s Lung, a form of Hypersensitivity Pneumonitis. Optimal management is removal of exposure to the antigen, although in the short term a course of oral corticosteroids is likely to be appropriate._x000D_
Inhaled Anticholinergics are used in the management of Chronic Obstructive Pulmonary Disease (COPD) and are therefore not appropriate here._x000D_
Inhaled Corticosteroids are not as effective as Oral Corticosteroids in managing Subacute or Chronic Hypersensitivity Pneumonitis, so Oral Prednisolone is the treatment of choice after removal of exposure to the antigen has occurred._x000D_
There is no role for Antibiotic Therapy. -
Question 44 of 123
44. Question
A 30-year-old gentleman who had been diagnosed one week previously with tuberculosis (TB) of the mediastinal lymph nodes and who had been started on chemotherapy with rifampicin, isoniazid and pyrazinamide was admitted because of the increasing dyspnoea and stridor.
_x000D_
Chest x-ray showed compression of both main bronchi by carinal lymph node enlargement.
_x000D_
Which one of the following is the next step to be incorporated in the management?
CorrectIncorrectHint
The treatment approach for tuberculosis (TB) mediastinal lymphadenitis is generally the same as that for pulmonary tuberculosis (TB). Both conditions require a comprehensive treatment regimen to effectively combat the infection.
_x000D_
During the course of tuberculosis (TB) treatment, a phenomenon known as a ‘paradoxical reaction’ may occur. This reaction can lead to the development of new lesions or the worsening of existing lesions. The timing of this reaction is unpredictable, and it can manifest anywhere from a few days to several months after the initiation of treatment. The duration and severity of the reaction can vary greatly, making it challenging to differentiate from treatment failure, drug resistance, or a superadded infection. Paradoxical reactions are commonly observed in cases involving lymph node or cerebral disease, with approximately 30% of cases experiencing enlargement of affected lymph nodes. Fortunately, in most instances, these reactions tend to resolve on their own over time.
_x000D_
Corticosteroids have proven to be effective in reducing lymph node enlargement and inflammation associated with tuberculosis (TB) mediastinal lymphadenitis. By alleviating these symptoms, corticosteroids can help alleviate stridor and breathlessness in affected individuals. However, it is important to note that if the compression is occurring at the carina, the placement of a tracheal stent or tracheostomy will not provide relief from the obstruction.
_x000D_
In the case of tuberculosis (TB) mediastinal lymphadenitis, additional diagnostic procedures such as mediastinoscopy and biopsy, as well as a computed tomography (CT) scan of the mediastinum, may not yield any additional valuable information since the diagnosis is already established.
_x000D_
While standard tuberculosis (TB) treatment typically involves the administration of ethambutol, rifampicin, isoniazid, and pyrazinamide for an initial period of two months, the use of ethambutol may not always be necessary, especially when the susceptibility of the organism is known. It is unlikely that the exclusion of ethambutol has led to treatment failure, and therefore, reintroducing it at this point is unlikely to alleviate the symptoms.
_x000D_
In summary, the treatment approach for tuberculosis (TB) mediastinal lymphadenitis is similar to that for pulmonary tuberculosis (TB). Paradoxical reactions may occur during tuberculosis (TB) treatment, causing new lesions or the worsening of existing ones. Corticosteroids can be beneficial in reducing lymph node enlargement and inflammation. Additional diagnostic procedures may not be necessary if the diagnosis is already established. The omission of ethambutol in the treatment regimen is unlikely to be the cause of treatment failure and may not provide symptom relief if reintroduced.
-
Question 45 of 123
45. Question
A 30-year-old gentleman presents with gradual onset of cough and shortness of breath on exertion.
_x000D_
He has no medical history of note. He is single and smokes 25 cigarettes a day. He reports in the last year he has had a number of infections, and last month was diagnosed with Oesophageal Candidiasis on Endoscopy.
_x000D_
His Oxygen Saturations are 93% on Room Air At Rest, but drop to 82% on Minimal Exertion. Chest Examination is Unremarkable. A Chest X-Ray is requested.
_x000D_
What is the most likely diagnosis?
CorrectIncorrectHint
The history of recurrent infections in this young patient should lead you to consider Immunosuppression as an underlying diagnosis and therefore Pneumocystis jiroveci pneumonia._x000D_
Pneumocystis jiroveci is a eukaryotic microorganism. In immunosuppressed patients it can cause a pneumonia, which is most recognised in patients with Acquired Immune Deficiency Syndrome [AIDS] but can also be seen in those with Organ Transplants or when undergoing Chemotherapy. A CD4 Count of less than 200 is associated with significant risk. In Europe, the USA and Australia P. jiroveci pneumonia in HIV-positive patients is seen largely in those unaware of their HIV Status. Unfortunately, it is a major cause of death in Africa, especially in children. Previously it was thought that disease was caused by reactivation of latent infection acquired in childhood, but de novo infection is increasingly recognised._x000D_
The pneumonia caused by P. jiroveci is potentially severe and fatal in immunosuppressed patients. Clinically it presents with several weeks’ history of dry cough, fever and dyspnoea. Examination findings are often subtle, but include tachypnoea, tachycardia, cyanosis and fine respiratory crackles. Typically, patients desaturate markedly on exertion. There may be Reduced Transfer Factor, Vital Capacity and Total Lung Capacity on Spirometry. Bronchoalveolar Lavage or Induced Sputum can be used to demonstrate the organism (Open Lung Biopsy is Gold Standard, but rarely performed in clinical practice). Giemsa, Papanicolaou and Grocott’s Stains are used._x000D_
There are a variety of different chest radiograph findings. Typically it causes bilateral symmetrical perihilar reticular or granular interstitial shadowing. Less often there can be asymmetric shadowing, or progression to a reticular-alveolar pattern. The following can occasionally be seen:_x000D_
? lobar Consolidation_x000D_
? Nodular Lesions_x000D_
? Prominent Pulmonary Arteries_x000D_
? Pneumothorax_x000D_
? Pneumomediastinum_x000D_
? Cysts, or_x000D_
? Pneumatoceles._x000D_
In patients who have been on prophylactic inhaled pentamidine the infiltrates may predominantly affect the upper lobes. A normal chest x-ray does not exclude the diagnosis. Pleural effusions and lymphadenopathy are not typical, but be aware of the possibility of multiple disease processes in an immunosuppressed patient._x000D_
If allowed to progress, P. jiroveci can disseminate via the lymphatic and haematogenous routes to affect the thyroid, liver, bone marrow, lymph nodes and spleen._x000D_
If PCP is suspected, treatment with full dose Co-Trimoxazole should be started as soon as possible. It should be given for 21 days in HIV-Positive cases, but shorter doses can be used in other causes of immunosuppression. In patients who are intolerant or Co-Trimoxazole, Intravenous Pentamidine can be used. Some studies have shown that Corticosteroids can reduce the risk of respiratory failure, and they are therefore used in some cases._x000D_
Prophylaxis should be used in immunosuppressed patients who are at risk of developing PCP: all those with a CD4 Count of <200, patients started on High Dose Steroids, and those on Chemotherapeutic Regimens associated with significant immunosuppression. Co-Trimoxazole is also the First Line Prophylactic Agent._x000D_
It would be unusual for someone this young to develop Chronic Obstructive Pulmonary Disease [COPD], even if they are a heavy smoker. Asthma would have classical signs on examination if severe enough to cause hypoxia. If the patient had a Lung Abscess you would expect some systemic features of sepsis. A Pulmonary Embolism could account for these clinical signs, but there are no risk factors given in the question which makes PCP more likely. -
Question 46 of 123
46. Question
A 30-year-old gentleman presented to the Emergency Department with complaint of Severe Shortness Of Breath.
_x000D_
His Blood Pressure drops to 80/50 mmHg. He is Tachycardic and Apyrexial. His O2 Saturation is 74% on Room Air. Examination has revealed raised Jugular Venous Pressure (JVP) and Trachea Deviated to the Left Side.
_x000D_
What is the immediate next step to be taken?
CorrectIncorrectHint
Pneumothorax can be classified as either Primary or Secondary. However, more important in this case is the presence of tension._x000D_
Tension Pneumothorax results from progressive build-up of air within the pleural space, which is not able to return. This build-up of pressure pushes the mediastinum to the opposite hemithorax and obstructs venous return. The patient then develops severe breathlessness, hypotension, mediastinal shift and, ultimately, cardiac arrest (often electromechanical dissociation: PEA). The classical signs of a Tension Pneumothorax are deviation of the trachea away from the side of tension, hyper-expanded chest that moves minimally with respiration and increased percussion note. Jugular Venous Pressure (JVP) may be raised. However, more commonly the only signs are Tachycardia, Tachypnoea and Hypoxia._x000D_
Management of Tension Pneumothorax is immediate Chest Decompression with Needle Thoracostomy. A 14-16G Intravenous Cannula is inserted into the Second Rib Space in the Mid-Clavicular Line, on the side of the Pneumothorax. By acting as a valve, this converts a Tension Pneumothorax into a Simple Pneumothorax._x000D_
Definitive treatment of a Tension Pneumothorax is Chest Drain Insertion. This is usually done in the Emergency Department with Ultrasound Guidance. Once the Pleura is entered, the tension is relieved and the remainder of the procedure can be undertaken with less time pressure._x000D_
In this case, the option of Needle Thoracostomy is on the incorrect side, so the correct option is inserting a chest drain on the right side. High-Flow Oxygen is indicated whilst performing this, and Chest X-Ray should be ordered to check the position of the Drain. Arterial Blood Gases may be indicated once the drain is in situ._x000D_
Although the above explanation remains correct for the MRCP Examination, it is important to be aware that there has been discussion about the use of Needle Thoracostomy in the medical literature. It has been shown that on occasion it is ineffective in relieving a Tension Pneumothorax, especially in patients who have very thick chest walls. In addition they are prone to blockage, kinking and dislodging, and can cause a lung laceration which subsequently can lead to air embolisation (especially if a Pneumothorax was not present). Some groups are therefore recommending an urgent Chest X-Ray if there is no haemodynamic compromise. -
Question 47 of 123
47. Question
A 30-year-old Professional Saxophonist presents with a prolonged history of epistaxis and rapidly progressive shortness of breath.
_x000D_
The KCO and Eosinophil Count are raised.
_x000D_
Which of the following is the most likely diagnosis?
CorrectIncorrectHint
This patient has shortness of breath and a raised KCO, which leads to you to a diagnosis of Alveolar Haemorrhage. The condition which links Epistaxis and Alveolar Haemorrhage in the list above is Granulomatosis with Polyangiitis. KCO measures the uptake of carbon monoxide by the lungs, and is equivalent to the transfer factor._x000D_
Granulomatosis with Polyangiitis is a multi-organ autoimmune disease, which can be fatal. The classical triad consists of Necrotising Granulomatous Inflammation of the Respiratory Tract, Glomerulonephritis and a Small-Vessel Vasculitis. A prolonged history of epistaxis or sinusitis is commonly found in Granulomatosis with Polyangiitis, which in some patients is also associated with an Eosinophilia. The detection of Antineutrophil Cytoplasmic Antibodies directed against Proteinase-3 is highly specific, but is found in only 50% of patients with disease localised to the respiratory tract and 95% with generalised Granulomatosis with Polyangiitis._x000D_
Standard Therapy is with Cyclophosphamide and Corticosteroids. TNF-Alpha Blocking Agents, Anti-Thymocyte Globulin and Monoclonal Anti T-Cell Antibodies can be used in disease refractory to these agents._x000D_
Systemic Inflammation and Vasculitis contribute to accelerated Atherosclerosis in patients with Granulomatosis with Polyangiitis and there is therefore a significantly increased incidence of Stroke, Myocardial Infarction and Occlusive Artery Disease._x000D_
Alveolar Proteinosis is a rare diffuse lung condition, characterised by alveolar and interstitial accumulation of phospholipid protein derived from surfactant. It can be congenital, secondary or acquired, and patients often present with recurrent respiratory infections. Transfer factor (KCO) is typically reduced._x000D_
Eosinophilic Granulomatosis with Polyangiitis (EGPA) is a rare systemic vasculitis which affects small and medium sized vessels, in association with asthma. Patients initially present with allergic rhinitis and asthma, followed by eosinophilia and associated infiltrative disease (e.g. gastroenteritis) and then granulomatous inflammation classically within 3 years._x000D_
Goodpasture’s syndrome is an important, and potentially rapidly fatal, cause of alveolar haemorrhage. It is caused by circulating antiglomerular basement membrane antibodies, and typically causes an acute glomerulonephritis. Epistaxis is not as common an association._x000D_
Microscopic Polyangiitis is a Small Vessel Vasculitis which Classically Spares the Upper Respiratory Tract. -
Question 48 of 123
48. Question
A 29-year-old lady with a known history of asthma who lives in the town comes to the Pulmonology Clinic for review.
_x000D_
She has progressively worsening symptom and is currently taking high dose Seretide and Montelukast. Apparently she coughs every night, occasionally with Frank Haemoptysis and is finding it impossible to do any regular exercise at all.
_x000D_
On examination her Blood Pressure is 130/80 mmHg, Pulse Rate is 74 Beats Per Minute and Regular. There is extensive wheeze and scattered coarse crackles on auscultation of the chest. Her Peak Flow is 318 (580 predicted).
_x000D_
Investigations show:
_x000D_
_x000D_ _x000D_
_x000D_ _x000D_ Haemoglobin
_x000D_
_x000D_
_x000D_ 118 g/L
_x000D_
_x000D_
_x000D_ (115 – 160)
_x000D_
_x000D_
_x000D_
_x000D_ _x000D_ White Blood Cell Count
_x000D_
_x000D_
_x000D_ 8.1 ×109/L
_x000D_
_x000D_
_x000D_ (4 – 11)
_x000D_
_x000D_
_x000D_
_x000D_ _x000D_ Eosinophil Count
_x000D_
_x000D_
_x000D_ 1.1 ×109/L
_x000D_
_x000D_
_x000D_ (0.04 – 0.4)
_x000D_
_x000D_
_x000D_
_x000D_ _x000D_ Platelet Count
_x000D_
_x000D_
_x000D_ 171 ×109/L
_x000D_
_x000D_
_x000D_ (150 – 400)
_x000D_
_x000D_
_x000D_
_x000D_ _x000D_ Serum Sodium
_x000D_
_x000D_
_x000D_ 140 mmol/L
_x000D_
_x000D_
_x000D_ (135 – 146)
_x000D_
_x000D_
_x000D_
_x000D_ _x000D_ Serum Potassium
_x000D_
_x000D_
_x000D_ 4.1 mmol/L
_x000D_
_x000D_
_x000D_ (3.5 – 5)
_x000D_
_x000D_
_x000D_
_x000D_ _x000D_ Serum Creatinine
_x000D_
_x000D_
_x000D_ 101 µmol/L
_x000D_
_x000D_
_x000D_ (79 – 118)
_x000D_
_x000D_
_x000D_
_x000D_
_x000D_
Which of the following is the best investigation?
CorrectIncorrectHint
The most likely diagnosis here is Allergic Bronchopulmonary Aspergillosis (ABPA); as such Aspergillus Precipitins can contribute significantly to confirming the diagnosis._x000D_
Oral Steroids are the mainstay of initial treatment and Anti-Fungals such as Itraconazole may also improve resolution of symptoms in some cases._x000D_
Whilst both chest X-Ray and CT Scan Thorax may show evidence of infiltrates this is relatively non-specific and therefore would not be optimal investigations; they may, however, prove useful in assessing severity of disease._x000D_
IgE is elevated in Allergic Bronchopulmonary Aspergillosis [ABPA] but is not as specific an investigation as Aspergillus Precipitins._x000D_
Sputum Culture may be positive but negative culture would not exclude the disease. -
Question 49 of 123
49. Question
A 29-year-old lady with a known history of asthma who lives in the countryside comes to the pulmonology clinic for review.
_x000D_
She has progressively worsening disease and is currently taking high dose seretide and montelukast. Apparently she coughs every night, occasionally with frank haemoptysis and is finding it impossible to do any regular exercise at all.
_x000D_
On examination, her blood pressure is 130/80 mmHg, and pulse rate is 74 beats per minute and regular. There is extensive wheeze and scattered coarse crackles on auscultation of the chest. Her peak flow is 318 (580 predicted).
_x000D_
Investigations Show:
_x000D_
_x000D_ _x000D_
_x000D_ _x000D_ Haemoglobin
_x000D_
_x000D_
_x000D_ 118 g/L
_x000D_
_x000D_
_x000D_ (115 – 160)
_x000D_
_x000D_
_x000D_
_x000D_ _x000D_ White Blood Cell Count
_x000D_
_x000D_
_x000D_ 8.1 ×109/L
_x000D_
_x000D_
_x000D_ (4 – 11)
_x000D_
_x000D_
_x000D_
_x000D_ _x000D_ Eosinophil Count
_x000D_
_x000D_
_x000D_ 1.1 ×109/L
_x000D_
_x000D_
_x000D_ (0.04 – 0.4)
_x000D_
_x000D_
_x000D_
_x000D_ _x000D_ Platelet Count
_x000D_
_x000D_
_x000D_ 171 ×109/L
_x000D_
_x000D_
_x000D_ (150 – 400)
_x000D_
_x000D_
_x000D_
_x000D_ _x000D_ Serum Sodium
_x000D_
_x000D_
_x000D_ 140 mmol/L
_x000D_
_x000D_
_x000D_ (135 – 146)
_x000D_
_x000D_
_x000D_
_x000D_ _x000D_ Serum Potassium
_x000D_
_x000D_
_x000D_ 4.1 mmol/L
_x000D_
_x000D_
_x000D_ (3.5 – 5)
_x000D_
_x000D_
_x000D_
_x000D_ _x000D_ Serum Creatinine
_x000D_
_x000D_
_x000D_ 101 µmol/L
_x000D_
_x000D_
_x000D_ (79 – 118)
_x000D_
_x000D_
_x000D_
_x000D_
_x000D_
Which one of the following is the best investigation?
CorrectIncorrectHint
Based on the provided information, the most probable diagnosis in this case is allergic bronchopulmonary aspergillosis (ABPA). Aspergillus precipitins, a type of antibody specific to the fungus Aspergillus, can play a crucial role in confirming this diagnosis.
_x000D_
Allergic bronchopulmonary aspergillosis is a hypersensitivity reaction to the presence of Aspergillus in the respiratory system. This condition primarily affects individuals with underlying asthma or cystic fibrosis. Common symptoms include wheezing, coughing, shortness of breath, and recurrent respiratory infections.
_x000D_
To confirm the diagnosis of allergic bronchopulmonary aspergillosis (ABPA), laboratory tests can be performed to detect the presence of specific antibodies known as aspergillus precipitins. These antibodies indicate an immune response to Aspergillus and can significantly contribute to confirming the diagnosis.
_x000D_
It is important to accurately diagnose allergic bronchopulmonary aspergillosis (ABPA) as early as possible to initiate appropriate management, which may involve antifungal medications, corticosteroids, and management of underlying respiratory conditions. The presence of aspergillus precipitins can provide valuable evidence to support the diagnosis of allergic bronchopulmonary aspergillosis in this case.
_x000D_
Therefore, correct answer to the above question is D. Aspergillus precipitins. This is a blood test that detects antibodies to Aspergillus, a type of fungus that can cause allergic reactions and infections in the lungs. Aspergillus precipitins can help diagnose allergic bronchopulmonary aspergillosis (ABPA), a condition that affects some people with asthma and causes inflammation, mucus plugging, and bronchiectasis. Allergic bronchopulmonary aspergillosis (ABPA) can also cause haemoptysis, which is the coughing up of blood from the lungs.
_x000D_
Other investigations for asthma and haemoptysis may include:
_x000D_
- _x000D_
- A. Computed tomography (CT) scan of thorax: This is an imaging test that can show detailed pictures of the lungs and the airways. It can help identify the cause and the source of haemoptysis, such as lung cancer, tuberculosis, bronchiectasis, or pulmonary embolism. It can also help evaluate the extent and severity of allergic bronchopulmonary aspergillosis (ABPA).
- B. IgE: This is a blood test that measures the level of immunoglobulin E, a type of antibody that is involved in allergic reactions. IgE levels are usually elevated in people with asthma and allergic bronchopulmonary aspergillosis (ABPA). However, IgE levels can also be affected by other factors, such as parasitic infections, eczema, or anaphylaxis. Therefore, IgE alone is not a specific test for allergic bronchopulmonary aspergillosis (ABPA) or haemoptysis.
- C. Sputum culture: This is a laboratory test that analyses the mucus that is coughed up from the lungs. It can help identify the presence and the type of bacteria, fungi, or viruses that may cause infection or inflammation in the lungs. Sputum culture can be useful for diagnosing pneumonia, tuberculosis, or aspergillosis, which are possible causes of haemoptysis.
- E. Chest x-ray: This is a simple and quick imaging test that can show the shape and size of the lungs and the heart. It can help detect abnormalities such as masses, nodules, consolidation, or cavitation, which may indicate lung cancer, tuberculosis, pneumonia, or pulmonary oedema. However, chest x-ray has limited sensitivity and specificity for diagnosing the cause of haemoptysis, and it may miss subtle or early changes in the lungs.
_x000D_
_x000D_
_x000D_
_x000D_
-
Question 50 of 123
50. Question
A 29-year-old gentleman, who works in a Car Body Shop (Spray Painting Vehicles), comes to the Respiratory Clinic complaining of increasing shortness of breath and wheeze. He says that he is fine at the end of a weekend off, and particularly well when he goes on holiday, but steadily gets worsening symptoms from Monday to Friday.
_x000D_
He is a non-smoker who has no significant past medical history. On examination his Blood Pressure is 120/80 mmHg, his Pulse Rate is 75/Minute and Regular. He has significant bilateral wheeze and a non-productive cough.
_x000D_
Investigations Show:
_x000D_
_x000D_ _x000D_
_x000D_ _x000D_ Haemoglobin
_x000D_
_x000D_
_x000D_ 151 g/L
_x000D_
_x000D_
_x000D_ (135-180)
_x000D_
_x000D_
_x000D_
_x000D_ _x000D_ White Blood Cell Count
_x000D_
_x000D_
_x000D_ 6.0 ×109/L
_x000D_
_x000D_
_x000D_ (4-11)
_x000D_
_x000D_
_x000D_
_x000D_ _x000D_ Platelet Count
_x000D_
_x000D_
_x000D_ 200 ×109/L
_x000D_
_x000D_
_x000D_ (150-400)
_x000D_
_x000D_
_x000D_
_x000D_ _x000D_ Serum Sodium
_x000D_
_x000D_
_x000D_ 142 mmol/L
_x000D_
_x000D_
_x000D_ (135-146)
_x000D_
_x000D_
_x000D_
_x000D_ _x000D_ Serum Potassium
_x000D_
_x000D_
_x000D_ 4.2 mmol/L
_x000D_
_x000D_
_x000D_ (3.5-5)
_x000D_
_x000D_
_x000D_
_x000D_ _x000D_ Serum Creatinine
_x000D_
_x000D_
_x000D_ 92 μmol/L
_x000D_
_x000D_
_x000D_ (79-118)
_x000D_
_x000D_
_x000D_
_x000D_ _x000D_ Peak Expiratory Flow Rate [PEFR]
_x000D_
_x000D_
_x000D_ 250 Litre/Minute
_x000D_
_x000D_
_x000D_
_x000D_
_x000D_
Which one of the following tests is the best initial way to elucidate any link between the workplace and possible asthma?
CorrectIncorrectHint
The most obvious initial investigation is to get the patient to keep a peak flow diary with multiple measurements at different times in the day and involving both weekdays and weekends._x000D_
Detailed Pulmonary Function Tests, and Radioallergosorbent (RAST) or Skin Testing may also be part of the work up, but the peak flow diary is the best way to establish the temporal relationship between Asthma and the Paint Spraying. Once the specific trigger is identified, he may either need to be given special equipment to protect him from exposure, or moved to a different role._x000D_
Simply treating his asthma with appropriate medication is not appropriate. -
Question 51 of 123
51. Question
A 29-year-old gentleman with known Acquired Immune Deficiency Syndrome [AIDS] presents with a history of increasing breathlessness and dry cough. He is investigated by the Respiratory Team and diagnosed with Pneumocystis pneumonia (PCP).
_x000D_
Which of the following features is most accurate regarding Pneumocystis pneumonia (PCP)?
CorrectIncorrectHint
Patients with Pneumocystis pneumonia often present with signs and symptoms of respiratory distress, although on examination of the chest there is often no abnormality detected._x000D_
Although initially reported as a disease associated with Acquired Immune Deficiency Syndrome [AIDS], Pneumocystis pneumonia (PCP) can affect any Immunocompromised patient including Transplant Patients._x000D_
The Trophozoite does not enter the Blood Stream, and the organism is usually identified in Pulmonary Secretions._x000D_
Co-Trimoxazole (Septrin) or Pentamidine are the treatments of choice. -
Question 52 of 123
52. Question
A 28-year-old male came to cardiology clinic with complaints of progressively increasing exercise intolerance. On detailed discussion it was found that he was suffering severe weakness during outdoor game mainly. His general and clinical examinations are unremarkable except on auscultation a faint systolic murmur is heard. The murmur increases its intensity during Valsalva Manoeuvre.
_x000D_
Among the following options which one is the best appropriate for above mentioned murmur characteristics?
CorrectIncorrectHint
Usually all stenotic and regurgitant valvular lesions with different types of murmur are best being heard during squatting position (due to increased intensity) and become less audible (due to decreased intensity). Only in Hypertrophic Obstructive Cardiomyopathy (HOCM) and Mitral Prolapse the murmur becomes louder during Valsalva Manoeuvre.
-
Question 53 of 123
53. Question
A 41-year-old gentleman presents at the respiratory clinic with dyspnoea and right pleuritic chest pain.
_x000D_
He previously had right pneumothorax 8 – 10 months ago for which he had a chest drain. He works as a welder. He is an ex-smoker.
_x000D_
On examination, he had decreased air entry on the right side and hyper-resonance. A chest x-ray confirms a large pneumothorax. A chest drain was inserted and the lung completely re-expanded.
_x000D_
Which one of the following is the best plan of action for this gentleman?
CorrectIncorrectHint
The best plan of action for this gentleman is A. Video assisted thoracoscopic surgery (VATS).
_x000D_
Video assisted thoracoscopic surgery (VATS) is a minimally invasive surgical procedure that involves making small incisions in the chest wall and inserting a camera and instruments to perform various operations on the lung. Video assisted thoracoscopic surgery (VATS) can be used to treat pneumothorax by removing the air from the pleural space and creating adhesions between the lung and the chest wall to prevent recurrence. Video assisted thoracoscopic surgery (VATS) has several advantages over other methods, such as faster recovery, less pain, lower risk of complications, and higher success rate.
_x000D_
Video assisted thoracoscopic surgery (VATS) is indicated in several conditions, including:
_x000D_
- _x000D_
- Second ipsilateral pneumothorax: This refers to a recurrence of a collapsed lung on the same side as a previous episode. Video assisted thoracoscopic surgery (VATS) can be performed to address the underlying cause and prevent further recurrences.
- Bilateral spontaneous pneumothorax: In some cases, both lungs may experience a spontaneous collapse. Video assisted thoracoscopic surgery (VATS) can be used to treat both sides simultaneously, reducing the risk of future collapses.
- Spontaneous haemothorax: This condition occurs when blood accumulates in the pleural space without any apparent cause. Video assisted thoracoscopic surgery (VATS) allows for the identification and management of the bleeding source, helping to resolve the haemothorax.
- Persistent air leak: If a patient continues to have air leakage from the lung for more than five to seven days, video assisted thoracoscopic surgery (VATS) can be employed to locate and repair the site of the leak, promoting lung re-expansion and preventing complications.
- Certain occupations, such as pilots or divers: Individuals in professions that require excellent lung function, such as pilots or divers, may undergo video assisted thoracoscopic surgery (VATS) to address any underlying lung conditions or complications that could potentially impact their performance and safety.
_x000D_
_x000D_
_x000D_
_x000D_
_x000D_
_x000D_
In summary, video assisted thoracoscopic surgery (VATS) is a valuable surgical approach for various conditions, including recurrent pneumothorax, bilateral pneumothorax, spontaneous haemothorax, persistent air leaks, and specific occupational considerations. It offers a minimally invasive option for diagnosis, treatment, and prevention of complications, ultimately improving patient outcomes and quality of life.
_x000D_
The other options are not optimal because:
_x000D_
· B. Chemical pleurodesis is a procedure that involves injecting a chemical agent (such as talc, doxycycline, or bleomycin) into the pleural space to cause inflammation and scarring, which seals the leak and prevents recurrence. Chemical pleurodesis can be effective, but it is associated with more pain, longer hospital stay, and higher risk of infection, and lower success rate than video assisted thoracoscopic surgery (VATS). Moreover, chemical pleurodesis is a treatment option employed in elderly patients who experience recurrent pneumothorax and are deemed high-risk candidates for surgery. As already mentioned earlier, this procedure involves the instillation of a chemical agent into the pleural space to induce inflammation and adhesion formation, ultimately preventing the recurrence of pneumothorax. However, it is important to note that the success rates of chemical pleurodesis can range from 10% to 20%, meaning that there is a possibility of the procedure not achieving the desired outcome in some cases.
_x000D_
- _x000D_
- Removing the drain and discharging the gentleman without any definitive treatment is not advisable, as he has a high risk of recurrence due to his history of previous pneumothorax, his occupation as a welder, and his ex-smoking status. Recurrent pneumothorax can cause more lung damage, respiratory failure, and even death.
- Rechecking if the gentleman still smokes and advising him to quit is a good preventive measure, as smoking is a major risk factor for pneumothorax. However, it is not sufficient as a definitive treatment, as he may still have a recurrence even if he quits smoking. Smoking cessation should be combined with a definitive procedure such as video assisted thoracoscopic surgery (VATS) or chemical pleurodesis.
- Advising the gentleman to rest for two weeks and then repeat the chest x-ray is not an effective treatment, as it does not address the underlying cause of the pneumothorax or prevent recurrence. Resting may help this gentleman to recover from the symptoms, but it does not seal the leak or prevent air from re-entering the pleural space. Repeating the chest x-ray may help to monitor the condition, but it does not provide a definitive diagnosis or treatment.
_x000D_
_x000D_
_x000D_
-
Question 54 of 123
54. Question
A 40-year-old lady presents to the respiratory clinic with shortness of breath.
_x000D_
She has been treated for asthma by her general physician with an inhaled steroid, but the general physician has documented an eosinophilia of 1.1×109/L (14%) (normal <0.1×10). Her serum creatinine has also been found to be 347 µmol/l (60-110).
_x000D_
Which one of the following would most support a diagnosis of eosinophilic granulomatosis with polyangiitis (EGPA)?
CorrectIncorrectHint
The correct answer is B. Extravascular eosinophils on vascular biopsy. Extravascular eosinophils are a hallmark of eosinophilic granulomatosis with polyangiitis (EGPA), as they indicate tissue infiltration and damage by these cells. Vascular biopsy is the gold standard for diagnosing eosinophilic granulomatosis with polyangiitis (EGPA), as it can reveal necrotizing vasculitis, granulomas, and eosinophils in the vessel wall or surrounding tissue. However, biopsy is not always feasible or positive, so other clinical and laboratory criteria are also used to support the diagnosis.
_x000D_
Let us now discuss the other answer options:
_x000D_
- _x000D_
- Fixed pulmonary infiltrates on chest radiographs: This is an incorrect answer. Fixed pulmonary infiltrates are not specific for eosinophilic granulomatosis with polyangiitis (EGPA), as they can be seen in other conditions that cause chronic lung inflammation or fibrosis, such as chronic eosinophilic pneumonia, sarcoidosis, or tuberculosis. Eosinophilic granulomatosis with polyangiitis (EGPA) can cause various pulmonary manifestations, such as asthma, eosinophilic pneumonia, pleural effusion, or pulmonary haemorrhage, but these are usually transient or migratory, not fixed.
- Peripheral ‘stocking’ neuropathy: This is not the correct answer because peripheral ‘stocking’ neuropathy is a type of peripheral nerve damage that affects the lower limbs in a symmetric pattern, usually starting from the toes and ascending upwards. It can be caused by various factors, such as diabetes mellitus, alcoholism, or vitamin B12 deficiency. Eosinophilic granulomatosis with polyangiitis (EGPA) can also cause peripheral neuropathy, but it is usually asymmetric and affects the upper and lower limbs, often sparing the feet. It is due to vasculitis of the vasa nervorum, the small blood vessels that supply the nerves.
- Peripheral alveolar filling infiltrate predominantly in the upper lobes on a chest radiograph: This is an incorrect answer because peripheral alveolar filling infiltrate is a radiographic pattern that shows fluffy or hazy opacities in the lung periphery, usually indicating alveolar oedema, pneumonia, or lung cancer. It is not a typical feature of eosinophilic granulomatosis with polyangiitis (EGPA), which tends to cause more central or diffuse pulmonary involvement. Moreover, eosinophilic granulomatosis with polyangiitis (EGPA) does not have a predilection for the upper lobes, unlike other granulomatous diseases such as tuberculosis or histoplasmosis.
- Peak flow <300 L/minute: This is an incorrect answer because peak flow is a measure of the maximum speed of expiration, which reflects the degree of airway obstruction. A peak flow <300 L/minute indicates severe airflow limitation, which can be seen in various respiratory diseases, such as asthma, chronic obstructive pulmonary disease (COPD), or bronchiectasis. It is not specific for eosinophilic granulomatosis with polyangiitis (EGPA), which can cause mild to severe asthma, depending on the individual patient. Furthermore, peak flow is not a reliable indicator of eosinophilic granulomatosis with polyangiitis (EGPA) activity, as it can vary depending on the time of day, medication use, or environmental factors.
_x000D_
_x000D_
_x000D_
_x000D_
-
Question 55 of 123
55. Question
A 40-year-old gentleman is referred to the Chest Clinic with a nine-month history of progressive shortness of breath. He has smoked 20 – 25/day for 20 years.
_x000D_
Investigations reveal a diagnosis of Moderate Emphysema. On questioning he informs the consultant that his father died from Chronic Obstructive Pulmonary Disease [COPD] in his early 50s. Following a diagnosis of Alpha-1 Antitrypsin (A1AT) Deficiency, he undergoes Genetic Testing and is found to have the PiSZ Genotype.
_x000D_
What levels of Alpha1 Antitrypsin would be expected if they were to be measured?
CorrectIncorrectHint
The Serum Levels of some of the Common Genotypes are:_x000D_
? PiMM: 100% (Normal)._x000D_
? PiMS: 80% of Normal Serum Level Of A1AT._x000D_
? PiSS: 60% Of Normal Serum Level Of A1AT._x000D_
? PiMZ: 60% Of Normal Serum Level Of A1AT._x000D_
? PiSZ: 40% Of Normal Serum Level Of A1AT._x000D_
? PiZZ: 10-15% (Severe Alpha 1-Antitrypsin Deficiency)._x000D_
Cigarette Smoking is especially harmful to those with A1AT Deficiency and can Accelerate the Progression of Emphysema by 10 years. -
Question 56 of 123
56. Question
A 40-year-old gentleman is admitted to the Intensive Care Unit for Ventilatory Support several hours following a Near Drowning Incident.
_x000D_
He is extremely Hypoxic and a Chest X-Ray shows Bilateral Infiltrates. He is diagnosed with Acute Respiratory Distress Syndrome (ARDS).
_x000D_
Which of the following is a direct pulmonary cause of Acute Respiratory Distress Syndrome [ARDS]?
CorrectIncorrectHint
This question addresses the phenomena of Acute Respiratory Distress Syndrome [ARDS] and its causes both Direct and Indirect._x000D_
Direct Pulmonary causes include:_x000D_
? Inhalation of Gastric Contents (pH <2)_x000D_
? Infective (Pneumonia, Tuberculosis)_x000D_
? Pulmonary Trauma_x000D_
? Near Drowning_x000D_
? Toxic Gas Inhalation and_x000D_
? Oxygen Toxicity._x000D_
Indirect causes include:_x000D_
? Sepsis_x000D_
? Non-Thoracic Trauma_x000D_
? Uraemia_x000D_
? Bowel Infraction_x000D_
? Anaphylaxis and_x000D_
? Burns._x000D_
Acute Respiratory Distress Syndrome [ARDS] Mortality is generally high (40%), but is determined by the cause with Aspiration Pneumonia having a mortality rate of almost 80% when associated with Acute Respiratory Distress Syndrome [ARDS]. -
Question 57 of 123
57. Question
A 40-year-old gentleman is admitted to the Intensive Care Unit (ICU) for ventilatory support several hours after being admitted to the emergency department following a near drowning incident.
_x000D_
He is extremely hypoxic and a chest x-ray shows bilateral infiltrates. He is diagnosed with acute respiratory distress syndrome (ARDS).
_x000D_
Which one of the following is a direct pulmonary cause of acute respiratory distress syndrome (ARDS)?
CorrectIncorrectHint
A direct pulmonary cause of acute respiratory distress syndrome (ARDS) is a cause that directly damages the lung tissue and leads to fluid leakage and inflammation in the air sacs (alveoli). Based on this definition, the correct answer is A. Tuberculosis. Tuberculosis is an infectious disease that affects the lungs and can cause granulomas, cavities, and fibrosis in the lung parenchyma. These lesions can impair the integrity of the alveolar-capillary membrane and allow fluid to accumulate in the alveoli, resulting in acute respiratory distress syndrome (ARDS).
_x000D_
The other options are incorrect, because they are indirect causes of acute respiratory distress syndrome (ARDS), meaning that they cause systemic inflammation or injury that affects the lungs secondarily. These include:
_x000D_
- _x000D_
- B. Sepsis, which is a serious and widespread infection of the bloodstream that can trigger a cytokine storm and increase the permeability of the alveolar-capillary membrane.
- C. Anaphylaxis, which is a severe allergic reaction that can cause bronchoconstriction, vasodilation, and increased vascular permeability.
- D. Post arrest, which refers to the period after a cardiac arrest, when the lack of blood flow and oxygen to the lungs can cause ischaemia-reperfusion injury and oxidative stress.
- E. Burns, which can cause a systemic inflammatory response syndrome and increase the capillary leakage in the lungs.
_x000D_
_x000D_
_x000D_
_x000D_
-
Question 58 of 123
58. Question
A 40-year-old Chef, who arrived in the United Kingdom from Burma seven months ago, presents to the Chest Clinic with a three-month history of weight loss, dry cough and night sweats.
_x000D_
Investigations reveal he has Multi-Drug-Resistant Tuberculosis (MDR-TB).
_x000D_
What is the minimum overall duration of treatment for Multi-Drug-Resistant Tuberculosis (MDR-TB) once the sputum is negative?
CorrectIncorrectHint
This question requires an understanding of the definition of Multi-Drug-Resistant Tuberculosis (MDR-TB) and the differences in treatment duration._x000D_
Multi-Drug-Resistant Tuberculosis (MDR-TB) as the name suggests is Mycobacterium tuberculosis resistant to two or more first line agents, which most commonly are isoniazid and rifampicin. There can be monoresistance to each of the first line agents, but this is not Multi-Drug-Resistant Tuberculosis (MDR-TB)._x000D_
Treatment of Multi-Drug-Resistant Tuberculosis (MDR-TB) is complex and time consuming. Treatment must be continued for a minimum of 18 months, with at least 9 months of treatment after the patient becomes culture-negative. _x000D_
Risk factors for acquiring Multi-Drug-Resistant Tuberculosis (MDR-TB) include previous Tuberculosis (TB) Treatment, Human Immunodeficiency Virus (HIV) Infection, Contact With Drug Resistant Disease and Treatment Failure._x000D_
Initial treatment includes the use of five agents until sputum is negative and then continuation of three to which the Tuberculosis (TB) is sensitive to for a minimum of nine months but sometimes up to 24 months. -
Question 59 of 123
59. Question
A 37-year-old lady presents with a two-month history of arthralgia, increasing fatigue and occasional nose bleeds. More recently she has become short of breath and has had three episodes of haemoptysis.
_x000D_
On investigation she is found to have acute kidney injury (AKI), with a serum creatinine of 665 μmol/L and urine dipstick is positive for blood and protein. A chest x-ray is performed which shows several nodules throughout both lung fields.
_x000D_
The treating physician suspects granulomatosis with polyangiitis.
_x000D_
Which one of the below autoantibodies is most associated with this condition?
CorrectIncorrectHint
The autoantibodies that are most associated with granulomatosis with polyangiitis (GPA) are B. PR3-anti-neutrophil cytoplasmic antibodies (ANCA)12. These are a type of autoantibodies that target proteinase 3 (PR3), an enzyme found in the cytoplasm of neutrophils. PR3-anti-neutrophil cytoplasmic antibodies (ANCA) are thought to play a role in the pathogenesis of granulomatosis with polyangiitis (GPA), as they can activate neutrophils and cause inflammation and damage to the blood vessels. PR3-anti-neutrophil cytoplasmic antibodies (ANCA) are detected in about 80-90% of patients with active granulomatosis with polyangiitis (GPA), and their levels may correlate with disease activity.
_x000D_
The other options are not associated with granulomatosis with polyangiitis (GPA), because:
_x000D_
- _x000D_
- A. MPO-anti-neutrophil cytoplasmic antibodies (ANCA) are another type of autoantibodies that target myeloperoxidase (MPO), an enzyme found in the cytoplasm of neutrophils. MPO-anti-neutrophil cytoplasmic antibodies (ANCA) are more commonly associated with microscopic polyangiitis (MPA) and eosinophilic granulomatosis with polyangiitis (EGPA), which are two other forms of anti-neutrophil cytoplasmic antibodies (ANCA)-associated vasculitis.
- C. Anti-dsDNA are autoantibodies that target double-stranded DNA, a component of the genetic material of cells. Anti-dsDNA are specific for systemic lupus erythematosus (SLE), a chronic autoimmune disease that can affect various organs.
- D. Anti-GBM are autoantibodies that target the glomerular basement membrane (GBM), a part of the kidney’s filtration system. Anti-GBM are responsible for anti-GBM disease, also known as Goodpasture syndrome, a rare condition that causes kidney failure and lung haemorrhage.
- E. p-anti-neutrophil cytoplasmic antibodies (p-ANCA) are a subtype of anti-neutrophil cytoplasmic antibodies (ANCA) that target perinuclear antigens, such as myeloperoxidase (MPO) or elastase. p-anti-neutrophil cytoplasmic antibodies (p-ANCA) are not specific for any disease, but they can be found in various conditions, such as inflammatory bowel disease, rheumatoid arthritis, or drug-induced vasculitis.
_x000D_
_x000D_
_x000D_
_x000D_
-
Question 60 of 123
60. Question
A 37-year-old lady presents with a two-month history of arthralgia, increasing fatigue and occasional nose bleeds. More recently she has become short of breath and has had one episodes of haemoptysis.
_x000D_
On investigation she is found to have Acute Kidney Injury [AKI], with a Serum Creatinine of 670 μmol/L and Urine Dipstick is Positive for Blood and Protein. A Chest X-Ray is performed which shows Several Nodules Both Lung Fields.
_x000D_
The Treating Physician suspects Granulomatosis with Polyangiitis.
_x000D_
Which of the below Autoantibodies are most associated with this condition?
CorrectIncorrectHint
Anti-Neutrophil Cytoplasmic Antibodies (ANCA) are diagnostic Markers for Vasculitis, although they may not be pathological. They are characterised by Neutrophil Staining; Cytoplasmic Staining Uptake (c-ANCA) and Peri-Nuclear (p-ANCA)._x000D_
These are largely synonymous with PR3-ANCA (targeting peroxidise-3) and MPO-ANCA (targeting myeloperoxidase)._x000D_
PR3-ANCA has 60-90% sensitivity and more than 90% specificity for Granulomatosis with Polyangiitis._x000D_
Churg-Strauss and Microscopic Polyangiitis are more commonly associated with MPO-Anti-Neutrophil Cytoplasmic Antibodies [ANCA] or p-Anti-Neutrophil Cytoplasmic Antibodies [ANCA], although may be positive for PR3 Anti-Neutrophil Cytoplasmic Antibodies [ANCA] also._x000D_
Goodpasture’s is associated with Anti-Glomerular Basement Membrane Antibody (Anti-GBM)._x000D_
Systemic Lupus Erythematosus (SLE) is associated with Anti-dsDNA antibodies. -
Question 61 of 123
61. Question
A 37-year-old lady presented at the clinic with daytime sleepiness and fatigue.
_x000D_
She does not take any regular medications. She denies snoring at night. On detailed history, she describes having episodes of probable cataplexy.
_x000D_
Clinical examination is unremarkable. Thyroid function tests are normal.
_x000D_
She is suspected of having narcolepsy.
_x000D_
Which one of the following tests will be most useful in diagnosis?
CorrectIncorrectHint
Multiple sleep latency test (option E) is the correct answer because the multiple sleep latency test (MSLT) is the most useful test for diagnosing narcolepsy in adults, according to the NICE guidelines. The MSLT measures how quickly a person falls asleep during the day and how often they enter rapid eye movement (REM) sleep. People with narcolepsy typically have a short sleep latency (less than 8 minutes) and at least 2 sleep-onset rapid eye movement (REM) periods. The multiple sleep latency test (MSLT) is usually done after an overnight polysomnography, which records the brain activity, eye movements, muscle tone, heart rate, and breathing during sleep. The multiple sleep latency test (MSLT) can help differentiate narcolepsy from other causes of excessive daytime sleepiness, such as sleep apnoea, idiopathic hypersomnia, or insufficient sleep syndrome.
_x000D_
The incorrect answer options can be ruled out with appropriate explanation for each of them as follows:
_x000D_
- _x000D_
- Trial of continuous positive airway pressure (CPAP): This is an incorrect answer because continuous positive airway pressure (CPAP) is a treatment for obstructive sleep apnoea, not narcolepsy. Continuous positive airway pressure (CPAP) delivers pressurized air through a mask to keep the airway open during sleep. Continuous positive airway pressure (CPAP) can improve the quality of sleep and reduce daytime sleepiness in people with sleep apnoea, but it has no effect on the symptoms of narcolepsy, such as cataplexy, sleep paralysis, or hallucinations.
- Genetic analysis: This is an incorrect answer because genetic analysis is not useful for diagnosing narcolepsy. Narcolepsy is not caused by a single gene, but by a complex interaction of genetic and environmental factors. About 98% of people with narcolepsy type 1 (with cataplexy) have a specific gene variant called HLA-DQB1*0602, but this gene is also present in 12% to 38% of the general population. Therefore, genetic testing cannot confirm or exclude the diagnosis of narcolepsy, and it is not recommended by the National Institute for Health and Care Excellence (NICE) guidelines.
- Polysomnography: This is an incorrect answer because polysomnography alone is not sufficient for diagnosing narcolepsy. Polysomnography is a test that records the brain activity, eye movements, muscle tone, heart rate, and breathing during sleep. Polysomnography can help rule out other sleep disorders, such as sleep apnoea, periodic limb movement disorder, or rapid eye movement (REM) sleep behaviour disorder. However, polysomnography cannot detect the abnormal sleep patterns that are characteristic of narcolepsy, such as short sleep latency and frequent sleep-onset rapid eye movement (REM) periods. Therefore, polysomnography is usually followed by the multiple sleep latency test (MSLT), which is the definitive test for narcolepsy.
- High hypocretin levels in cerebrospinal fluid (CSF) analysis: This is an incorrect answer because high hypocretin levels in cerebrospinal fluid (CSF) analysis are not a reliable marker of narcolepsy. Hypocretin (also known as orexin) is a neurotransmitter that regulates wakefulness and arousal. People with narcolepsy type 1 (with cataplexy) usually have low or undetectable levels of hypocretin in their cerebrospinal fluid (CSF), due to the loss of hypocretin-producing neurons in the hypothalamus. However, hypocretin levels can vary depending on the time of day, the stage of sleep, and the severity of the condition. Moreover, hypocretin levels are normal in people with narcolepsy type 2 (without cataplexy) and in some people with narcolepsy type 1. Therefore, hypocretin levels in cerebrospinal fluid (CSF) analysis are not a consistent or specific indicator of narcolepsy, and they are not recommended by the National Institute for Health and Care Excellence (NICE) guidelines.
_x000D_
_x000D_
_x000D_
_x000D_
-
Question 62 of 123
62. Question
A 37-year-old gentleman presents after two months of chronic cough with purulent sputum and shortness of breath on exertion.
_x000D_
He gives a history of at least two sinus or bronchial infections per year requiring treatment with antibiotics. He also says he and his wife have been unable to have children. He smokes 20 cigarettes per day.
_x000D_
Examination is normal except for some wheezing and an area of focal crackles at the left lung base. Chest X-Ray shows patchy infiltrates at both bases.
_x000D_
Investigations Revealed:
_x000D_
_x000D_ _x000D_
_x000D_ _x000D_ FEV1
_x000D_
_x000D_
_x000D_ 2.1 L
_x000D_
_x000D_
_x000D_
_x000D_ _x000D_ FVC
_x000D_
_x000D_
_x000D_ 2.6 L
_x000D_
_x000D_
_x000D_
_x000D_ _x000D_ pH
_x000D_
_x000D_
_x000D_ 7.40
_x000D_
_x000D_
_x000D_
_x000D_ _x000D_ PaCO2
_x000D_
_x000D_
_x000D_ 44 mmHg
_x000D_
_x000D_
_x000D_
_x000D_ _x000D_ PaO2
_x000D_
_x000D_
_x000D_ 84 mmHg
_x000D_
_x000D_
_x000D_
_x000D_
_x000D_
What is the most likely diagnosis?
CorrectIncorrectHint
Immotile Cilia Syndrome, also known as Primary Ciliary Dyskinesia and includes Kartagener’s Syndrome, is an inherited condition where the cilia lining the airways fail to function or function ineffectively._x000D_
A defect in the Dynein Molecule causes the cilia either totally to cease to function or to function ineffectively._x000D_
Kartagener’s Syndrome is a subset of patients that account for about half of all people with Immotile Cilia Syndrome._x000D_
Other associated conditions of Immotile Cilia Syndrome are:_x000D_
? Male Infertility_x000D_
? Congenital Heart Defects_x000D_
? Deafness_x000D_
? Hydrocephalus._x000D_
Cystic Fibrosis is unlikely to present at this age._x000D_
Infertility is not typically associated with Hypogammaglobulinaemia. -
Question 63 of 123
63. Question
A 36-year-old gentleman is referred to the medical admission unit with increasing breathlessness over the last six months since starting work as a stablehand at a new yard.
_x000D_
He reports initially symptoms of dyspnoea in the evenings with a dry cough. He has also noted occasional fevers at night. His breathing has become steadily worse.
_x000D_
A chest x-ray is performed which shows some fluffy nodular shadowing.
_x000D_
Which one of the following is the most likely causative agent for his condition?
CorrectIncorrectHint
The most likely causative agent for the condition of this gentleman is C. Thermophilic Actinomyces bacteria.
_x000D_
Thermophilic Actinomyces bacteria are a group of bacteria that thrive in warm and moist environments, such as hay, compost, and animal bedding. They can cause a type of hypersensitivity pneumonitis, also known as farmer’s lung, which is an allergic reaction to inhaled organic dusts.
_x000D_
Farmer’s lung typically presents with breathlessness, dry cough, fever, and malaise, especially after exposure to the offending dust. The symptoms may be acute, subacute, or chronic, depending on the frequency and intensity of exposure.
_x000D_
The chest x-ray of farmer’s lung may show fluffy nodular shadowing, which represents areas of inflammation and fibrosis in the lung parenchyma. The nodules are usually bilateral and symmetric, and may coalesce to form larger opacities.
_x000D_
The other options are less likely for the following reasons:
_x000D_
- _x000D_
- Penicillium species are fungi that can cause allergic bronchopulmonary aspergillosis (ABPA), a hypersensitivity reaction to Aspergillus antigens in the airways. Allergic bronchopulmonary aspergillosis (ABPA) usually affects patients with asthma or cystic fibrosis, and presents with wheezing, cough, and expectoration of brownish plugs. The chest x-ray of allergic bronchopulmonary aspergillosis (ABPA) may show central bronchiectasis, mucoid impaction, or transient infiltrates.
- Epicoccum nigrum is a fungus that can cause allergic fungal sinusitis (AFS), a hypersensitivity reaction to fungal antigens in the paranasal sinuses. Allergic fungal sinusitis (AFS) usually affects patients with chronic rhinosinusitis, and presents with nasal congestion, discharge, and polyps. The chest x-ray of allergic fungal sinusitis (AFS) is usually normal, unless there is secondary pulmonary involvement.
- Avian proteins are proteins derived from birds, such as feathers, droppings, or serum. They can cause bird fancier’s lung, another type of hypersensitivity pneumonitis, which is similar to farmer’s lung but occurs in people who are exposed to birds or their products. The chest x-ray of bird fancier’s lung may also show fluffy nodular shadowing, but the history of working as a stablehand makes this less likely than farmer’s lung.
- Klebsiella is a bacterium that can cause pneumonia, especially in patients with underlying lung disease, alcoholism, or diabetes. Klebsiella pneumonia usually presents with fever, productive cough, and pleuritic chest pain. The chest x-ray of Klebsiella pneumonia may show lobar consolidation, cavitation, or abscess formation.
_x000D_
_x000D_
_x000D_
_x000D_
-
Question 64 of 123
64. Question
A 36-year-old gentleman is referred to the Medical Admission Unit with increasing breathlessness over the last six months since starting work as a Stablehand at a New Yard.
_x000D_
He reports initially symptoms of dyspnoea in the evenings with a dry cough. He has also noted occasional fevers at night. His breathing has become steadily worse.
_x000D_
A Chest X-Ray is performed which shows some fluffy nodular shadowing.
_x000D_
What is the most likely causative agent for his condition?
CorrectIncorrectHint
The most common example of Allergic Alveolitis is Farmer’s Lung._x000D_
This is caused by dust from mouldy hay contaminated with Thermophilic Actinomyces bacteria, and as a Stablehand this gentleman is likely to have been in close contact with this allergen. Typically symptoms tend to occur several hours after exposure. Chest X-Ray may demonstrate fluffy nodular shadowing or ground glass appearances._x000D_
Avian Proteins are the allergen responsible for pigeon fancier’s lung._x000D_
Penicillium species are responsible for respiratory disease in cheese and cork workers and are due to fungal contaminants of materials._x000D_
Klebsiella and Epicoccum nigrum are other bacterial causes of Allergic Alveolitis and are found in contaminated water, wood shavings, etc. -
Question 65 of 123
65. Question
A 33-year-old lady presents to the Emergency Department of her Local Hospital.
_x000D_
She is complaining of increasing breathless on exertion over the last few weeks. She also reports a dry cough and occasional night sweats. Of note, she is under the care of the Infectious Disease Team for Human Immunodeficiency Virus [HIV] and is also 26 weeks pregnant.
_x000D_
Examination reveals that she is hypoxic, with her oxygen saturations falling still when she is asked to walk. A Departmental Chest X-Ray demonstrates Bilateral Infiltrates.
_x000D_
What is the most appropriate antibiotic treatment for this lady given the likely diagnosis?
CorrectIncorrectHint
This patient is immunocompromised and given the clinical history and radiological findings, the likely diagnosis in this lady is Pneumocystis pneumonia (PCP). She is also pregnant which affects treatment choice._x000D_
Although Pneumocystis pneumonia (PCP) is officially a fungal infection it does not respond to anti-fungal medications._x000D_
Trimethoprim-Sulfamethoxazole (TMP-SMX / Co-Trimoxazole / Septrin) has been shown to be as effective as Intravenous Pentamidine and more effective than other alternative treatment regimens._x000D_
TMP-SMX is the preferred initial therapy during pregnancy according to consensus guidelines. The BNF States that there is a Teratogenic Risk in the First Trimester (as Trimethoprim is a Folate Antagonist), and Neonatal Haemolysis and Methaemoglobinaemia in the Third Trimester. However, there is also considerable risk of harm to the foetus if the mother is unwell. The benefits in this situation therefore outweigh the risks, and it should be used._x000D_
For the treatment of infections that are resistant to TMP-SMX, the combination of Clindamycin and Primaquine is likely to be more effective than Intravenous Pentamidine._x000D_
Rifampicin, Ethambutol, Isoniazid, Pyrazinamide are the first line drugs to treat Tuberculosis (TB). Although this woman is at risk from Tuberculosis (TB) infection, the clinical history is more in keeping with a diagnosis of Pneumocystis pneumonia (PCP)._x000D_
Adjuvant Steroid Therapy is indicated in patients with severe Pneumocystis pneumonia (PCP) infection who have underlying Human Immunodeficiency Virus [HIV] Infection. -
Question 66 of 123
66. Question
A 33-year-old gentleman, who is healthy, fit and well, was driving a car at high speed and got involved in a Road Traffic Accident two days ago.
_x000D_
He has sustained a closed fracture of his femur, which has been treated surgically with an Intramedullary Nail, as well as fractures of his right clavicle and left radius. He was managed according to Advanced Trauma Life Support Protocol [ATLS Protocol] when he attended the Emergency Department.
_x000D_
On examination, he is acutely short of breath and has a temperature of 37.5°C. The patient seems confused when the Treating Physician speaks to him, and as he examines him, he notes petechial haemorrhages.
_x000D_
Which do you think is the most likely diagnosis?
CorrectIncorrectHint
The two diagnoses which should be considered first in this scenario are Pulmonary Embolism and Fat Embolism._x000D_
Although the patient is at high risk of Pulmonary Embolism, and appropriate measures should be undertaken to reduce this, the clinical scenario is more suggestive of Fat Embolism. Fat Embolism is thought to occur as a result of release of Lipid Globules from Damaged Bone Marrow Fat Cells. Another mechanism that has been suggested is the Increased Mobilisation Of Fatty Acids Peripherally._x000D_
The effects that are seen clinically depend on what part of the Microvasculature is affected by the Lipid Globules._x000D_
Pulmonary Symptoms are caused by Ventilation Perfusion Mismatch._x000D_
Confusion (Cerebral Effects) may be seen, as well as a petechial rash caused by capillary damage in the skin. -
Question 67 of 123
67. Question
A 32-year-old gentleman who is a lifelong non-smoker presents to the pulmonology clinic with shortness of breath, wheeze and a chronic cough. He works as a stone mason, carving grave stones and has a property on a farm.
_x000D_
At the same time, his father maintains that he only ever used to smoke between five and ten cigarettes per day, yet he is 59-years-old with severe chronic obstructive pulmonary disease (COPD) on home oxygen.
_x000D_
On examination, the gentleman has bilateral wheeze and coarse crackles consistent with obstructive lung disease.
_x000D_
Investigations Show:
_x000D_
_x000D_ _x000D_
_x000D_ _x000D_ Haemoglobin
_x000D_
_x000D_
_x000D_ 121 g/L
_x000D_
_x000D_
_x000D_ (135-177)
_x000D_
_x000D_
_x000D_
_x000D_ _x000D_ White Blood Cell Count
_x000D_
_x000D_
_x000D_ 6.2 ×109/L
_x000D_
_x000D_
_x000D_ (4-11)
_x000D_
_x000D_
_x000D_
_x000D_ _x000D_ Platelet Count
_x000D_
_x000D_
_x000D_ 172 ×109/L
_x000D_
_x000D_
_x000D_ (150-400)
_x000D_
_x000D_
_x000D_
_x000D_ _x000D_ Serum Sodium
_x000D_
_x000D_
_x000D_ 137 mmol/L
_x000D_
_x000D_
_x000D_ (135-146)
_x000D_
_x000D_
_x000D_
_x000D_ _x000D_ Serum Potassium
_x000D_
_x000D_
_x000D_ 4.2 mmol/L
_x000D_
_x000D_
_x000D_ (3.5-5)
_x000D_
_x000D_
_x000D_
_x000D_ _x000D_ Serum Creatinine
_x000D_
_x000D_
_x000D_ 88 μmol/L
_x000D_
_x000D_
_x000D_ (79-118)
_x000D_
_x000D_
_x000D_
_x000D_ _x000D_ Serum Alanine Aminotransferase [ALT]
_x000D_
_x000D_
_x000D_ 90 U/L
_x000D_
_x000D_
_x000D_ (5-40)
_x000D_
_x000D_
_x000D_
_x000D_ _x000D_ Peak Expiratory Flow Rate [PEFR]
_x000D_
_x000D_
_x000D_ 278 l/min
_x000D_
_x000D_
_x000D_ (Predicted 550)
_x000D_
_x000D_
_x000D_
_x000D_ _x000D_ Chest X-Ray
_x000D_
_x000D_
_x000D_ Evidence Of Lower Lobe Emphysema
_x000D_
_x000D_
_x000D_
_x000D_
_x000D_
Which one of the following is the most likely diagnosis?
CorrectIncorrectHint
Based on the clinical history and family history of this gentleman in this question, the most likely diagnosis is E. Alpha-1 antitrypsin deficiency. This is a genetic disorder that causes low levels of a protein (alpha-1 antitrypsin) that protects the lungs from damage. People with this condition are at risk of developing emphysema, cirrhosis, and other diseases, especially if they smoke or are exposed to dust.
_x000D_
Some of the clues that point to this diagnosis are:
_x000D_
- _x000D_
- The gentleman is a lifelong non-smoker, yet he has symptoms of obstructive lung disease similar to emphysema.
- The father of this gentleman has severe chronic obstructive pulmonary disease (COPD) at a young age, despite smoking only a few cigarettes per day. This suggests a familial predisposition to lung disease.
- The gentleman has evidence of lower lobe emphysema on chest x-ray, which is typical of alpha-1 antitrypsin deficiency.
- The gentleman has elevated serum alanine aminotransaminase (ALT), which indicates liver damage. This can be caused by the accumulation of abnormal alpha-1 antitrypsin in the liver.
- The gentleman’s other blood tests are normal or near normal, which rules out other causes of lung disease such as asthma, silicosis, extrinsic allergic alveolitis, or chronic bronchitis.
_x000D_
_x000D_
_x000D_
_x000D_
_x000D_
_x000D_
The diagnosis can be confirmed by measuring the level of alpha-1 antitrypsin in the blood or by genetic testing.
_x000D_
The treatment of alpha-1 antitrypsin (AAT) deficiency may include medications, intravenous infusions of alpha-1 antitrypsin protein, or in severe cases, lung or liver transplantation. This gentleman should also avoid smoking and environmental dust, and receive vaccinations for influenza, pneumococcus, and hepatitis.
_x000D_
The other answer options in the above question can be ruled out based on the following explanations:
_x000D_
- _x000D_
- A. Asthma is a chronic inflammatory disorder of the airways that causes reversible bronchoconstriction and increased mucus production. Asthma is usually associated with a history of atopy, allergic triggers, episodic symptoms, and a positive response to bronchodilators. Asthma does not typically cause lower lobe emphysema or elevated serum alanine aminotransaminase (ALT).
- B. Silicosis is an occupational lung disease caused by inhalation of silica dust, which is commonly found in stone masonry, mining, and sandblasting. Silicosis causes fibrotic nodules in the upper lobes of the lungs, which can coalesce into progressive massive fibrosis. Silicosis also increases the risk of tuberculosis and lung cancer. Silicosis does not cause wheeze, lower lobe emphysema, or elevated serum alanine aminotransaminase (ALT).
- C. Extrinsic allergic alveolitis, also known as hypersensitivity pneumonitis, is an immune-mediated lung disease caused by repeated inhalation of various organic antigens, such as molds, bacteria, bird droppings, or animal proteins. Extrinsic allergic alveolitis causes granulomatous inflammation and fibrosis in the lung parenchyma and small airways. Extrinsic allergic alveolitis may present with acute, subacute, or chronic symptoms, depending on the type and duration of exposure. Extrinsic allergic alveolitis does not cause lower lobe emphysema or elevated serum alanine aminotransaminase (ALT).
- D. Chronic bronchitis is a form of chronic obstructive pulmonary disease (COPD) characterized by a productive cough for at least three months in each of two consecutive years. Chronic bronchitis is usually caused by cigarette smoking or other environmental irritants. Chronic bronchitis causes inflammation and mucus hypersecretion in the large airways, leading to airflow obstruction and increased risk of respiratory infections. Chronic bronchitis does not cause lower lobe emphysema or elevated serum alanine aminotransaminase (ALT).
_x000D_
_x000D_
_x000D_
_x000D_
-
Question 68 of 123
68. Question
A 45-year-old gentleman presents to the Chest Clinic with a long history of productive cough and breathlessness. He had complained of halitosis and exacerbations of productive sputum, chest pain and haemoptysis.
_x000D_
Chest examination revealed bilateral inspiratory crackles.
_x000D_
Which one of the following treatments is likely to decrease the frequency of his exacerbations?
CorrectIncorrectHint
The presence of bronchiectasis in this gentleman is supported by several clinical findings. Firstly, he regularly produces sputum and experiences breathlessness, which are common symptoms of bronchiectasis. Additionally, he has a history of recurrent lung infections, which is another characteristic feature of this condition. Furthermore, the presence of bilateral inspiratory crackles during lung examination is indicative of widespread disease in the bronchial tubes. Bronchiectasis is a chronic condition characterized by the permanent dilation and damage of the bronchial tubes, leading to the accumulation of mucus and recurrent infections. The production of sputum, breathlessness, and lung infections are all consequences of the impaired clearance of mucus from the airways. The bilateral inspiratory crackles heard during lung examination suggest that the bronchiectasis has affected multiple areas of the lungs, further supporting the diagnosis of widespread disease in this gentleman.
_x000D_
Here, the answer to above question is B. Postural drainage. This is a type of airway clearance technique that involves positioning the body in different ways to help drain mucus from the lungs. Postural drainage can reduce the risk of infections and improve lung function in people with bronchiectasis.
_x000D_
Other treatments for bronchiectasis may include antibiotics, mucus thinning medication, airway clearance devices, chest physical therapy, oxygen therapy, or surgery, depending on the severity and cause of the condition. It is also important to quit smoking, maintain a healthy diet, stay hydrated, and get vaccinated to prevent complications and flare-ups.
_x000D_
The other options given in the above question are not likely to decrease the frequency of exacerbations in this gentleman because:
_x000D_
- _x000D_
- A. Inhaled corticosteroids: These are anti-inflammatory drugs that can help reduce airway inflammation and improve lung function in some people with bronchiectasis. However, they do not directly affect the frequency of exacerbations, and their long-term use may have adverse effects such as osteoporosis, diabetes, and cataracts.
- C. Cyclical antibiotic therapy: This is a preventive strategy that involves taking antibiotics regularly for a certain period of time, such as every month or every other month, to reduce the bacterial load and prevent infections. It may be beneficial for some people with frequent or severe exacerbations, but it is not recommended for everyone, as it may increase the risk of antibiotic resistance, side effects, and drug interactions.
- D. Surgical resection: This is a procedure that involves removing the diseased part of the lung that is affected by bronchiectasis. It may be considered as a last resort for people who have localized bronchiectasis that does not respond to medical therapy, or who have life-threatening complications such as massive haemoptysis (coughing up blood). However, it is not a common or routine treatment, as it carries significant risks and does not cure the underlying condition.
- E. Nebulised bronchodilators: These are drugs that are inhaled through a nebulizer, a device that turns liquid medicine into a fine mist that can be breathed in. They work by relaxing the muscles around the airways and making it easier to breathe. They may be helpful for some people with bronchiectasis, who also have asthma or chronic obstructive pulmonary disease (COPD), but they do not directly affect the frequency of exacerbations, and their use should be guided by a lung function test.
_x000D_
_x000D_
_x000D_
_x000D_
-
Question 69 of 123
69. Question
A 45-year-old gentleman presents to the Chest Clinic with a long history of productive cough and breathlessness. He had complained of halitosis and exacerbations of productive cough, chest pain and haemoptysis.
_x000D_
Chest Examination revealed Bilateral Inspiratory Crackles.
_x000D_
Which of the following treatments is likely to decrease the frequency of his exacerbations?
CorrectIncorrectHint
This man has Bronchiectasis as evidenced by his regular production of sputum associated with breathlessness, his repeated lung infections and the signs of bilateral inspiratory crackles._x000D_
Retained mucus is the most important reason why bronchiectatic patients become infected. Postural drainage is, therefore, the cornerstone to treating bronchiectasis and should be undertaken at least once per day and more frequently during exacerbations._x000D_
There have been trials looking at regular antibiotic therapy versus symptomatic treatment in patients with cystic fibrosis colonised with Pseudomonas (for example, Elborn JS et al. Thorax 2000;55: 355-358) but there is currently no evidence that this approach is of benefit in bronchiectasis._x000D_
Similarly, inhaled corticosteroids should not be used routinely in bronchiectasis until further evidence of their effect on lung function and exacerbation frequency is available._x000D_
Surgical resection as a curative procedure can be performed for localised disease when underlying causes such as primary ciliary dyskinesia have been excluded._x000D_
In this patient, the bilateral crackles suggest widespread disease. -
Question 70 of 123
70. Question
A 45-year-old gentleman is referred to the Chest Clinic with worsening asthma symptoms. He had been diagnosed with late onset asthma aged 35 years.
_x000D_
On questioning the patient reports a short history of malaise, fever and tender subcutaneous nodules on his legs. He has had no haemoptysis.
_x000D_
A Complete Blood Count is performed and the results are as follows:
_x000D_
_x000D_ _x000D_
_x000D_ _x000D_ Haemoglobin
_x000D_
_x000D_
_x000D_ 150 g/L
_x000D_
_x000D_
_x000D_ (130-180)
_x000D_
_x000D_
_x000D_
_x000D_ _x000D_ Platelet Count
_x000D_
_x000D_
_x000D_ 245 ×109/L
_x000D_
_x000D_
_x000D_ (150-400)
_x000D_
_x000D_
_x000D_
_x000D_ _x000D_ White Blood Cell Count
_x000D_
_x000D_
_x000D_ 12.7 ×109/L
_x000D_
_x000D_
_x000D_ (4-11)
_x000D_
_x000D_
_x000D_
_x000D_ _x000D_ Neutrophils
_x000D_
_x000D_
_x000D_ 7.7 ×109/L
_x000D_
_x000D_
_x000D_ (1.5-7)
_x000D_
_x000D_
_x000D_
_x000D_ _x000D_ Lymphocytes
_x000D_
_x000D_
_x000D_ 2.6 ×109/L
_x000D_
_x000D_
_x000D_ (1.5-4)
_x000D_
_x000D_
_x000D_
_x000D_ _x000D_ Monocytes
_x000D_
_x000D_
_x000D_ 0.1 ×109/L
_x000D_
_x000D_
_x000D_ (0-0.8)
_x000D_
_x000D_
_x000D_
_x000D_ _x000D_ Eosinophils
_x000D_
_x000D_
_x000D_ 2.1 ×109/L
_x000D_
_x000D_
_x000D_ (0.04-0.4)
_x000D_
_x000D_
_x000D_
_x000D_ _x000D_ Basophils
_x000D_
_x000D_
_x000D_ 0.07 ×109/L
_x000D_
_x000D_
_x000D_ (0-0.1)
_x000D_
_x000D_
_x000D_
_x000D_
_x000D_
A Complete Blood Count from three years earlier was reviewed and its results were as follows:
_x000D_
_x000D_ _x000D_
_x000D_ _x000D_ Haemoglobin
_x000D_
_x000D_
_x000D_ 123 g/L
_x000D_
_x000D_
_x000D_ (130-180)
_x000D_
_x000D_
_x000D_
_x000D_ _x000D_ Platelet Count
_x000D_
_x000D_
_x000D_ 164 ×109/L
_x000D_
_x000D_
_x000D_ (150-400)
_x000D_
_x000D_
_x000D_
_x000D_ _x000D_ White Blood Cell Count
_x000D_
_x000D_
_x000D_ 9.4 ×109/L
_x000D_
_x000D_
_x000D_ (4-11)
_x000D_
_x000D_
_x000D_
_x000D_ _x000D_ Neutrophils
_x000D_
_x000D_
_x000D_ 5.2 ×109/L
_x000D_
_x000D_
_x000D_ (1.5-7)
_x000D_
_x000D_
_x000D_
_x000D_ _x000D_ Lymphocytes
_x000D_
_x000D_
_x000D_ 3.3 ×109/L
_x000D_
_x000D_
_x000D_ (1.5-4)
_x000D_
_x000D_
_x000D_
_x000D_ _x000D_ Monocytes
_x000D_
_x000D_
_x000D_ 0.5 ×109/L
_x000D_
_x000D_
_x000D_ (0-0.8)
_x000D_
_x000D_
_x000D_
_x000D_ _x000D_ Eosinophils
_x000D_
_x000D_
_x000D_ 0.7 ×109/L
_x000D_
_x000D_
_x000D_ (0.04-0.4)
_x000D_
_x000D_
_x000D_
_x000D_ _x000D_ Basophils
_x000D_
_x000D_
_x000D_ 0.1 ×109/L
_x000D_
_x000D_
_x000D_ (0-0.1)
_x000D_
_x000D_
_x000D_
_x000D_
_x000D_
A Chest X-Ray is performed which shows Patchy Pulmonary Infiltrates.
_x000D_
Given the patient’s history and the results of initial investigations, which is the most likely diagnosis?
CorrectIncorrectHint
Churg-Strauss Syndrome (CSS) is a rare form of small-vessel vasculitis, characterised by asthma, allergic rhinitis and prominent peripheral blood eosinophilia. Two of these are present in the history and should lead you to the diagnosis of Churg Strauss._x000D_
The most commonly involved organ is the lung, followed by the skin. CSS, however, can affect any organ system, including the cardiovascular, gastrointestinal, renal, and central nervous systems. The unifying feature of patients presenting with CSS is asthma. Vasculitis involving the peripheral nervous system is also a characteristic feature, and mononeuritis multiplex occurs in 75% of patients._x000D_
Vasculitis of extrapulmonary organs is largely responsible for the morbidity and mortality associated with CSS. 40-60% are associated with positive ANCA, usually pANCA/MPO._x000D_
ANCA are antineutrophil cytoplasmic antibodies. Several different staining patterns and antigen specificities are recognised. The cytoplasmic staining pattern, cANCA, and specificity for the PR3 antigen is most specific for Granulomatosis with polyangiitis. Perinuclear staining, pANCA, and/or antibody to MPO are far less specific than cANCA and can be present in a range of inflammatory conditions such as microscopic polyangitis, Churg-Strauss syndrome and Goodpasture’s syndrome. MPO and pANCA may also be present in SLE, rheumatoid arthritis, Sj?gren’s syndrome and occasionally in chronic infections._x000D_
Intravenous Glucocorticoid is used for initial therapy of acute multi-organ involvement in Churg-Strauss Syndrome, followed by Oral Glucocorticoid Therapy, often with Azathioprine as a steroid-sparing agent._x000D_
Granulomatosis with Polyangiitis is a multi-organ autoimmune disease, which can be fatal. The classical triad consists of Necrotising Granulomatous Inflammation of the Respiratory Tract, Glomerulonephritis and a Small-Vessel Vasculitis. A prolonged history of Epistaxis or Sinusitis is commonly found in Granulomatosis with Polyangiitis, which in some patients is also associated with an Eosinophilia._x000D_
Microscopic Polyangiitis is microscopically similar to WG, but does not tend to involve the Nasopharynx. It has an association with Hepatitis B and C._x000D_
Acute Respiratory Distress Syndrome (ARDS) describes an Acute Diffuse Inflammatory Lung Injury, often in previously healthy lungs. It has an acute onset, and patients often require respiratory support. It can be precipitated by a number of insults, including drugs and pancreatitis._x000D_
The presence of systemic symptoms makes a diagnosis of asthma alone unlikely. -
Question 71 of 123
71. Question
A 45-year-old Caribbean lady, a comprehensive school teacher, complains of slowly increasing breathlessness. She has no smoking history.
_x000D_
Investigations reveal she has bilateral enlarged hilar lymph nodes, elevated serum calcium, interstitial lung disease, and enlarged liver and spleen.
_x000D_
Which one of the following is the most anticipated diagnosis?
CorrectIncorrectHint
The most anticipated diagnosis for this lady is A. Sarcoidosis. Sarcoidosis is a chronic inflammatory disorder that can affect multiple organs, especially the lungs and lymph nodes. It causes the formation of granulomas, which are clusters of immune cells that can interfere with normal organ function. Sarcoidosis is more common in people of African or Caribbean descent, and often presents with respiratory symptoms, such as breathlessness, cough, and chest pain. Sarcoidosis can also cause elevated serum calcium levels, due to increased production of vitamin D by the granulomas. Sarcoidosis can affect the liver and spleen, causing enlargement and dysfunction. Sarcoidosis is diagnosed by blood tests, chest x-ray, and biopsy of affected tissues.
_x000D_
The other options are less likely, based on the clinical presentation of this lady and her investigations. Some reasons for this are as below:
_x000D_
- _x000D_
- B. Hyperparathyroidism is a condition in which the parathyroid glands produce too much parathyroid hormone, which regulates calcium and phosphorus metabolism. Hyperparathyroidism can cause hypercalcaemia, but it does not typically cause bilateral enlarged hilar lymph nodes, interstitial lung disease, or hepatosplenomegaly. Hyperparathyroidism is diagnosed by measuring parathyroid hormone levels, along with calcium, phosphorus, and vitamin D levels.
- C. Tuberculosis is an infectious disease caused by the bacterium Mycobacterium tuberculosis, which mainly affects the lungs, but can also spread to other organs. Tuberculosis can cause pulmonary symptoms, such as cough, fever, and weight loss, and can also cause enlarged lymph nodes, liver, and spleen. In this case, tuberculosis (TB) is indeed a differential diagnosis. However, tuberculosis usually causes caseating granulomas, which are different from the non-caseating granulomas seen in sarcoidosis. Tuberculosis is diagnosed by sputum smear and culture, chest x-ray, and skin or blood tests.
- D. Coccidioidomycosis is a non-infective fungal infection caused by inhaling spores of the fungus Coccidioides immitis, which is endemic in certain regions of the south-western United States, Mexico, and Central and South America. Coccidioidomycosis can cause pulmonary symptoms, such as cough, fever, and chest pain, and can also disseminate to other organs, such as the skin, bones, and meninges. Coccidioidomycosis can cause granulomas and elevated serum calcium levels, but it is less common than sarcoidosis, and the patient does not have any history of travel or exposure to endemic areas. Coccidioidomycosis is diagnosed by blood tests, sputum culture, and tissue biopsy.
- E. Hypervitaminosis D is a rare but potentially serious condition that occurs when there is too much vitamin D in the body, either from excessive intake of supplements, or from certain medical conditions or medications that increase vitamin D levels. Hypervitaminosis D can cause hypercalcaemia, but it does not cause bilateral enlarged hilar lymph nodes, interstitial lung disease, or hepatosplenomegaly. Hypervitaminosis D is diagnosed by measuring vitamin D levels, along with calcium, phosphorus, and kidney function tests.
_x000D_
_x000D_
_x000D_
_x000D_
-
Question 72 of 123
72. Question
A 44-year-old lady, with disseminated malignancy and unknown primary, presents with oedema of the arms and face, with dilated neck veins.
_x000D_
You suspect Superior Vena Cava Obstruction (SVCO).
_x000D_
Which of the following statements is correct?
CorrectIncorrectHint
This is an Oncological Emergency._x000D_
Mediastinal Radiotherapy leads to symptomatic relief in 80% of patients, although case studies have shown this does not always correlate to patency of the superior vena cava. _x000D_
If possible, an attempt should be made to obtain a tissue diagnosis, as some tumours respond to radiotherapy whereas others are more sensitive to chemotherapy. Tumours which are very chemosensitive, such as germ cell and lymphoma, can cause superior vena cava obstruction. _x000D_
Therefore the active pursuit of a diagnosis with active treatment of the SVCO is indicated and not merely palliative measures, especially in a patient this young._x000D_
Non-Small Cell Cancer and Small Cell Malignancy may both cause Superior Vena Cava Obstruction [SVCO]._x000D_
Intravenous Dexamethasone at high dose is of benefit in severe cases of SVCO._x000D_
It is important to note that in 2004 NICE recommended considering stenting in the majority of cases of Superior Vena Cava Obstruction [SVCO]. This is a minimally invasive procedure which relieves symptoms quicker than chemotherapy or radiotherapy. -
Question 73 of 123
73. Question
A 44-year-old lady comes to the pulmonology clinic with complaint of increasing shortness of breath. She has a history of systemic sclerosis, takes omeprazole for reflux symptoms and nifedipine slow release for Raynaud’s disease.
_x000D_
On examination, her blood pressure is 150/80 mmHg, pulse rate is 82 beats per minute and regular. There is peripheral calcinosis on examination of the hands and pinching of the corners of the mouth consistent with scleroderma. Auscultation of the chest reveals scattered inspiratory crackles across both lung fields.
_x000D_
Investigations Show:
_x000D_
_x000D_ _x000D_
_x000D_ _x000D_ Haemoglobin
_x000D_
_x000D_
_x000D_ 124 g/L
_x000D_
_x000D_
_x000D_ (115-160)
_x000D_
_x000D_
_x000D_
_x000D_ _x000D_ Total Leucocyte Count
_x000D_
_x000D_
_x000D_ 8.3 ×109/L
_x000D_
_x000D_
_x000D_ (4-11)
_x000D_
_x000D_
_x000D_
_x000D_ _x000D_ Platelet Count
_x000D_
_x000D_
_x000D_ 201 ×109/L
_x000D_
_x000D_
_x000D_ (150-400)
_x000D_
_x000D_
_x000D_
_x000D_ _x000D_ Erythrocyte Sedimentation Rate (ESR)
_x000D_
_x000D_
_x000D_ 65 mm/hr
_x000D_
_x000D_
_x000D_ (<10)
_x000D_
_x000D_
_x000D_
_x000D_ _x000D_ Serum Sodium
_x000D_
_x000D_
_x000D_ 140 mmol/L
_x000D_
_x000D_
_x000D_ (135-146)
_x000D_
_x000D_
_x000D_
_x000D_ _x000D_ Serum Potassium
_x000D_
_x000D_
_x000D_ 4.5 mmol/L
_x000D_
_x000D_
_x000D_ (3.5-5)
_x000D_
_x000D_
_x000D_
_x000D_ _x000D_ Serum Creatinine
_x000D_
_x000D_
_x000D_ 134 µmol/L
_x000D_
_x000D_
_x000D_ (79-118)
_x000D_
_x000D_
_x000D_
_x000D_
_x000D_
Chest x-ray shows bilateral interstitial infiltrates. Echocardiogram shows ejection fraction of 53%.
_x000D_
Which one of the following is most likely the confirmative diagnosis?
CorrectIncorrectHint
Based on the clinical presentation of this lady and her investigations, the most likely confirmative diagnosis for her is B. Interstitial fibrosis. Interstitial fibrosis is a type of interstitial lung disease (ILD) that causes progressive scarring of the lung tissue. It is a common complication of systemic sclerosis (SSc), a connective tissue disease that affects the skin and multiple internal organs. The risk of developing interstitial lung disease (ILD) is higher in patients with diffuse cutaneous systemic sclerosis (SSc), anti-Scl-70/anti-topoisomerase I antibody, and in the absence of anti-centromere antibody.
_x000D_
This lady’s history of systemic sclerosis (SSc), peripheral calcinosis, and Raynaud’s disease are suggestive of diffuse cutaneous systemic sclerosis (SSc). Her symptoms of increasing shortness of breath and dry cough, along with the physical findings of inspiratory crackles and the chest x-ray showing bilateral interstitial infiltrates, are consistent with interstitial lung disease (ILD). The elevated erythrocyte sedimentation rate (ESR) and serum creatinine indicate inflammation and renal impairment, which are also common in systemic sclerosis (SSc). The echocardiogram shows a normal ejection fraction, which makes pulmonary oedema unlikely. The other options (idiopathic pulmonary fibrosis, eosinophilic pneumonitis, and bacterial pneumonia) are less likely because they are not associated with systemic sclerosis (SSc) and have different clinical and radiographic features (we will rule out these options elaborately with explanations below).
_x000D_
To confirm the diagnosis of interstitial fibrosis, this lady may need further tests, such as high-resolution computed tomography (HRCT) of the chest, pulmonary function tests, and bronchoscopy or surgical lung biopsy. The treatment of interstitial fibrosis depends on the severity and progression of the disease, and may include immunosuppressant therapies, ant fibrotic agents, and lung transplantation.
_x000D_
The incorrect answer options for the above question can be ruled out as follows:
_x000D_
- _x000D_
- A. Pulmonary oedema: This is a condition where fluid accumulates in the lungs, causing difficulty in breathing. It can be caused by heart failure, kidney failure, lung infections, or high altitude. This lady does not have any signs of heart failure, such as elevated jugular venous pressure, peripheral oedema, or S3 gallop. Her serum sodium and potassium are normal, which makes kidney failure unlikely. She does not have any signs of infection, such as fever, chills, or purulent sputum. She does not have any history of exposure to high altitude as well. The chest x-ray of pulmonary oedema typically shows bilateral alveolar infiltrates with a bat-wing pattern, which is different from the interstitial pattern seen in this case.
- C. Idiopathic pulmonary fibrosis: This is a type of interstitial lung disease that has no known cause. It is characterized by progressive fibrosis of the lung parenchyma, leading to respiratory failure. This lady has a known cause of interstitial lung disease, which is systemic sclerosis. Therefore, the diagnosis of idiopathic pulmonary fibrosis is excluded by definition. The chest x-ray of idiopathic pulmonary fibrosis typically shows bilateral reticular opacities with honeycombing and traction bronchiectasis, which may not be seen in this case.
- D. Eosinophilic pneumonitis: This is a type of interstitial lung disease that is caused by an allergic reaction to inhaled antigens, such as dust, fungi, or parasites. It is characterized by eosinophilic infiltration of the lung tissue, leading to inflammation and fibrosis. This lady does not have any history of exposure to allergens, such as farming, bird keeping, or travel to endemic areas. She does not have any signs of allergy, such as rash, wheezing, or eosinophilia. The chest x-ray of eosinophilic pneumonitis typically shows bilateral patchy or diffuse opacities with a peripheral or upper lobe predominance, which is different from the interstitial pattern seen in this case.
- E. Bacterial pneumonia: This is a type of lung infection that is caused by bacteria, such as Streptococcus pneumoniae, Haemophilus influenzae, or Staphylococcus aureus. It is characterized by inflammation and consolidation of the lung alveoli, leading to impaired gas exchange. This lady does not have any signs of infection, such as fever, chills, or purulent sputum. Her total leucocyte count and platelet count are normal, which makes infection unlikely. The chest x-ray of bacterial pneumonia typically shows focal or lobar consolidation with air bronchograms, which is different from the interstitial pattern seen in this case.
_x000D_
_x000D_
_x000D_
_x000D_
-
Question 74 of 123
74. Question
A 44-year-old lady comes to the Pulmonologist Clinic with complaint of increasing shortness of breath. She has a history of Systemic Sclerosis, takes Omeprazole for Reflux Symptoms and Nifedipine Slow Release for Raynaud’s Disease.
_x000D_
On examination, her Blood Pressure is 150/80 mmHg, Pulse Rate is 82 Beats Per Minute and Regular. There is Peripheral Calcinosis on examination of the hands and pinching of the corners of the mouth consistent with Scleroderma. Auscultation of the chest reveals Scattered Inspiratory Crackles across Both Lung Fields.
_x000D_
Investigations Show:
_x000D_
_x000D_ _x000D_
_x000D_ _x000D_ Haemoglobin
_x000D_
_x000D_
_x000D_ 134 g/L
_x000D_
_x000D_
_x000D_ (115-160)
_x000D_
_x000D_
_x000D_
_x000D_ _x000D_ Total Leucocyte Count
_x000D_
_x000D_
_x000D_ 7.8 ×109/L
_x000D_
_x000D_
_x000D_ (4-11)
_x000D_
_x000D_
_x000D_
_x000D_ _x000D_ Platelet Count
_x000D_
_x000D_
_x000D_ 201 ×109/L
_x000D_
_x000D_
_x000D_ (150-400)
_x000D_
_x000D_
_x000D_
_x000D_ _x000D_ ESR
_x000D_
_x000D_
_x000D_ 65 mm/hr
_x000D_
_x000D_
_x000D_ (<10)
_x000D_
_x000D_
_x000D_
_x000D_ _x000D_ Serum Sodium
_x000D_
_x000D_
_x000D_ 140 mmol/L
_x000D_
_x000D_
_x000D_ (135-146)
_x000D_
_x000D_
_x000D_
_x000D_ _x000D_ Serum Potassium
_x000D_
_x000D_
_x000D_ 4.5 mmol/L
_x000D_
_x000D_
_x000D_ (3.5-5)
_x000D_
_x000D_
_x000D_
_x000D_ _x000D_ Serum Creatinine
_x000D_
_x000D_
_x000D_ 137 µmol/L
_x000D_
_x000D_
_x000D_ (79-118)
_x000D_
_x000D_
_x000D_
_x000D_
_x000D_
Chest X-Ray shows Bilateral Interstitial Infiltrates. Echocardiogram shows Ejection Fraction: 53%.
_x000D_
Which of the following is most likely the confirmative diagnosis?
CorrectIncorrectHint
Systemic Sclerosis is a Chronic Autoimmune Disease characterised by Increased Fibroblast Activity and Fibrosis in a number of different organ systems. 90-95% of patients have Positive Antinuclear Antibodies [ANA]. There are Two Major Subtypes:_x000D_
? Limited Cutaneous, and_x000D_
? Diffuse Cutaneous._x000D_
CREST Syndrome is an older term for the Limited Cutaneous Form (Calcinosis, Raynaud?s’ Phenomenon, Oeosophageal Dysmotility, Sclerodactyly, Telangiectasia). Patients with Systemic Sclerosis can present with Skin Abnormalities, Musculoskeletal Changes, Gastrointestinal Complications, Pulmonary Disease, Renal Crisis and Dry Eyes and Dry Mouth._x000D_
A number of Autoantibodies Against Extractable Nuclear Antigens can be detected in patients with Systemic Sclerosis. Anti-Centromere Antibodies and Anti-Topoisomerase I Antibodies are the Classic Autoantibodies associated with the disease. Anti-Centromere Antibodies are linked with limited cutaneous involvement and Isolated Pulmonary Hypertension, and a good prognosis, whereas Anti-Topoisomerase I and linked with Diffuse Skin Disease and Pulmonary Fibrosis and a Higher Mortality._x000D_
Respiratory Symptoms are common in patients with Systemic Sclerosis, but it is often difficult to distinguish between Interstitial Lung Disease [ILD] and Pulmonary Hypertension as the cause. Echocardiography, Pulmonary Hypertension and Chest Radiographs or CT Scans are often required._x000D_
Inspiratory Crackles heard here, coupled with Bilateral Interstitial Infiltrates on Chest Radiograph, make Interstitial Lung Disease the most likely diagnosis, but it is important to note they are not always present in the early stages of disease. The condition may be Steroid Responsive; as such a Trial Of Prednisolone plus or minus an appropriate Second Line Agent is indicated. Signs of Pulmonary Hypertension are Jugular Venous Distension, Right Ventricular Heave and an Accentuated Pulmonary Second Heart Sound._x000D_
Given the Chronic Nature of her disease Bacterial Infection is unlikely._x000D_
Idiopathic Pulmonary Fibrosis is by definition of unknown aetiology and this too therefore cannot be the correct answer._x000D_
Given the Normal Ejection Fraction Pulmonary Oedema is also unlikely._x000D_
The absence of Leukocytosis, and Normal Eosinophil Count makes Eosinophilic Pneumonitis unlikely. -
Question 75 of 123
75. Question
A 44-year-old gentleman, with disseminated malignancy and unknown primary, presents with oedema of the arms and face, with dilated neck veins.
_x000D_
Superior vena cava obstruction (SVCO) is suspected by the examining physician.
_x000D_
Which one of the following statements is correct?
CorrectIncorrectHint
The situation at hand is an emergency in the field of oncology.
_x000D_
The correct answer is B. Mediastinal radiotherapy relieves symptoms in 80% of patients. This is because radiotherapy is the traditional treatment of choice for patients with superior vena cava obstruction (SVCO), especially those with malignant aetiology. Radiotherapy can reduce the tumour mass and improve the venous flow. It can also be combined with chemotherapy and/or stent insertion depending on the histology, urgency of presentation, and previous treatment of the patient.
_x000D_
The other statements are incorrect for the following reasons:
_x000D_
- _x000D_
- A. Palliative treatment alone is not indicated, as there are effective treatments available for superior vena cava obstruction (SVCO) that can improve the quality of life and survival of the patient.
- C. Small cell lung cancer is not unlikely to be the cause, as it is one of the most common malignancies associated with superior vena cava obstruction (SVCO). Other common causes include lymphoma, metastatic disease, mesothelioma, and thymoma.
- D. Loss of pulsation in the venous system of the neck is of clinical use in the diagnosis of superior vena cava obstruction (SVCO), as it can help differentiate superior vena cava obstruction (SVCO) from other causes of neck swelling, such as heart failure or hypothyroidism. The loss of pulsation is due to the increased venous pressure and the reduced cardiac output caused by superior vena cava obstruction (SVCO).
- E. IV dexamethasone at high dose is of benefit in case of severe superior vena cava obstruction (SVCO), as it can reduce the oedema and inflammation caused by superior vena cava obstruction (SVCO) and improves the symptoms of the patient. It can also be used as a bridge therapy until definitive treatment is initiated.
_x000D_
_x000D_
_x000D_
_x000D_
_x000D_
The National Institute for Health and Care Excellence (NICE) guidelines (updated on 26th July, 2023) for superior vena cava obstruction (SVCO) treatment are as follows:
_x000D_
- _x000D_
- Stent placement for vena caval obstruction is a minimally invasive procedure that can relieve the obstruction and improve the symptoms of the patient. It can replace or supplement radiotherapy and chemotherapy depending on the case.
- The procedure is safe and effective, with few adverse events reported. It can provide immediate or rapid relief of obstruction, compared with other treatments that may take several weeks to show response.
- The procedure is indicated for patients with malignant vena caval obstruction, especially those with lung cancer. It is not recommended for patients with a mediastinal mass on chest x-ray that may disappear quickly with chemotherapy.
_x000D_
_x000D_
_x000D_
-
Question 76 of 123
76. Question
A 44-year-old gentleman with a history of nasal congestion, breathlessness, cough and wheeze presents with a left foot drop.
_x000D_
Which one of the following is the most likely diagnosis?
CorrectIncorrectHint
Eosinophilic Granulomatosis with Polyangiitis (EGPA) is a rare form of small-vessel vasculitis, characterised by asthma, allergic rhinitis and prominent peripheral blood eosinophilia. Rarely, it can cause either an anterior or a posterior ischaemic optic neuropathy, which presents with visual loss._x000D_
The most commonly involved organ is the lung, followed by the skin. EGPA, however, can affect any organ system, including the cardiovascular, gastrointestinal, renal, and central nervous systems. The unifying feature of patients presenting with EGPA is asthma. Vasculitis involving the peripheral nervous system is also a characteristic feature, and mononeuritis multiplex occurs in 75% of patients, which accounts for the foot drop in this case._x000D_
Vasculitis of Extrapulmonary Organs is largely responsible for the morbidity and mortality associated with EGPA. 40-60% are associated with positive ANCA, usually pANCA/MPO._x000D_
Intravenous Glucocorticoid is used for initial therapy of acute multi-organ disease, followed by oral Glucocorticoid Therapy, often with Azathioprine as a Steroid-Sparing Agent. Loss of vision must be treated aggressively._x000D_
Granulomatosis with Polyangiitis is a multi-organ autoimmune disease, which can be fatal. The classical triad consists of necrotising granulomatous inflammation of the respiratory tract, glomerulonephritis and a small-vessel vasculitis. A prolonged history of epistaxis or sinusitis is commonly found in granulomatosis with polyangiitis, which in some patients is also associated with an eosinophilia._x000D_
Hypereosinophilic syndrome is characterised by a peripheral blood eosinophil count of >1.5 for more than 6 months. Generalised symptoms are fatigue, myalgia, fever, night sweats, diarrhoea and pruritus. Other symptoms depend on the organ involved: cardiac disease causes chest pain and dyspnoea, respiratory disease presents with a dry cough._x000D_
Diabetes mellitus can cause a mononeuritis multiplex (and therefore a foot drop), however the other features do not fit with this diagnosis._x000D_
Polyarteritis nodosa is a vasculitis which affects medium and small arteries, resulting in microaneurysms, aneurysmal rupture, thrombosis and subsequently organ ischaemia and infarction. It most commonly affects skin, joints, peripheral nerves, the gastrointestinal tract and the kidney. The lungs are usually spared. A typical presentation is with fever, night sweats, weight loss, skin ulceration and tender nodules, and severe muscle and joint pains. -
Question 77 of 123
77. Question
A 42-year-old gentleman is admitted to the Emergency Department after being involved in a house fire. He is extremely drowsy but the attending doctor notices on examining him that he seems well perfused, with his cheeks looking almost pink.
_x000D_
His blood pressure is 100/60 mmHg and his pulse rate is 95 beats per minute and regular. Blood gas analysis reveals a CO level of 12% and a metabolic acidosis with a pH of 7.15.
_x000D_
Which one of the following is the most appropriate next intervention?
CorrectIncorrectHint
The correct answer to the above question is B. 100% oxygen by mask. This is because:
_x000D_
- _x000D_
- 100% oxygen by mask is the standard treatment for mild to moderate cases of carbon monoxide poisoning. It involves breathing pure oxygen through a face mask that covers the nose and mouth. This reduces the half-life of carbon monoxide (CO) in the blood from about 5 hours to about 1 hour.
- The gentleman in the scenario has a carbon monoxide level of 12%, which indicates moderate poisoning, and a pH of 7.15, which indicates metabolic acidosis. He also has signs of impaired consciousness and possible brain damage, such as being extremely drowsy and having pink cheeks. These symptoms can be improved by 100% oxygen by mask, which helps to remove the carbon monoxide (CO) from the blood and restore oxygen delivery to the body.
- The prognosis of the patient depends on the duration of exposure and the extent of poisoning. The key to recovery is to remove the patient from the source of carbon monoxide (CO) as quickly as possible, and start 100% oxygen by mask treatment. At a level of 12%, long term psychological disturbance or memory loss is possible, but not inevitable.
_x000D_
_x000D_
_x000D_
_x000D_
The other options are incorrect for the following reasons:
_x000D_
- _x000D_
- A. Hyperbaric oxygen is a more intensive treatment for severe cases of carbon monoxide poisoning, especially if there are signs of brain or heart damage. It involves breathing pure oxygen in a pressurized chamber that increases the amount of oxygen dissolved in the blood. This reduces the half-life of carbon monoxide (CO) in the blood from about 1 hour to about 20 minutes. It also helps to prevent or reverse the effects of carbon monoxide (CO) on the brain and other organs. However, hyperbaric oxygen therapy is not widely available, and its effectiveness is controversial. Some countries, such as the United States, recommend it for certain cases of carbon monoxide poisoning, but others, such as the United Kingdom, do not, as there appears to be no significant improvement in outcome versus high flow oxygen alone. Therefore, hyperbaric oxygen therapy is not the most appropriate next intervention for the patient in the scenario, unless there is evidence of severe poisoning or organ damage that does not respond to 100% oxygen by mask.
- C. Nebulised salbutamol is a medication that is inhaled through a nebulizer, a device that turns liquid medicine into a fine mist. It is used to treat asthma and other respiratory conditions by relaxing the muscles in the airways and making breathing easier. It has no role in the treatment of carbon monoxide poisoning and may even worsen the condition by increasing the oxygen demand of the body. Therefore, nebulised salbutamol is not indicated for the gentleman in this scenario, and may be harmful.
- D. IV sodium bicarbonate is a solution that is given intravenously, through a needle or a tube inserted into a vein. It is used to treat metabolic acidosis, a condition where the blood becomes too acidic due to various causes. It works by neutralizing the excess acid in the blood and restoring the normal pH balance. It may be used in some cases of carbon monoxide poisoning, but only as an adjunct to oxygen therapy, not as a substitute. It is not effective in reversing the effects of carbon monoxide (CO) on the brain and other organs. Therefore, IV sodium bicarbonate is not the most appropriate next intervention for the patient in the scenario, unless the metabolic acidosis is severe and does not improve with oxygen therapy.
- E. IV mannitol is a solution that is given intravenously, through a needle or a tube inserted into a vein. It is used to treat cerebral oedema, a condition where the brain swells due to excess fluid accumulation. It works by drawing water out of the brain tissue and reducing the pressure inside the skull. It may be used in some cases of carbon monoxide poisoning, but only as an adjunct to oxygen therapy, not as a substitute. It is not effective in removing carbon monoxide from the blood or preventing further damage to the brain and other organs. Therefore, IV mannitol is not the most appropriate next intervention for the patient in the scenario, unless there is suspicion of cerebral oedema that does not respond to oxygen therapy.
_x000D_
_x000D_
_x000D_
_x000D_
-
Question 78 of 123
78. Question
A 48-year-old lady presents to her general physician with Cushingoid facies and hyperpigmentation of the skin on her face and chest.
_x000D_
She has smoked 20 cigarettes per day for 30 years.
_x000D_
Examination reveals no gross abnormalities. Her chest x-ray reveals a 2 cm irregularly shaped mass in the right upper lobe, in proximity to the mediastinum.
_x000D_
A computed tomography (CT) guided needle biopsy of the lung lesion is performed.
_x000D_
Which one of the following would be the most likely cytological finding?
CorrectIncorrectHint
The correct answer is B. Small cell (oat cell) carcinoma. This is because this lady has clinical features that are suggestive of ectopic adrenocorticotropic hormone (ACTH) syndrome, which is a paraneoplastic syndrome, caused by the production of adrenocorticotropic hormone (ACTH) by a non-pituitary tumour. Ectopic adrenocorticotropic hormone (ACTH) syndrome is most commonly associated with small cell lung cancer, which accounts for about 50-80% of cases. Small cell lung cancer is a highly aggressive neuroendocrine tumour that often presents with mediastinal involvement and distant metastases.
_x000D_
Small cell lung cancer can be diagnosed by cytology, which typically shows small, round to oval cells with scant cytoplasm, finely granular nuclear chromatin, inconspicuous nucleoli, and frequent mitoses. The cells may form rosettes or clusters, and may show crush artifact or molding. The tumour cells are positive for neuroendocrine markers such as synaptophysin, chromogranin, and CD56, and may also express TTF1.
_x000D_
The other answer options are incorrect for the following reasons:
_x000D_
- _x000D_
- A. Benign bronchial adenoma: Benign bronchial adenoma is a rare and misleading term for a group of low-grade malignant tumours that arise from the bronchial mucosa or glands. They include carcinoid tumours, muco-epidermoid carcinomas, and adenoid cystic carcinomas. These tumours are usually slow-growing and rarely cause paraneoplastic syndromes. Cytology may show epithelial or neuroendocrine cells with variable morphology and atypia.
- C. Adenocarcinoma: Adenocarcinoma is the most common type of lung cancer, accounting for about 40% of cases. It is more common in non-smokers, women, and younger patients. It usually arises from the peripheral lung parenchyma and may show various histologic patterns, such as acinar, papillary, micropapillary, lepidic, or solid. Cytology may show glandular or pneumocyte differentiation, mucin production, or TTF1 expression.
- D. Squamous cell carcinoma: Squamous cell carcinoma is the second most common type of lung cancer, accounting for about 30% of cases. It is strongly associated with smoking and usually arises from the central bronchi. It may cause obstruction, atelectasis, or post obstructive pneumonia. Cytology may show keratinization, intercellular bridges, or squamous pearls.
- E. Bronchoalveolar cell carcinoma (BAC): Bronchoalveolar cell carcinoma (BAC) is an outdated term for a subtype of lung adenocarcinoma that shows lepidic growth along the alveolar walls, without invasion of the stroma or vessels. It is now classified as adenocarcinoma in situ or minimally invasive adenocarcinoma, depending on the presence or absence of invasion. Bronchoalveolar cell carcinoma (BAC) is more common in non-smokers, women, and younger patients. It may present as a solitary nodule or as multifocal or diffuse disease. Cytology may show columnar or cuboidal cells with abundant cytoplasm and TTF1 expression.
_x000D_
_x000D_
_x000D_
_x000D_
-
Question 79 of 123
79. Question
A 48-year-old lady presents with acute chest pain and dyspnoea.
_x000D_
Examination reveals Bilateral Ankle Oedema with 24-Hour Urine Protein Assessment showing 6 g/d (<0.2).
_x000D_
Which is the most appropriate explanation for these findings?
CorrectIncorrectHint
This patient has developed Nephrotic Syndrome and appears to have had a Thromboembolic Event._x000D_
The suggestion is that she has had Nephrotic Syndrome before developing the Thromboembolism._x000D_
Deranged Coagulation associated with Nephrotic Syndrome is a consequence of AT III Deficiency, Increased Fibrinogen and Increased Factor VIIIc. -
Question 80 of 123
80. Question
A 48-year-old lady presents with acute chest pain and dyspnoea.
_x000D_
Examination reveals bilateral ankle oedema with 24-hour urine protein assessment showing 6 g/d (<0.2).
_x000D_
Which one of the following is the most appropriate explanation for these findings?
CorrectIncorrectHint
The most likely explanation for these findings is D. Reduced antithrombin III levels. This lady has nephrotic syndrome, which is a condition characterized by excessive proteinuria, hypoalbuminaemia, and oedema. Nephrotic syndrome causes a hypercoagulable state, which increases the risk of thromboembolic events, such as pulmonary embolism, which may present as acute chest pain and dyspnoea. The hypercoagulability in nephrotic syndrome is mainly due to urinary loss of antithrombin III, a natural anticoagulant that inhibits thrombin and other coagulation factors. In addition, nephrotic syndrome stimulates hepatic synthesis of fibrinogen and factor VIIIc, which are procoagulant factors that promote clot formation.
_x000D_
The other options are not consistent with the findings in this lady, because they do not cause a hypercoagulable state or are not related to nephrotic syndrome. For example:
_x000D_
- _x000D_
- Reduced concentration of von Willebrand’s factor (vWF) would impair the primary haemostasis and increase the bleeding tendency, not the thrombotic tendency. Low levels of von Willebrand factor (vWF) can be caused by heart failure, which may also cause chest pain and dyspnoea, but not proteinuria or oedema.
- Reduced fibrinogen concentration would impair the secondary haemostasis and increase the bleeding tendency, not the thrombotic tendency. Low levels of fibrinogen can be caused by liver disease, disseminated intravascular coagulation, or inherited disorders, but not by nephrotic syndrome.
_x000D_
_x000D_
_x000D_
Factor V Leiden is a genetic mutation that causes resistance to activated protein C, a natural anticoagulant that inhibits factor V and factor VIII. Factor V Leiden increases the risk of venous thrombosis, especially in the lower extremities, but not of arterial thrombosis, such as pulmonary embolism. Factor V Leiden is not related to nephrotic syndrome or proteinuria
-
Question 81 of 123
81. Question
A 48-year-old lady is referred to the Respiratory Clinic with shortness of breath. She has been unable to work due to a long term back injury and is therefore on long term sickness benefit.
_x000D_
On examination her Blood Pressure is 160/88 mmHg, Pulse Rate is 85/Minute and Regular. Her BMI is 32, there is Bilateral Lower Limb Pitting Oedema with Varicose Vein Formation. Heart sounds are normal, auscultation of the chest is clear.
_x000D_
Investigations Show:
_x000D_
_x000D_ _x000D_
_x000D_ _x000D_ pH
_x000D_
_x000D_
_x000D_ 7.44
_x000D_
_x000D_
_x000D_ (7.35 – 7.45)
_x000D_
_x000D_
_x000D_
_x000D_ _x000D_ PCO2
_x000D_
_x000D_
_x000D_ 5.6 kPa
_x000D_
_x000D_
_x000D_ (4.8 – 6.1)
_x000D_
_x000D_
_x000D_
_x000D_ _x000D_ pO2
_x000D_
_x000D_
_x000D_ 9.7 kPa
_x000D_
_x000D_
_x000D_ (10 – 13.3)
_x000D_
_x000D_
_x000D_
_x000D_
_x000D_
Which of the following is likely to be the most effective therapy?
CorrectIncorrectHint
Given this lady’s obesity, slowly progressive shortness of breath, absence of chest signs, and presence of peripheral oedema and varicose veins, chronic pulmonary emboli are the most likely possibility. As such warfarinisation is the optimal long term strategy to reduce the risk of further clots. She should also be encouraged to lose a significant amount of weight._x000D_
Salbutamol and Seretide are treatments for Asthma and Chronic Obstructive Pulmonary Disease (COPD), and Tiotropium is a treatment for Chronic Obstructive Pulmonary Disease (COPD) alone, therefore none of the three are appropriate here._x000D_
We are given no history of wheeze or cough which fits with either COPD or Asthma as the underlying diagnosis._x000D_
Whilst Furosemide may relieve Peripheral Oedema, it will not have a positive effect on the underlying cause of Right Heart Failure. -
Question 82 of 123
82. Question
A 48-year-old gentleman is seen in the Emergency Department complaining of cough and dyspnoea.
_x000D_
On examination, he is disorientated and febrile at 38.5°C. He has a Pulse Rate of 100/Minute and his Blood Pressure is 90/60 mmHg. He has Oxygen Saturations of 89% on Room Air and has a Respiratory Rate of 36/Minute.
_x000D_
Chest X-Ray shows Left Basal Consolidation.
_x000D_
Investigation Results Show:
_x000D_
_x000D_ _x000D_
_x000D_ _x000D_ Serum Sodium
_x000D_
_x000D_
_x000D_ 143 mmol/L
_x000D_
_x000D_
_x000D_ (137-144)
_x000D_
_x000D_
_x000D_
_x000D_ _x000D_ Serum Potassium
_x000D_
_x000D_
_x000D_ 3.9 mmol/L
_x000D_
_x000D_
_x000D_ (3.5-4.9)
_x000D_
_x000D_
_x000D_
_x000D_ _x000D_ Serum Urea
_x000D_
_x000D_
_x000D_ 9.8 mmol/L
_x000D_
_x000D_
_x000D_ (2.5-7.5)
_x000D_
_x000D_
_x000D_
_x000D_ _x000D_ Serum Creatinine
_x000D_
_x000D_
_x000D_ 98 µmol/L
_x000D_
_x000D_
_x000D_ (60-110)
_x000D_
_x000D_
_x000D_
_x000D_
_x000D_
Which of the following is not part of the CURB Score?
CorrectIncorrectHint
The CURB Score is calculated by assessment of core adverse prognostic features which are used in assessment of severity of pneumonia._x000D_
Two from four features indicate a severe pneumonia and hospital admission is advised._x000D_
The CURB Score is calculated using:_x000D_
? Confusion Abbreviated Mental Test Score 7 mmol/L_x000D_
? Respiratory Rate >30/min_x000D_
? Blood Pressure: Systolic Blood Pressure <90 mmHg or Diastolic Blood Pressure <60 mmHg. -
Question 83 of 123
83. Question
A 48-year-old Seaman presents to his Primary Care Physician with cough and fever.
_x000D_
A Chest X-Ray demonstrates a Cavitating Lung Lesion.
_x000D_
Which of the following is the most likely cause?
CorrectIncorrectHint
Histoplasmosis normally evolves slowly over as long as 20 years but may follow a more rapid course in the Immunocompromised (Seamen may be more prone to Sexually Transmitted Diseases such as Human Immunodeficiency Virus [HIV])._x000D_
Amoebic Abscesses can develop in the Right Lower Lobe following Transdiaphragmatic Spread from Amoebic Liver Abscess (Tender Hepatomegaly, Malaise, Spiking Temperature)._x000D_
Amoebiasis is also a Fresh Water Pathogen. -
Question 84 of 123
84. Question
A 47-year-old gentleman, who works as a manufacturer of plastics, presents with a five month history of wheezing and dyspnoea. His symptoms improve significantly when at home and at weekends.
_x000D_
What is the likely causative agent?
CorrectIncorrectHint
This patient presents with typical symptoms of Occupational Asthma and the most likely causative substance is Isocyanates which is used in the manufacture of Foams / Plastics._x000D_
There are an estimated 1500 to 3000 cases of Occupational Asthma reported each year._x000D_
The most commonly implicated substances in Occupational Asthma are Isocyanates, Flour / Grain, Adhesives, Metals, Resins, Colophony, Fluxes, Latex, Animals, Aldehydes and Wood Dust. Cotton Dust can be associated, but is less recognised than Isocyanates._x000D_
Asbestos is associated with Pleural Plaques, Pleural Thickening, Pleural Effusions, Interstitial Lung Disease, Mesothelioma and Lung Carcinoma but not Occupational Asthma._x000D_
Silica (which is found in coal dust) exposure results in Pulmonary Fibrosis._x000D_
Simple Coal Worker’s Lung Disease describes a Nodular Interstitial Lung Disease. -
Question 85 of 123
85. Question
A 47-year-old gentleman presents with a couple of weeks history of facial swelling and breathlessness.
_x000D_
His chest x-ray reveals marked mediastinal lymphadenopathy, and a mass in the right lower lobe. Subsequent computed tomography (CT) scan of chest confirms these findings, and diagnoses superior vena cava obstruction (SVCO).
_x000D_
Which one of the following statements is the most accurate one regarding superior vena cava obstruction (SVCO)?
CorrectIncorrectHint
The most accurate statement regarding superior vena cava obstruction (SVCO) is E. It may be associated with voice hoarseness. This is because voice hoarseness can result from laryngeal oedema or compression of the recurrent laryngeal nerve by the enlarged mediastinal lymph nodes or the lung mass. Voice hoarseness is one of the possible clinical features of superior vena cava obstruction (SVCO), along with facial and neck swelling, facial flushing, bilateral upper extremity swelling, neurological signs, dyspnoea, headache, and cough.
_x000D_
The other statements are inaccurate for the following reasons:
_x000D_
- _x000D_
- A. Kussmaul’s sign is an increase in jugular venous pressure during inspiration, which is seen in patients with constrictive pericarditis or right-sided heart failure. It is not associated with superior vena cava obstruction (SVCO), as superior vena cava obstruction (SVCO) causes a non-pulsatile distension of the neck and chest veins that does not vary with respiration.
- B. The most common cause of superior vena cava obstruction (SVCO) is malignancy, but not specifically small cell carcinoma of the lung. Lung cancer is the most common malignancy associated with superior vena cava obstruction (SVCO), affecting 2-4% of lung cancer patients, but other types of lung cancer, such as non-small cell lung cancer, can also cause superior vena cava obstruction (SVCO). Other common malignancies that can cause superior vena cava obstruction (SVCO) include lymphoma, metastatic disease, breast cancer, colon cancer, and oesophageal cancer.
- C. The commonest symptom of superior vena cava obstruction (SVCO) is not stridor, but breathlessness. Stridor is a high-pitched sound that occurs due to upper airway obstruction, which can be caused by laryngeal oedema or compression in superior vena cava obstruction (SVCO). However, it is not a frequent or specific symptom of superior vena cava obstruction (SVCO), and it may indicate a life-threatening situation that requires immediate intervention. Breathlessness is a more common and less severe symptom of superior vena cava obstruction (SVCO), which occurs due to reduced venous return to the heart and impaired gas exchange in the lungs.
- D. The treatment of choice for superior vena cava obstruction (SVCO) is not always radiotherapy, as it depends on the underlying cause, the severity of symptoms, and the patient’s condition. Radiotherapy is the traditional treatment for patients with superior vena cava obstruction (SVCO) due to malignant causes; especially lung cancer and lymphoma as it can reduce the tumour mass and improve the venous flow. However, radiotherapy may not be effective for all types of malignancies, and it may take several weeks to show a response. Other treatments that may be used for superior vena cava obstruction (SVCO) include chemotherapy, stent placement, anticoagulation, corticosteroids, and supportive care.
_x000D_
_x000D_
_x000D_
_x000D_
-
Question 86 of 123
86. Question
A 47-year-old gentleman presents with a couple of weeks history of facial swelling and breathlessness.
_x000D_
His chest X-Ray reveals marked mediastinal Lymphadenopathy, and a Mass in the left Lower Lobe. Subsequent CT Scan Chest confirms these findings, and diagnoses Superior Vena Cava Obstruction (SVCO).
_x000D_
Which of the following statements is the most accurate one regarding Superior Vena Cava Obstruction [SVCO]?
CorrectIncorrectHint
Superior Vena Cava Obstruction [SVCO] is most commonly caused by Mediastinal Lymphadenopathy, typically due to Lung Carcinoma (of which Non-Small Cell is more common than Small Cell). It can also occur Secondary to Lymphoma and Germ Cell Tumours. Non-Malignant Causes include Aortic Aneurysm, Mediastinal Fibrosis and a Large Goitre._x000D_
The Lymphadenopathy can result in a Recurrent Laryngeal Nerve Palsy, which presents as a Hoarse Voice. There may also be Horner’s Syndrome due to involvement of Sympathetic Chain. _x000D_
Superior Vena Cava Obstruction [SVCO] itself typically presents with Insidious Onset Shortness Of Breath, which is Worse on Lying Flat. The patients may also have cough and chest pain, due to the distortion of Mediastinal Anatomy. Physical signs are often absent or minimal, but classically these are Facial and Periorbital Oedema, Chemosis and Distended Chest Wall Veins. It is associated with an Elevated Non-Pulsatile Jugular Venous Pressure (JVP). Kussmaul’s Sign is the paradoxical rise in Jugular Venous Pressure (JVP) on Inspiration due to Constrictive Pericarditis or Significant Pericardial Effusion._x000D_
Treatment depends on the underlying cause. If malignant, it also depends on the underlying chemosensitivity of the primary tumour and how unwell the patient is. Steroids (typically dexamethasone) can be given to reduce symptoms whilst specific treatments are commenced. Lymphoma and small cell carcinoma are relatively chemo-sensitive and therefore SVCO can be treated with chemotherapy alone in these settings, but radiotherapy is usually required if the underlying cause is non-small cell lung carcinoma (depending on the extent of disease and the symptoms). -
Question 87 of 123
87. Question
A 46-year-old lady re-attends the chest clinic following investigations that have confirmed a diagnosis of sarcoidosis.
_x000D_
Which one of the following would be a poor prognostic sign?
CorrectIncorrectHint
The poor prognostic sign of sarcoidosis is A. Lupus pernio.
_x000D_
Sarcoidosis is a chronic inflammatory disorder that causes granulomas, or lumps of immune cells, to form in various organs, especially the lungs and lymph nodes. The cause of sarcoidosis is unknown, but it may be triggered by infections or environmental factors in genetically susceptible people.
_x000D_
The prognosis of sarcoidosis varies depending on the extent and severity of organ involvement, the response to treatment, and the presence of complications. Some patients may have spontaneous remission, while others may have chronic or progressive disease.
_x000D_
Lupus pernio is a type of skin lesion that occurs in sarcoidosis, characterized by purple or red bumps or plaques on the nose, cheeks, ears, or fingers. Lupus pernio is a poor prognostic sign, as it indicates chronic and severe sarcoidosis, often associated with lung fibrosis, eye involvement, and bone lesions.
_x000D_
The other options are not poor prognostic signs of sarcoidosis for the following reasons:
_x000D_
- _x000D_
- Markedly elevated serum angiotensin-converting enzyme (ACE) level is a marker of sarcoidosis activity, but it is not specific or reliable for predicting the course or outcome of the disease. Serum angiotensin-converting enzyme (ACE) level may vary depending on the assay method, the organ involvement, the treatment, and other factors.
- Erythema nodosum is another type of skin lesion that occurs in sarcoidosis, characterized by tender red nodules on the lower legs. Erythema nodosum is a good prognostic sign, as it indicates acute and self-limited sarcoidosis, often associated with hilar lymphadenopathy and a favorable response to treatment.
- Current smoker is a risk factor for many lung diseases, but not for sarcoidosis. In fact, smoking may have a protective effect against sarcoidosis, possibly by suppressing the immune system or altering the lung microbiome. However, smoking is not recommended for sarcoidosis patients, as it may worsen the lung function and increase the risk of infections and malignancies.
- Caucasian race is not a poor prognostic sign for sarcoidosis, although the incidence and presentation of the disease may vary by race and ethnicity. Sarcoidosis is more common and more severe in African-Americans than in Caucasians, and it may affect different organs and respond differently to treatment.
_x000D_
_x000D_
_x000D_
_x000D_
-
Question 88 of 123
88. Question
A 46-year-old lady re-attends the Chest Clinic following investigations that have confirmed a diagnosis of Sarcoidosis.
_x000D_
Which of the following would be a poor prognostic sign?
CorrectIncorrectHint
The American Thoracic Society has produced a statement on the prognostic indicators associated with Sarcoidosis._x000D_
Lupus Pernio is a Chronic Raised Indurated (Hardened) Lesion of the Skin, often purplish in colour, and is associated with Sarcoid. It is noted to be an adverse prognostic factor._x000D_
Other Adverse Prognostic Factors Include;_x000D_
? Age of onset >40 years._x000D_
? Afro-Caribbean or Afro-American Race._x000D_
? Cardiac Involvement._x000D_
? Chronic Hypercalcaemia._x000D_
? Nasal mucosal Involvement._x000D_
? Neurosarcoidosis._x000D_
? Progressive Pulmonary Sarcoidosis. -
Question 89 of 123
89. Question
A 45-year-old single lady presented to her primary care physician with complaint of increasing shortness of breath.
_x000D_
She has been previously noted to have a raised serum alanine aminotransferase (ALT), which was put down by the primary care physician to a problem with excessive drinking. She was given a salbutamol inhaler which brought about some relief, particularly when she was playing beach volleyball with her friends, but most recently she has had to give up due to reduced exercise tolerance.
_x000D_
On examination, her blood pressure is 145/85 mmHg. She is thin with a body mass index (BMI) of 19 kg/m2, and her chest looks hyper-expanded. There are scattered wheezes throughout both lung fields. Abdominal examination is normal.
_x000D_
Investigations Show:
_x000D_
_x000D_ _x000D_
_x000D_ _x000D_ Haemoglobin
_x000D_
_x000D_
_x000D_ 126 g/L
_x000D_
_x000D_
_x000D_ (135-180)
_x000D_
_x000D_
_x000D_
_x000D_ _x000D_ White Blood Cell Count
_x000D_
_x000D_
_x000D_ 8.6 ×109/L
_x000D_
_x000D_
_x000D_ (4-10)
_x000D_
_x000D_
_x000D_
_x000D_ _x000D_ Platelet Count
_x000D_
_x000D_
_x000D_ 215 ×109/L
_x000D_
_x000D_
_x000D_ (150-400)
_x000D_
_x000D_
_x000D_
_x000D_ _x000D_ Serum Sodium
_x000D_
_x000D_
_x000D_ 142 mmol/L
_x000D_
_x000D_
_x000D_ (134-143)
_x000D_
_x000D_
_x000D_
_x000D_ _x000D_ Serum Potassium
_x000D_
_x000D_
_x000D_ 4.2 mmol/L
_x000D_
_x000D_
_x000D_ (3.5-5)
_x000D_
_x000D_
_x000D_
_x000D_ _x000D_ Serum Creatinine
_x000D_
_x000D_
_x000D_ 115 μmol/L
_x000D_
_x000D_
_x000D_ (60-120)
_x000D_
_x000D_
_x000D_
_x000D_ _x000D_ FEV1
_x000D_
_x000D_
_x000D_ 70% of predicted
_x000D_
_x000D_
_x000D_
_x000D_ _x000D_ FEV1/FVC
_x000D_
_x000D_
_x000D_ 60% of predicted
_x000D_
_x000D_
_x000D_
_x000D_
_x000D_
Which one of the following is the most likely diagnosis?
CorrectIncorrectHint
The most likely diagnosis for this lady with shortness of breath, raised serum alanine aminotransaminase (ALT), and wheezes is E. Alpha-1 antitrypsin deficiency.
_x000D_
Alpha-1 antitrypsin deficiency (AATD) is a genetic disorder that causes low levels of alpha-1 antitrypsin (AAT), a protein that protects the lungs and liver from damage. Alpha-1 antitrypsin deficiency (AATD) can cause early-onset emphysema, chronic bronchitis, and liver disease.
_x000D_
The symptoms and signs of this lady are consistent with alpha-1 antitrypsin deficiency (AATD). She has shortness of breath, wheezes, reduced exercise tolerance, and hyperinflated chest, which suggest emphysema. She also has raised serum alanine aminotransaminase (ALT), which indicates liver injury. She is thin and has a low body mass index (BMI), which may reflect malnutrition or muscle wasting due to chronic lung disease. She has a normal abdominal examination, which excludes ascites or hepatomegaly that may occur in advanced liver disease.
_x000D_
The investigations in this lady also support the diagnosis of alpha-1 antitrypsin deficiency (AATD). She has a normal white blood cell count, platelet count, serum sodium, serum potassium, and serum creatinine, which exclude infection, bleeding, dehydration, or renal impairment. She has mildly low haemoglobin, which may be due to anaemia of chronic disease or blood loss. She has a reduced FEV1 and FEV1/FVC ratio, which indicate airflow obstruction and emphysema.
_x000D_
The other options are less likely for the following reasons:
_x000D_
- _x000D_
- Asthma is a chronic inflammatory disorder of the airways that causes reversible bronchoconstriction, mucus production, and cough. Asthma usually presents with episodic wheezing, dyspnoea, and chest tightness, triggered by allergens, infections, exercise, or cold air. Asthma does not cause raised serum alanine aminotransaminase (ALT) or emphysema.
- Primary biliary cirrhosis (PBC) is an autoimmune disorder that causes progressive destruction of the bile ducts in the liver, leading to cholestasis, fibrosis, and cirrhosis. Primary biliary cirrhosis (PBC) usually presents with fatigue, pruritus, jaundice, and xanthomas. Primary biliary cirrhosis (PBC) does not cause wheezing or emphysema.
- Idiopathic pulmonary fibrosis (IPF) is a chronic interstitial lung disease that causes progressive scarring and stiffening of the lung tissue, leading to restrictive lung disease and respiratory failure. Idiopathic pulmonary fibrosis (IPF) usually presents with dyspnoea, dry cough, and clubbing. Idiopathic pulmonary fibrosis (IPF) does not cause raised serum alanine aminotransaminase (ALT) or wheezing.
- Chronic bronchitis is a type of chronic obstructive pulmonary disease (COPD) that is characterized by chronic productive cough and mucus hypersecretion in the airways. Chronic bronchitis is usually caused by smoking or exposure to air pollutants. Chronic bronchitis may cause wheezing, dyspnoea, and hyperinflation, but it does not cause raised serum alanine aminotransaminase (ALT) or emphysema.
_x000D_
_x000D_
_x000D_
_x000D_
-
Question 90 of 123
90. Question
A 45-year-old single lady presented to her Primary Care Physician with complaint of increasing shortness of breath.
_x000D_
She has been previously noted to have a Raised Serum Alanine Aminotransferase [ALT], which was put down by the Primary Care Physician to a problem with excessive drinking. She was given a Salbutamol Inhaler which brought about some relief, particularly when she was playing beach volleyball with her friends, but most recently she has had to give up due to reduced exercise tolerance.
_x000D_
On examination her Blood Pressure is 145/85 mmHg. She is thin with a BMI of 19 kg/m2, and her chest looks hyper-expanded. There is scattered wheeze throughout both lung fields. Abdominal examination is normal.
_x000D_
Investigations Show:
_x000D_
_x000D_ _x000D_
_x000D_ _x000D_ Haemoglobin
_x000D_
_x000D_
_x000D_ 126 g/L
_x000D_
_x000D_
_x000D_ (135-180)
_x000D_
_x000D_
_x000D_
_x000D_ _x000D_ White Blood Cell Count
_x000D_
_x000D_
_x000D_ 8.6 ×109/L
_x000D_
_x000D_
_x000D_ (4-10)
_x000D_
_x000D_
_x000D_
_x000D_ _x000D_ Platelet Count
_x000D_
_x000D_
_x000D_ 215 ×109/L
_x000D_
_x000D_
_x000D_ (150-400)
_x000D_
_x000D_
_x000D_
_x000D_ _x000D_ Serum Sodium
_x000D_
_x000D_
_x000D_ 142 mmol/L
_x000D_
_x000D_
_x000D_ (134-143)
_x000D_
_x000D_
_x000D_
_x000D_ _x000D_ Serum Potassium
_x000D_
_x000D_
_x000D_ 4.2 mmol/L
_x000D_
_x000D_
_x000D_ (3.5-5)
_x000D_
_x000D_
_x000D_
_x000D_ _x000D_ Serum Creatinine
_x000D_
_x000D_
_x000D_ 115 μmol/L
_x000D_
_x000D_
_x000D_ (60-120)
_x000D_
_x000D_
_x000D_
_x000D_ _x000D_ FEV1
_x000D_
_x000D_
_x000D_ 70% Of Predicted
_x000D_
_x000D_
_x000D_
_x000D_ _x000D_ FEV1/FVC
_x000D_
_x000D_
_x000D_ 60% Of Predicted
_x000D_
_x000D_
_x000D_
_x000D_
_x000D_
Which of the following is the most likely diagnosis?
CorrectIncorrectHint
This lady’s abnormal Alanine Aminotransferase (ALT) raises the possibility of Accelerated Liver Disease, characterised by Accelerated Hepatic Fibrosis / Cirrhosis as a result of Alpha-1 Antitrypsin Deficiency._x000D_
Additionally, her Lung Function Tests fit the diagnosis of Chronic Obstructive Pulmonary Disease (COPD). As such, Emphysema is the most likely underlying respiratory diagnosis. It is likely that she has the PiMS, PiMZ or PiSS Genotype._x000D_
Replacement Alpha-1 Antitrypsin is available, although it?s cost effectiveness and long term clinical effectiveness is yet to be established. -
Question 91 of 123
91. Question
A 45-year-old lady presents to the respiratory clinic with increasing shortness of breath and a dry cough. She also has fever and night sweats which have worsened over the past seven to nine months and has lost a few kilograms in weight.
_x000D_
On examination, her body mass index (BMI) is 23 Kg/m2, blood pressure is 140/75 mmHg, pulse rate is 73 beats per minute, and there are scattered crackles on auscultation of the chest. She also has erythema nodosum on examination of both lower limbs.
_x000D_
Investigations Show:
_x000D_
_x000D_ _x000D_
_x000D_ _x000D_ Haemoglobin
_x000D_
_x000D_
_x000D_ 124 g/L
_x000D_
_x000D_
_x000D_ (115-160)
_x000D_
_x000D_
_x000D_
_x000D_ _x000D_ White Blood Cell count
_x000D_
_x000D_
_x000D_ 9.3 ×109/L
_x000D_
_x000D_
_x000D_ (4-11)
_x000D_
_x000D_
_x000D_
_x000D_ _x000D_ Platelet Count
_x000D_
_x000D_
_x000D_ 197 ×109/L
_x000D_
_x000D_
_x000D_ (150-400)
_x000D_
_x000D_
_x000D_
_x000D_ _x000D_ Serum Sodium
_x000D_
_x000D_
_x000D_ 143 mmol/L
_x000D_
_x000D_
_x000D_ (135-146)
_x000D_
_x000D_
_x000D_
_x000D_ _x000D_ Serum Potassium
_x000D_
_x000D_
_x000D_ 4.1 mmol/L
_x000D_
_x000D_
_x000D_ (3.5-5)
_x000D_
_x000D_
_x000D_
_x000D_ _x000D_ Serum Creatinine
_x000D_
_x000D_
_x000D_ 110 µmol/L
_x000D_
_x000D_
_x000D_ (79-118)
_x000D_
_x000D_
_x000D_
_x000D_ _x000D_ pH
_x000D_
_x000D_
_x000D_ 7.42
_x000D_
_x000D_
_x000D_ (7.35-7.45)
_x000D_
_x000D_
_x000D_
_x000D_ _x000D_ pCO2
_x000D_
_x000D_
_x000D_ 4.6 kPa
_x000D_
_x000D_
_x000D_ (4.8-6.1)
_x000D_
_x000D_
_x000D_
_x000D_ _x000D_ pO2
_x000D_
_x000D_
_x000D_ 9.9 kPa
_x000D_
_x000D_
_x000D_ (10-13.3)
_x000D_
_x000D_
_x000D_
_x000D_ _x000D_ Chest X-Ray
_x000D_
_x000D_
_x000D_ Bilateral Hilar Lymphadenopathy
_x000D_
_x000D_
_x000D_
_x000D_
_x000D_
Which one of the following is the most appropriate initial treatment?
CorrectIncorrectHint
Based on the clinical presentation and the chest x-ray finding of bilateral hilar lymphadenopathy, the most likely diagnosis for this lady is sarcoidosis. The diagnosis of sarcoidosis is also supported by erythema nodosum on both of her lower limbs and mild hypoxia on blood gas analysis. Sarcoidosis is a systemic granulomatous disease that affects primarily the lung and lymphatic system, but can involve any organ of the body. The cause of sarcoidosis is unknown, but it may be related to genetic, environmental, or infectious factors.
_x000D_
The most appropriate initial treatment for sarcoidosis depends on the severity and extent of the disease, as well as the presence of any organ-threatening complications. In general, patients with mild or asymptomatic disease do not require treatment, whereas patients with significantly symptomatic or progressive pulmonary disease or serious extrapulmonary disease (such as cardiac or neurologic involvement) require corticosteroid therapy.
_x000D_
Therefore, the best answer to this question is B. Prednisolone. Prednisolone is a type of corticosteroid that can reduce inflammation and suppress the immune system. It is the mainstay of therapy for sarcoidosis and can improve symptoms, lung function, and radiographic findings. The usual starting dose of prednisolone for sarcoidosis is 20 to 40 mg per day, which can be tapered gradually over several months to the lowest effective dose.
_x000D_
The other options are not recommended as initial treatment for sarcoidosis. Hydroxychloroquine, infliximab, methotrexate, and azathioprine are immunosuppressive agents that may be used as steroid-sparing agents or as second-line therapy for refractory or complex cases of sarcoidosis. However, they have not been proven to be superior to corticosteroids and have more potential side effects and risks. Therefore, they should be reserved for patients who cannot tolerate or respond to corticosteroids, or who have contraindications to corticosteroid use.
-
Question 92 of 123
92. Question
A 45-year-old lady comes to the health check clinic complaining of rapidly worsening lethargy and nausea. Over the past few days she has become increasingly unwell and is now barely able to get out of the house.
_x000D_
Other symptoms of note include progressive shortness of breath and a cough productive of blood stained sputum. Her only consultations with the doctor over the past eight months have been about the shape of her nose; she has suffered some collapse of her nasal bridge and is considering plastic surgery.
_x000D_
On examination, the physician notices collapse of the bridge of her nose, and nasal congestion when she speaks. Her blood pressure is elevated at 160/90 mmHg. Physician can hear crepitations on auscultation of the chest.
_x000D_
Investigations Show:
_x000D_
_x000D_ _x000D_
_x000D_ _x000D_ Haemoglobin
_x000D_
_x000D_
_x000D_ 135 g/L
_x000D_
_x000D_
_x000D_ (115-165)
_x000D_
_x000D_
_x000D_
_x000D_ _x000D_ Total Leucocyte Count
_x000D_
_x000D_
_x000D_ 12.5 ×109/L
_x000D_
_x000D_
_x000D_ (4-11)
_x000D_
_x000D_
_x000D_
_x000D_ _x000D_ Platelet Count
_x000D_
_x000D_
_x000D_ 210 ×109/L
_x000D_
_x000D_
_x000D_ (150-400)
_x000D_
_x000D_
_x000D_
_x000D_ _x000D_ Serum Sodium
_x000D_
_x000D_
_x000D_ 142 mmol/L
_x000D_
_x000D_
_x000D_ (135-146)
_x000D_
_x000D_
_x000D_
_x000D_ _x000D_ Serum Potassium
_x000D_
_x000D_
_x000D_ 6.1 mmol/L
_x000D_
_x000D_
_x000D_ (3.5-5)
_x000D_
_x000D_
_x000D_
_x000D_ _x000D_ Serum Creatinine
_x000D_
_x000D_
_x000D_ 290 µmol/L
_x000D_
_x000D_
_x000D_ (79-118)
_x000D_
_x000D_
_x000D_
_x000D_ _x000D_ Chest X-Ray
_x000D_
_x000D_
_x000D_ Patchy Interstitial Shadowing
_x000D_
_x000D_
_x000D_
_x000D_ _x000D_ C-Anti-Neutrophil Cytoplasmic Antibodies (C-ANCA)
_x000D_
_x000D_
_x000D_ Positive
_x000D_
_x000D_
_x000D_
_x000D_
_x000D_
Which one of the following is the most appropriate treatment?
CorrectIncorrectHint
The most obvious diagnosis in this case is granulomatosis with polyangiitis (GPA), formerly known as Wegener’s granulomatosis. This is a rare autoimmune disease that causes inflammation of the blood vessels and granulomas in various organs, especially the upper respiratory tract, lungs and kidneys. The symptoms and signs of this lady are consistent with granulomatosis with polyangiitis (GPA), such as nasal bridge collapse, nasal congestion, coughing up blood, shortness of breath, high blood pressure, elevated serum potassium and creatinine, and positive C-anti-neutrophil cytoplasmic antibodies (C-ANCA).
_x000D_
The most appropriate treatment for this lady is methylprednisolone and cyclophosphamide (option D). This is the standard regimen for inducing remission in severe granulomatosis with polyangiitis (GPA), as it has been shown to improve survival and preserve renal function. Methylprednisolone is a corticosteroid that suppresses inflammation, and cyclophosphamide is an immunosuppressant that targets the abnormal immune cells that cause granulomatosis with polyangiitis (GPA). These drugs are usually given intravenously for 3 to 6 months, followed by maintenance therapy with less toxic agents, such as azathioprine, methotrexate, or rituximab.
_x000D_
The other options are not as effective or appropriate for this lady:
_x000D_
Option A (methylprednisolone and methotrexate) is a possible alternative for mild to moderate granulomatosis with polyangiitis (GPA), but not for severe disease with renal involvement. Option B (methylprednisolone and azathioprine) is a common maintenance therapy after remission is achieved, but not for induction therapy. Option C (prednisolone) is a lower dose of corticosteroid that is insufficient for severe granulomatosis with polyangiitis (GPA), and should not be used alone without an immunosuppressant. Option E (infliximab) is a biologic agent that blocks tumour necrosis factor-alpha, a pro-inflammatory cytokine, but it has not been proven to be effective or safe for granulomatosis with polyangiitis (GPA), and may increase the risk of infections and malignancies.
-
Question 93 of 123
93. Question
A 51-year-old gentleman’s arterial blood gas (ABG) analysis gives the following results:
_x000D_
_x000D_ _x000D_
_x000D_ _x000D_ pO2
_x000D_
_x000D_
_x000D_ 10.3 kPa/75 mmHg
_x000D_
_x000D_
_x000D_ (11.3-12.6 kPa)
_x000D_
_x000D_
_x000D_
_x000D_ _x000D_ pCO2
_x000D_
_x000D_
_x000D_ 7.2 kPa/52 mmHg
_x000D_
_x000D_
_x000D_ (4.7-6.0 kPa)
_x000D_
_x000D_
_x000D_
_x000D_ _x000D_ pH
_x000D_
_x000D_
_x000D_ 7.49
_x000D_
_x000D_
_x000D_ (7.36-7.44)
_x000D_
_x000D_
_x000D_
_x000D_ _x000D_ Bicarbonate (pHCO3)
_x000D_
_x000D_
_x000D_ 40 mmol/L
_x000D_
_x000D_
_x000D_ (20-28)
_x000D_
_x000D_
_x000D_
_x000D_
_x000D_
Which one of the following is the most likely cause?
CorrectIncorrectHint
Based on the arterial blood gas (ABG) results, the most likely cause is B. Pyloric obstruction. This is because the gentleman has a metabolic alkalosis, which is a condition where the blood pH is higher than normal due to an excess of bicarbonate or a loss of acid. This gentleman’s pO2 and pCO2 are also slightly lower than normal, indicating a mild respiratory compensation.
_x000D_
Pyloric obstruction is a condition where the opening between the stomach and the small intestine (the pylorus) is narrowed or blocked, preventing food from passing through. This can cause vomiting, dehydration, and loss of gastric acid, leading to metabolic alkalosis. Other causes of metabolic alkalosis include diuretic use, excessive antacid intake, and primary hyperaldosteronism.
_x000D_
The other answer options can be ruled out with the following explanations:
_x000D_
- _x000D_
- Diabetic coma: Diabetic coma is a life-threatening complication of diabetes, where the blood sugar level is too high or too low. Diabetic coma can cause either a metabolic acidosis or a metabolic alkalosis, depending on the type of diabetes and the underlying cause. However, in both cases, the pCO2 would be low, not high, as the respiratory system tries to compensate for the metabolic disturbance.
- Pulmonary embolism: Pulmonary embolism is a condition where a blood clot blocks one or more of the pulmonary arteries, reducing blood flow to the lungs. Pulmonary embolism can cause a respiratory acidosis, where the blood pH is lower than normal due to a build-up of carbon dioxide. This gentleman’s pO2 would also be very low, indicating hypoxaemia.
- Acute exacerbation of chronic obstructive pulmonary disease (COPD): Chronic obstructive pulmonary disease (COPD) is a chronic lung disease that causes airflow obstruction and difficulty breathing. An acute exacerbation of chronic obstructive pulmonary disease (COPD) is a sudden worsening of symptoms, often triggered by an infection or environmental factors. An acute exacerbation of chronic obstructive pulmonary disease (COPD) can cause a respiratory acidosis, similar to pulmonary embolism, but with a higher pCO2 and a lower pH.
- Chronic hyperventilation syndrome: Chronic hyperventilation syndrome is a condition where a person breathes faster or deeper than normal for a prolonged period of time, without a medical reason. Chronic hyperventilation syndrome can cause a respiratory alkalosis, where the blood pH is higher than normal due to a loss of carbon dioxide. This gentleman’s pCO2 would be very low, not high, as the respiratory system overcompensates for the perceived lack of oxygen.
_x000D_
_x000D_
_x000D_
_x000D_
-
Question 94 of 123
94. Question
A 51-year-old gentleman with a long history of alcoholism attends respiratory clinic with cough, haemoptysis and pleuritic chest pain. He has had night sweats and 12 kilogram weight loss in the last four months. On chest x-ray there is a subtle nodular pattern throughout the lung.
_x000D_
He underwent a transbronchial biopsy, which showed multinucleated giant cells, epithelioid cells and necrotic debris.
_x000D_
Which one of the following is the most plausible diagnosis?
CorrectIncorrectHint
This is a challenging question that requires careful analysis of the clinical and pathological features. Based on the information provided as above, the most plausible diagnosis among the options is B. Tuberculosis (TB).
_x000D_
Tuberculosis is a chronic infectious disease caused by Mycobacterium tuberculosis, a bacterium that usually affects the lungs but can also involve other organs. Tuberculosis can present with various symptoms, such as cough, haemoptysis, chest pain, fever, night sweats, weight loss, and fatigue. Tuberculosis can also cause granulomatous inflammation in the lung tissue, which is characterized by the presence of epithelioid histiocytes, multinucleated giant cells, and necrotic debris. These features are consistent with the transbronchial biopsy findings in this case.
_x000D_
The other options are less likely because:
_x000D_
- _x000D_
- A. Small cell carcinoma is a type of lung cancer that typically occurs in heavy smokers and is associated with rapid growth, early metastasis, and poor prognosis. Small cell carcinoma has a characteristic morphology of small, round, blue cells with scant cytoplasm, nuclear molding, and necrosis. Small cell carcinoma does not form granulomas or multinucleated giant cells.
- C. Squamous cell carcinoma is another type of lung cancer that is also related to smoking and usually arises in the central bronchi. Squamous cell carcinoma has a distinctive appearance of keratinizing cells with intercellular bridges, keratin pearls, and necrosis. Squamous cell carcinoma does not produce granulomas or multinucleated giant cells.
- D. Aspergillosis is a fungal infection caused by Aspergillus species, which can cause various pulmonary manifestations, such as allergic bronchopulmonary aspergillosis, aspergilloma, chronic necrotizing aspergillosis, and invasive aspergillosis. Aspergillosis can cause granulomatous inflammation, but the granulomas are usually well-formed and contain fungal hyphae that can be seen with special stains. Aspergillosis is more common in immunocompromised patients or those with underlying lung diseases.
- E. Pneumocystis jirovecii pneumonia (PCP) is a type of pneumonia caused by Pneumocystis jirovecii, an opportunistic fungus that infects the alveoli and causes diffuse interstitial inflammation. Pneumocystis jirovecii pneumonia is mainly seen in patients with HIV/AIDS or other immunosuppressive conditions. Pneumocystis jirovecii pneumonia does not cause granulomatous inflammation, but rather foamy exudates and cysts that can be detected with special stains.
_x000D_
_x000D_
_x000D_
_x000D_
-
Question 95 of 123
95. Question
A 51-year-old gentleman has been diagnosed with pulmonary tuberculosis.
_x000D_
Which one of the following investigations is essential prior to starting antituberculous therapy?
CorrectIncorrectHint
The correct answer to this question is D. Liver function test (LFT). This is an essential investigation prior to starting antituberculous therapy, as recommended by the Joint Tuberculosis Committee of the British Thoracic Society.
_x000D_
The reason for this is that some of the antituberculous drugs, especially isoniazid and rifampicin, can cause hepatotoxicity (liver damage) in some patients. Therefore, it is important to check the baseline liver function before initiating treatment, and monitor it periodically during the course of therapy.
_x000D_
Liver function test includes measuring the levels of serum bilirubin, alanine aminotransferase (ALT), aspartate aminotransferase (AST), alkaline phosphatase (ALP), and gamma-glutamyl transferase (GGT). These enzymes and substances reflect the health and function of the liver cells and bile ducts.
_x000D_
If the liver function test shows abnormal results, such as elevated serum alanine aminotransaminase (ALT) or bilirubin, the antituberculous therapy may need to be modified, interrupted, or discontinued, depending on the severity of the liver injury and the clinical condition of the patient.
_x000D_
The other investigations listed in the question are not essential prior to starting antituberculous therapy, although they may be useful for other purposes, such as:
_x000D_
- _x000D_
- A. Vitamin B6: This is a vitamin that is involved in various metabolic processes in the body. Some patients who take isoniazid may develop vitamin B6 deficiency, which can cause peripheral neuropathy (nerve damage). Therefore, vitamin B6 supplementation is recommended for patients who are at risk of developing this condition, such as pregnant or breastfeeding women, malnourished patients, or patients with human immunodeficiency virus (HIV) infection.
- B. Complete blood count (CBC): This is a test that measures the number and types of blood cells, such as red blood cells, white blood cells, and platelets. Complete blood count (CBC) can help diagnose and monitor various conditions, such as anaemia, infection, or bleeding disorders. Some antituberculous drugs, such as rifampicin and pyrazinamide, can cause haematological adverse effects, such as haemolytic anaemia, thrombocytopaenia, or leukopaenia. Therefore, complete blood count (CBC) may be useful to check the baseline blood counts and monitor them during the treatment, especially if the individual develops symptoms of haematological toxicity, such as fatigue, pallor, bruising, or bleeding.
- C. Urine for acid-fast bacilli (AFB): This is a test that detects the presence of mycobacteria, the causative agents of tuberculosis, in the urine sample. The urine is stained with a special dye that makes the mycobacteria appear red under a microscope. Urine for acid-fast bacilli (AFB) can help diagnose and monitor urinary tract tuberculosis, which is a rare form of extrapulmonary tuberculosis that affects the kidneys, ureters, bladder, or genital organs. Urinary tract tuberculosis can cause symptoms such as flank pain, dysuria, haematuria, or renal failure. Urine for acid-fast bacilli (AFB) is not routinely done before starting antituberculous therapy, unless there is a suspicion of urinary tract tuberculosis based on the clinical presentation or imaging findings.
- E. Plasma glucose: This is a test that measures the level of glucose (sugar) in the blood. Plasma glucose can help diagnose and monitor diabetes mellitus, which is a chronic condition that affects the metabolism of glucose and insulin. Diabetes mellitus is a risk factor for developing tuberculosis, as it impairs the immune system and increases the susceptibility to infection. Therefore, plasma glucose may be useful to screen for diabetes mellitus in patients with tuberculosis, and to manage the blood glucose levels during the antituberculous therapy.
_x000D_
_x000D_
_x000D_
_x000D_
-
Question 96 of 123
96. Question
A 50-year-old lady presents to the Emergency Department with a two-week history of a non-productive cough and increasing breathlessness. She reports her breathing is much worse on exertion.
_x000D_
Her past medical history includes hypertension, migraines and a renal transplant for end stage hypertensive nephropathy (ESHN).
_x000D_
Examination reveals mild pyrexia of 100.5°F and most notably she was profoundly hypoxic with oxygen saturations of 80% on room air. An arterial blood gas confirmed her hypoxia. A chest x-ray showed some patchy bilateral infiltrates, more pronounced on the left.
_x000D_
She is diagnosed with community-acquired pneumonia (CAP) and treated empirically with recommended antibiotics. The next day she deteriorates and requires intubation and ventilation.
_x000D_
Which one of the following is the most likely causative organism?
CorrectIncorrectHint
The most likely causative organism of community-acquired pneumonia (CAP) in this lady is A. Pneumocystis jirovecii. This is because she has a history of renal transplant and is likely to be immunosuppressed, which increases the risk of Pneumocystis jirovecii pneumonia (PCP), a serious infection caused by this fungus. Pneumocystis jirovecii pneumonia (PCP) typically presents with non-productive cough, dyspnoea, fever, and hypoxia, and can cause bilateral interstitial infiltrates on chest x-ray. Pneumocystis jirovecii pneumonia (PCP) is one of the most common opportunistic infections in patients with human immunodeficiency virus (HIV) infection/acquired immunodeficiency syndrome (AIDS), but it can also affect other immunocompromised patients, such as those who have received organ transplants or are taking corticosteroids.
_x000D_
The other options are less likely because:
_x000D_
- _x000D_
- B. Legionella pneumophila is a bacterium that causes Legionnaires’ disease, a form of pneumonia that is usually associated with exposure to contaminated water sources, such as cooling towers, fountains, or showers. Legionnaires’ disease can affect healthy or immunocompromised people, but it is more common in older adults, smokers, and people with chronic lung diseases. Legionnaires’ disease may cause gastrointestinal symptoms, such as nausea, vomiting, and diarrhoea, and neurological symptoms, such as confusion or mental changes, in addition to respiratory symptoms.
- C. Pseudomonas aeruginosa is a bacterium that causes pneumonia, especially in people with underlying respiratory conditions, such as cystic fibrosis or chronic obstructive pulmonary disease. Pseudomonas pneumonia is also a common cause of hospital-acquired or ventilator-associated pneumonia, but it is less likely to cause community-acquired pneumonia. Pseudomonas pneumonia can be severe and difficult to treat because of antibiotic resistance.
- D. Chlamydia pneumoniae is a bacterium that causes respiratory tract infections, such as pneumonia, especially in children and young adults. Chlamydia pneumoniae pneumonia may be clinically indistinguishable from pneumonia caused by Mycoplasma pneumoniae, another common cause of atypical pneumonia. In some patients with Chlamydia pneumoniae, pneumonia, hoarseness, and sore throat may precede coughing, which may be persistent and complicated by bronchospasm.
- E. Pneumocystis carinii is an outdated name for Pneumocystis jirovecii, the fungus that causes Pneumocystis jirovecii pneumonia (PCP). The name was changed in 1999 to reflect the differences between the human and rat isolates of the organism. Therefore, option E is the same as option A.
_x000D_
_x000D_
_x000D_
_x000D_
-
Question 97 of 123
97. Question
A 50-year-old lady presenting with breathlessness has Arterial Blood Gases [ABGs] taken which give the following results:
_x000D_
_x000D_ _x000D_
_x000D_ _x000D_ pO2
_x000D_
_x000D_
_x000D_ 8.5 kPa / 65 mmHg
_x000D_
_x000D_
_x000D_ (11.3-12.6)
_x000D_
_x000D_
_x000D_
_x000D_ _x000D_ pCO2
_x000D_
_x000D_
_x000D_ 4.2 kPa / 33 mmHg
_x000D_
_x000D_
_x000D_ (4.7-6.0)
_x000D_
_x000D_
_x000D_
_x000D_ _x000D_ pH
_x000D_
_x000D_
_x000D_ 7.5
_x000D_
_x000D_
_x000D_ (7.36-7.44)
_x000D_
_x000D_
_x000D_
_x000D_ _x000D_ HCO3
_x000D_
_x000D_
_x000D_ 26 mmol/L
_x000D_
_x000D_
_x000D_ (20-28)
_x000D_
_x000D_
_x000D_
_x000D_
_x000D_
Which of the following is the most likely diagnosis?
CorrectIncorrectHint
The patient has an Acute Respiratory Alkalosis with associated Hypoxia. This is consistent with an Acute Asthmatic Attack._x000D_
A normal or rising CO2 is an ominous sign indicative of a life-threatening attack and the need to consider Ventilatory Support._x000D_
Patients with Hyperventilation Syndrome do show a respiratory alkalosis but this is not associated with hypoxia. -
Question 98 of 123
98. Question
A 50-year-old lady presented to the Pulmonology Clinic with shortness of breath, cough with heavy sputum production, and a low grade fever.
_x000D_
She has smoked 20 cigarettes per day for 30 years.
_x000D_
Her Arterial Blood Gases revealed:
_x000D_
_x000D_ _x000D_
_x000D_ _x000D_ pH
_x000D_
_x000D_
_x000D_ 7.41
_x000D_
_x000D_
_x000D_ 7.36-7.44
_x000D_
_x000D_
_x000D_
_x000D_ _x000D_ pCO2
_x000D_
_x000D_
_x000D_ 6.1 kPa
_x000D_
_x000D_
_x000D_ (4.5-6)
_x000D_
_x000D_
_x000D_
_x000D_ _x000D_ pO2
_x000D_
_x000D_
_x000D_ 7.8 kPa
_x000D_
_x000D_
_x000D_ (8-12)
_x000D_
_x000D_
_x000D_
_x000D_
_x000D_
What is the most likely diagnosis?
CorrectIncorrectHint
The most likely explanation based on the symptoms and the relative hypoxia with high pCO2 is an acute exacerbation of chronic obstructive airways disease (COAD) – towards the Chronic Bronchitic End Of The Spectrum.
-
Question 99 of 123
99. Question
A 50-year-old gentleman with moderate Chronic Obstructive Pulmonary Disease (COPD), and multiple previous presentations to the Emergency Department presents with a three-hour history of mild pleuritic chest pain.
_x000D_
He is minimally breathless, with Oxygen Saturations of 96% on Room Air (he usually has Saturations of 95-97% on Room Air).
_x000D_
A Chest X-Ray is performed and an observant CT1 recognises a 1.8 cm (18 mm) apical pneumothorax on a background of chronic changes consistent with emphysema / emphysematous changes.
_x000D_
Accordingly to current guidelines, what intervention should be undertaken?
CorrectIncorrectHint
This patient has a Small Apical Pneumothorax most likely as a result of his underlying lung disease and as such can be classed as a secondary pneumothorax._x000D_
According to current BTS Guidelines, if the patient is under 50, minimally breathless and the rim of air is less than 2 cm the initial intervention should be simple aspiration. Even if successful the patient should be admitted and observed for at least 24 hours._x000D_
Unlike a Primary Pneumothorax, a Secondary Pneumothorax always requires intervention._x000D_
Aspiration is less likely to be effective in a Secondary Pneumothorax and so if it fails or does not meet the above criteria, a Chest Drain needs to be inserted. The Seldinger Technique using a 16G is the preferred method for this._x000D_
Blunt Dissection is usually reserved for Trauma Cases. -
Question 100 of 123
100. Question
A 50-year-old lady presenting with breathlessness has arterial blood gases (ABGs) taken which give the following results:
_x000D_
_x000D_ _x000D_
_x000D_ _x000D_ pO2
_x000D_
_x000D_
_x000D_ 8.5 kPa / 65 mmHg
_x000D_
_x000D_
_x000D_ (11.3-12.6)
_x000D_
_x000D_
_x000D_
_x000D_ _x000D_ pCO2
_x000D_
_x000D_
_x000D_ 4.2 kPa / 33 mmHg
_x000D_
_x000D_
_x000D_ (4.7-6.0)
_x000D_
_x000D_
_x000D_
_x000D_ _x000D_ pH
_x000D_
_x000D_
_x000D_ 7.5
_x000D_
_x000D_
_x000D_ (7.36-7.44)
_x000D_
_x000D_
_x000D_
_x000D_ _x000D_ HCO3
_x000D_
_x000D_
_x000D_ 26 mmol/L
_x000D_
_x000D_
_x000D_ (20-28)
_x000D_
_x000D_
_x000D_
_x000D_
_x000D_
Which one of the following is the most likely diagnosis?
CorrectIncorrectHint
The correct answer is option E. Acute severe asthma. This is because the lady in the question has an acute respiratory alkalosis with associated hypoxia, which means that her blood pH is high (7.5) and her oxygen level is low (8.5 kPa / 65 mmHg). This can be caused by an acute asthmatic attack, which is a sudden worsening of asthma symptoms that does not respond to standard treatments. Asthma is a chronic condition that causes inflammation and narrowing of the airways, leading to difficulty breathing and wheezing. An acute asthmatic attack can be triggered by various factors, such as allergens, infections, stress, or exercise. During an attack, the airways become more inflamed and constricted, and mucus production increases, resulting in reduced airflow and gas exchange. This causes the lady to breathe faster and deeper to compensate for the lack of oxygen, which lowers her carbon dioxide level and raises her blood pH. However, this also causes her to lose more oxygen, which worsens her hypoxia. A normal or rising carbon dioxide level in an asthmatic patient is an ominous sign that indicates a life-threatening attack and the need to consider ventilatory support. This is because it means that the patient is unable to breathe adequately and is at risk of respiratory failure.
_x000D_
The incorrect options can be ruled out with the following explanations:
_x000D_
- _x000D_
- Option A is incorrect because emphysema is a type of chronic obstructive pulmonary disease (COPD) that causes respiratory acidosis, not alkalosis. Emphysema is a condition where the air sacs in the lungs are damaged and enlarged, leading to reduced gas exchange and increased carbon dioxide level.
- Option B is incorrect because opiate overdose is another cause of respiratory acidosis, not alkalosis. Opiate overdose is a condition where the use of opioids, such as heroin, morphine, or fentanyl, depresses the respiratory centre in the brain and reduces the breathing rate and depth.
- Option C is incorrect because kyphoscoliosis is a rare cause of respiratory alkalosis, and usually presents with chronic symptoms, not acute breathlessness. Kyphoscoliosis is a deformity of the spine that affects the shape and function of the chest wall and the lungs.
- Option D is incorrect because hyperventilation syndrome is a common cause of respiratory alkalosis, but it is not associated with hypoxia. Hyperventilation syndrome is a condition where a person breathes too fast or too deep due to anxiety, panic, or stress. This lowers the carbon dioxide level and raises the blood pH, but it does not affect the oxygen level. Hyperventilation syndrome can cause symptoms such as chest pain, dizziness, and tingling sensations, but it does not cause wheezing or reduced air entry.
_x000D_
_x000D_
_x000D_
_x000D_
-
Question 101 of 123
101. Question
A 50-year-old gentleman with asthma develops worsening breathlessness and his complete blood count (CBC) has revealed an eosinophilia. A diagnosis of allergic bronchopulmonary aspergillosis (ABPA) is suspected.
_x000D_
Which one of the following statements is true with regard to this diagnosis?
CorrectIncorrectHint
The correct answer is option C. Circulating IgG precipitins to Aspergillus fumigatus are positive.
_x000D_
Circulating IgG precipitins to Aspergillus fumigatus are positive in allergic bronchopulmonary aspergillosis (ABPA). This is one of the diagnostic criteria for allergic bronchopulmonary aspergillosis (ABPA), along with elevated IgE levels, Aspergillus-specific antibodies, and a positive skin test. IgG precipitins are antibodies that form complexes with antigens and precipitate out of solution. They indicate a type III hypersensitivity reaction to Aspergillus fumigatus, which is part of the pathogenesis of allergic bronchopulmonary aspergillosis (ABPA). IgG precipitins can be detected by immunodiffusion assay or enzyme-linked immunosorbent assay (ELISA).
_x000D_
Let us rule out the other options with explanations:
_x000D_
- _x000D_
- A. Pleural effusion (PE) is a complication: False: Pleural effusion is not a complication of allergic bronchopulmonary aspergillosis (ABPA). Pleural effusion (PE) is an abnormal accumulation of fluid in the pleural space, which can be caused by various conditions such as infection, inflammation, malignancy, or trauma. Allergic bronchopulmonary aspergillosis (ABPA) is a hypersensitivity reaction to Aspergillus fumigatus that affects the bronchi and bronchioles, causing mucoid impaction, bronchiectasis, and fibrosis. Allergic bronchopulmonary aspergillosis (ABPA) does not directly involve the pleura or cause fluid leakage into the pleural space.
- B. Recurrent haemoptysis is a characteristic feature: False: Recurrent haemoptysis is not a characteristic feature of allergic bronchopulmonary aspergillosis (ABPA), although it may occur in some cases. Haemoptysis is the coughing up of blood from the respiratory tract, which can be caused by various conditions such as infection, inflammation, trauma, or malignancy. Allergic bronchopulmonary aspergillosis (ABPA) causes bronchial wall damage and inflammation, which may lead to bleeding in some patients, but this is not a common or specific manifestation of allergic bronchopulmonary aspergillosis (ABPA).
- D. The CO transfer factor is unaffected: False: The CO transfer factor is reduced in allergic bronchopulmonary aspergillosis (ABPA). The CO transfer factor, also known as the diffusing capacity of the lung for carbon monoxide (DLCO), is a measure of how well the lungs can transfer gas from the air to the blood. It is affected by various factors such as lung volume, alveolar surface area, capillary blood volume, and haemoglobin concentration. Allergic bronchopulmonary aspergillosis (ABPA) causes airway obstruction and inflammation, which reduce the lung volume and alveolar surface area, leading to impaired gas exchange and lower diffusing capacity of the lung for carbon monoxide (DLCO).
- E. The immediate skin test to an extract of Aspergillus fumigatus is negative: False: The immediate skin test to an extract of Aspergillus fumigatus is positive in allergic bronchopulmonary aspergillosis (ABPA). This is another diagnostic criterion for allergic bronchopulmonary aspergillosis (ABPA), along with elevated IgE levels, Aspergillus-specific antibodies, and IgG precipitins. The skin test is a type of allergy test that involves injecting a small amount of Aspergillus fumigatus extract into the skin and observing the reaction. A positive skin test indicates a type I hypersensitivity reaction to Aspergillus fumigatus, which is also part of the pathogenesis of allergic bronchopulmonary aspergillosis (ABPA).
_x000D_
_x000D_
_x000D_
_x000D_
-
Question 102 of 123
102. Question
A 50-year-old gentleman with a known history of chronic alcohol abuse presented with a four-day history of fevers, night sweats and a cough productive of purulent sputum.
_x000D_
There was no past history of respiratory disease.
_x000D_
On examination, he was pyrexial (39.1°C). Percussion note was dull over the right apex and there was bronchial breathing in this area on auscultation.
_x000D_
The Chest X-Ray showed Right Upper Lobar Consolidation.
_x000D_
Other Investigations Revealed:
_x000D_
_x000D_ _x000D_
_x000D_ _x000D_ White Blood Cell Count
_x000D_
_x000D_
_x000D_ 25 ×109/L
_x000D_
_x000D_
_x000D_ (4-11)
_x000D_
_x000D_
_x000D_
_x000D_ _x000D_ Neutrophils
_x000D_
_x000D_
_x000D_ 18.5 ×109/L
_x000D_
_x000D_
_x000D_ (1.5-7)
_x000D_
_x000D_
_x000D_
_x000D_
_x000D_
What is the most likely diagnosis?
CorrectIncorrectHint
Community-acquired Klebsiella pneumonia is a disease of debilitated middle-aged and older men with alcoholism. Mortality rates are as high as 50% regardless of treatment Klebsiella pneumonia characteristically affects one of the upper lobes of the lung, although infection of the lower lobes is not uncommon. There is an increased tendency toward abscess formation._x000D_
Aspiration Pneumonia typically affects in right lower lobe in patients with impaired swallowing._x000D_
Legionnaires’ Disease is associated with contaminated air conditioning and water delivery systems._x000D_
Mycoplasma pneumoniae infections have an insidious onset with malaise, myalgia, sore throat and headache. Cough is characteristically dry. Chest X-Ray (CXR) Changes are usually patchy and involve the lower or middle lobes._x000D_
Primary Tuberculosis [TB] does not commonly cause a lobar pneumonia, therefore whilst it is a possibility it is not the most likely diagnosis. -
Question 103 of 123
103. Question
A 50-year-old gentleman attends the Emergency Department with symptoms suggestive of community-acquired pneumonia (CAP).
_x000D_
On examination, he is pyrexial at 100.4°F and has a respiratory rate of 32 breaths per minute, with a blood pressure of 90/60 mmHg. Serum urea is 7.5 mmol/L.
_x000D_
Which one of the following combinations of features is not necessarily an indication for urgent hospital admission?
CorrectIncorrectHint
The correct answer is B. Pyrexia of 100.4°F and serum urea of 7.5 mmol/L. This is because these features alone are not sufficient to indicate the need for urgent hospital admission in patients with community-acquired pneumonia (CAP). According to current guidelines, the decision to admit a patient with community-acquired pneumonia (CAP) should be based on a comprehensive assessment of the severity of illness, the presence of comorbidities, the risk of complications, and the availability of home support.
_x000D_
One of the tools that can be used to assess the severity of community-acquired pneumonia (CAP) is the CURB-65 score, which assigns one point for each of the following criteria: confusion, urea >7 mmol/L, respiratory rate ≥30 breaths per minute, blood pressure <90/60 mmHg, and age ≥65 years. A CURB-65 score of 0 or 1 indicates low risk of mortality and outpatient treatment, a score of 2 indicates moderate risk and possible hospital admission, and a score of 3 or more indicates high risk and definite hospital admission.
_x000D_
This gentleman in the question has a CURB-65 score of 3, which means that he should be admitted to the hospital. However, this score is based on the combination of his blood pressure, respiratory rate, serum urea, and age. If he only had pyrexia and serum urea of 7.5 mmol/L, his CURB-65 score would be 1, which would not necessarily warrant hospital admission, unless he had other factors that increased his risk.
_x000D_
The other answer options are incorrect for the following reasons:
_x000D_
- _x000D_
- A. Blood pressure of 90/60 mmHg and respiratory rate of 32 breaths per minute: These features indicate hypotension and tachypnoea, which are signs of respiratory failure and sepsis. They also contribute to the CURB-65 score and increase the risk of mortality. Therefore, they are indications for urgent hospital admission.
- C. Blood pressure of 90/60 mmHg and serum urea of 7.5 mmol/L: These features indicate hypotension and renal impairment, which are signs of sepsis and organ dysfunction. They also contribute to the CURB-65 score and increase the risk of mortality. Therefore, they are indications for urgent hospital admission.
- D. Respiratory rate of 32 and serum blood urea of 7.5 mmol/L: These features indicate tachypnoea and renal impairment, which are signs of respiratory failure and organ dysfunction. They also contribute to the CURB-65 score and increase the risk of mortality. Therefore, they are indications for urgent hospital admission.
- E. Confusion and blood pressure of 90/60 mmHg: These features indicate altered mental status and hypotension, which are signs of sepsis and encephalopathy. They also contribute to the CURB-65 score and increase the risk of mortality. Therefore, they are indications for urgent hospital admission.
_x000D_
_x000D_
_x000D_
_x000D_
-
Question 104 of 123
104. Question
A 49-year-old gentleman presents to the Chest Clinic with shortness of breath.
_x000D_
He has Cirrhosis secondary to Hepatitis C Infection.
_x000D_
He gives a chronic history of progressive shortness of breath on exertion and now becomes short of breath walking up steps. He gives a history of being more short of breath whilst sitting up, preferring to sleep with no pillows.
_x000D_
His Blood Pressure is 110/70 mmHg, Heart Rate is 82 Beats Per Minute, and Pulse Oximetry, breathing room air, shows saturations of 95% lying flat and 87% sitting up.
_x000D_
Which of the following tests is the most appropriate to confirm the diagnosis?
CorrectIncorrectHint
This patient has Hepatopulmonary Syndrome. This is characterised by an oxygenation defect induced by pulmonary vascular dilatation in patients with Liver Cirrhosis or Portal Hypertension._x000D_
This oxygenation defect consists of a wide alveolar-arterial gradient (>15 mmHg). The vascular dilatation is thought to be induced by increased pulmonary levels of nitric oxide. It is seen in 15-30% of patients with Cirrhosis and is a poor prognostic indicator._x000D_
Most patients with Hepatopulmonary Syndrome are asymptomatic or have dyspnoea with an insidious onset. Dyspnoea whilst standing (Platypnoea) and Hypoxaemia exacerbated by being upright (Orthodeoxia) are characteristic and are thought to be due to the predominance of vascular dilatation in the lung bases. Blood flow to these areas is increased in the upright position._x000D_
Desaturation during sleep is also often seen, as are clubbing and cyanosis._x000D_
Contrast-Enhanced Transthoracic Echocardiography is the best test to demonstrate Intrapulmonary Vascular Dilatation. It can also exclude Intracardiac Shunting which may result in similar signs and symptoms to Hepatopulmonary Syndrome._x000D_
This is performed by injecting agitated saline intravenously during Transthoracic Echocardiography. In a normal subject, microbubbles are visualised in the right ventricle within seconds, which are then absorbed in the alveoli. Immediate visualisation in the left ventricle (within three cardiac cycles) indicates intracardiac shunting. Delayed visualisation in the left ventricle (3-6 cardiac cycles) is diagnostic of intrapulmonary shunting._x000D_
Radionuclide lung perfusion scanning can also be used. Chest radiographs can be normal or show non-specific interstitial changes. ABGs should be taken in the sitting position to grade the severity of the condition based on the degree of hypoxaemia._x000D_
Liver transplantation is the only proven beneficial available treatment, with 85% of patients showing resolution or significant improvement in gas exchange postoperatively. -
Question 105 of 123
105. Question
A 55-year-old gentleman presents to the Respiratory Clinic. He has been involved in Nuclear Power Plant Construction for much of his life and has increasing shortness of breath and chronic cough over the past 12 months.
_x000D_
On examination, his Blood Pressure is 140/80 mmHg, Pulse Rate is 72/Minute and Heart Sounds are Normal. There are occasional crackles on auscultation of the chest.
_x000D_
Investigations Show:
_x000D_
_x000D_ _x000D_
_x000D_ _x000D_ Haemoglobin
_x000D_
_x000D_
_x000D_ 122 g/L
_x000D_
_x000D_
_x000D_ (135-177)
_x000D_
_x000D_
_x000D_
_x000D_ _x000D_ Total Leucocyte Count
_x000D_
_x000D_
_x000D_ 10.4 ×109/L
_x000D_
_x000D_
_x000D_ (4-11)
_x000D_
_x000D_
_x000D_
_x000D_ _x000D_ Platelet Count
_x000D_
_x000D_
_x000D_ 206 ×109/L
_x000D_
_x000D_
_x000D_ (150-400)
_x000D_
_x000D_
_x000D_
_x000D_ _x000D_ Serum Sodium
_x000D_
_x000D_
_x000D_ 141 mmol/L
_x000D_
_x000D_
_x000D_ (135-146)
_x000D_
_x000D_
_x000D_
_x000D_ _x000D_ Serum Potassium
_x000D_
_x000D_
_x000D_ 4.2 mmol/L
_x000D_
_x000D_
_x000D_ (3.5-5)
_x000D_
_x000D_
_x000D_
_x000D_ _x000D_ Serum Creatinine
_x000D_
_x000D_
_x000D_ 124 µmol/L
_x000D_
_x000D_
_x000D_ (79-118)
_x000D_
_x000D_
_x000D_
_x000D_
_x000D_
Chest X-Ray shows Bilateral Hilar Lymphadenopathy.
_x000D_
Which one of the following is the most likely diagnosis?
CorrectIncorrectHint
Exposure to Beryllium is seen in the Nuclear Power, Telecommunications, Semi-Conductor and Electronics Industries. It results in a similar clinical picture to that of Sarcoidosis; in this case it is the patient’s occupation that pushes us towards Berylliosis as the most appropriate answer._x000D_
Sarcoidosis is the major differential here, but it is the possible exposure to Beryllium because of occupation which points us away from this as the most likely diagnosis._x000D_
Idiopathic Pulmonary Fibrosis would be associated with evidence of Interstitial Fibrosis on Chest X-Ray._x000D_
We are not told of any smoking history consistent with chronic obstructive pulmonary disease (COPD) and there is no evidence of wheeze to suggest asthma. -
Question 106 of 123
106. Question
A 55-year-old Orthopaedic Secretary presents with a five-month history of dry cough. She has never smoked.
_x000D_
She denies haemoptysis, weight loss and dyspnoea. She has no post nasal drip.
_x000D_
Her medical history consists only of Hypertension for the last one year. She is on Ramipril 2.5 mg, Bendroflumethiazide 2.5 mg and Amlodipine 10 mg.
_x000D_
On examination. her Jugular Venous Pressure [JVP] is not raised and her chest is clear. A Chest X-Ray was requested.
_x000D_
What would be the most likely cause of her cough?
CorrectIncorrectHint
ACE Inhibitors can cause dry cough. They can then be replaced with an Angiotensin Receptor Blocker. It is important to ask about the onset of cough and the start of a new drug._x000D_
There no history to suggest Allergic Rhinitis._x000D_
Bronchiectasis causes a Productive Cough._x000D_
Interstitial Lung Disease and Lung Cancer may be causes but they are not the most likely and in this scenario there is nothing to suggest either. -
Question 107 of 123
107. Question
A 55-year-old Afro-American lady with severe rheumatoid arthritis has failed on most traditional disease-modifying antirheumatic drug (DMARD) treatments.
_x000D_
She is currently on methotrexate 20 mg weekly and for the last six months has been receiving regular infusions of the anti-TNF-alpha monoclonal antibody, infliximab. Her joint disease has dramatically improved.
_x000D_
She now presents with fever, pleuritic chest pain and a large left sided pleural effusion, but little evidence of joint synovitis.
_x000D_
Which one of the following is the most likely diagnosis?
CorrectIncorrectHint
The most likely diagnosis is option C. Tuberculosis. This is because infliximab, the anti-TNF-alpha monoclonal antibody that she has been receiving, is associated with an increased risk of tuberculosis and other infections. Infliximab may cause reactivation of latent tuberculosis or new infection with Mycobacterium tuberculosis, the bacteria that causes tuberculosis. Tuberculosis can cause pleural effusion, which is the accumulation of fluid in the pleural space between the lungs and the chest wall. Tuberculous pleural effusion is the second most common form of extrapulmonary tuberculosis and is the most common cause of pleural effusion in areas where tuberculosis is endemic. Tuberculous pleural effusion usually presents as an acute illness with fever, cough and pleuritic chest pain.
_x000D_
The other options are less likely for the following reasons:
_x000D_
- _x000D_
- Option A is incorrect because rheumatoid related effusion is usually bilateral, small, and asymptomatic. The lady in the question has a large, left sided, and symptomatic effusion.
- Option B is incorrect because primary bronchial carcinoma is rare in non-smokers and usually causes malignant pleural effusion, which is associated with weight loss, dyspnoea, and chest wall pain. The lady in the question has no history of smoking or these symptoms.
- Option D is incorrect because pulmonary embolus is unlikely in the absence of risk factors such as immobilization, surgery, trauma, or coagulopathy. The lady in the question has none of these risk factors.
- Option E is incorrect because pulmonary metastases are uncommon in rheumatoid arthritis and usually cause multiple nodules or masses in the lungs, not pleural effusion. The lady in the question has no evidence of pulmonary involvement.
_x000D_
_x000D_
_x000D_
_x000D_
-
Question 108 of 123
108. Question
A 55-year-old Afro-American lady with severe rheumatoid arthritis has failed on most traditional Disease-Modifying Antirheumatic Drug [DMARD] treatments.
_x000D_
She is currently on Methotrexate 20 mg weekly and for the last six months has been receiving regular infusions of the Anti-TNF-Alpha Monoclonal Antibody, Infliximab. Her Joint Disease has dramatically improved.
_x000D_
She now presents with fever, pleuritic chest pain and a large left sided pleural effusion, but little evidence of joint synovitis.
_x000D_
What is the most likely diagnosis?
CorrectIncorrectHint
The most likely answer is Tuberculosis [TB]._x000D_
All of the other answers are possible and need to be excluded._x000D_
A rheumatoid effusion is unlikely when peripheral joint disease is so well controlled._x000D_
Treatment with Anti-TNF-Alpha increases the risk of opportunistic infections and in particular, there is a significant increase in the risk of TB reactivation in conjunction with Infliximab. -
Question 109 of 123
109. Question
A 54-year-old gentleman presented to Emergency Department with acute respiratory failure during an episode of acute pancreatitis and was thought to have developed acute respiratory distress syndrome (ARDS).
_x000D_
Which one of the following would support a diagnosis of acute respiratory distress syndrome (ARDS)?
CorrectIncorrectHint
The correct answer is E. High protein pulmonary oedema.
_x000D_
Acute respiratory distress syndrome (ARDS) is a non-cardiogenic pulmonary oedema and diffuse lung inflammation syndrome that often complicates critical illness. The diagnosis of acute respiratory distress syndrome (ARDS) is based on fulfilling three criteria:
_x000D_
- _x000D_
- Acute onset (within 1 week)
- Bilateral opacities on chest x-ray
- Inability to explain respiratory failure by cardiac failure or fluid overload
_x000D_
_x000D_
_x000D_
_x000D_
Acute respiratory distress syndrome (ARDS) is thought to occur when a pulmonary or extrapulmonary insult causes the release of inflammatory mediators, promoting inflammatory cell accumulation in the alveoli and microcirculation of the lung. Inflammatory cells damage the vascular endothelium and alveolar epithelium, leading to pulmonary oedema, hyaline membrane formation, decreased lung compliance, and decreased gas exchange.
_x000D_
High protein pulmonary oedema is a hallmark of acute respiratory distress syndrome (ARDS), as it reflects the increased permeability of the alveolar-capillary membrane due to endothelial and epithelial injury. The protein content of the oedema fluid is usually greater than 75% of the plasma protein concentration.
_x000D_
The other options are not consistent with acute respiratory distress syndrome (ARDS) for the following reasons:
_x000D_
- _x000D_
- Hypercapnia is not a diagnostic criterion for acute respiratory distress syndrome (ARDS), although it may occur in some patients due to hypoventilation, ventilation-perfusion mismatch, or increased dead space.
- Increased lung compliance is not a feature of acute respiratory distress syndrome (ARDS), as the lung stiffness is increased due to the loss of surfactant, alveolar collapse, and fibrosis.
- Normal chest x-ray is not compatible with acute respiratory distress syndrome (ARDS), as bilateral opacities are required for the diagnosis. The chest x-ray may show diffuse or patchy infiltrates, air bronchograms, or a “white-out” appearance.
- High pulmonary capillary wedge pressure is indicative of cardiogenic pulmonary oedema, not acute respiratory distress syndrome (ARDS). The pulmonary capillary wedge pressure is usually normal or low in acute respiratory distress syndrome (ARDS), as the oedema is not due to hydrostatic pressure but rather increased permeability.
_x000D_
_x000D_
_x000D_
_x000D_
-
Question 110 of 123
110. Question
A 54-year-old gentleman presented to Emergency Department with Acute Respiratory Failure during an Episode Of Acute Pancreatitis and was thought to have developed Adult / Acute Respiratory Distress Syndrome (ARDS).
_x000D_
Which of the following would support a diagnosis of Adult / Acute Respiratory Distress Syndrome [ARDS]?
CorrectIncorrectHint
Acute Respiratory Distress Syndrome (ARDS) is a severe form of Acute Lung Injury (ALI) which describes a pulmonary syndrome characterised by Non-Cardiogenic Pulmonary Oedema._x000D_
It can be caused by a number of insults, the most common of which is systemic inflammation. This causes injury to the alveolar-capillary interface, with increase in the capillary permability and excudation of protein rich fluid into the interstitium and alveoli. In addition there is a deficiency in surfactant which reduces lung compliance and predisposes to collapse (especially in dependent zones). Chest radiographs, and CT, show heterogenous injury. Spirometry shows a reduced functional capacity, and the patient’s work of breathing increases._x000D_
Acute Lung Injury (ALI) has a specific definition:_x000D_
? PaO2/FiO2 Ratio Of Less Than 300_x000D_
? Bilateral Infiltrates On A Chest Radiograph_x000D_
? Pulmonary Capillary Wedge Pressure Of Less Than 18 mmHg_x000D_
Adult / Acute Respiratory Distress Syndrome [ARDS] has the same definition except that the PaO2/FiO2 Ratio is less than 200. There tends to be a Type 1 Respiratory Failure, rather than Type 2. -
Question 111 of 123
111. Question
A 52-year-old gentleman was diagnosed with Tuberculosis [TB] and had been on treatment for last three months.
_x000D_
He was lost to follow up. Four months later he presented with haemoptysis and fatigue. On examination his Temperature was 38.2°C. Sputum Analysis for Acid Fast Bacilli was positive.
_x000D_
He is suspected to have Multiple Drug Resistant Tuberculosis (MDR TB).
_x000D_
Which one of the following would be the strongest risk factor for MDR TB?
CorrectIncorrectHint
The Risk Factors for MDR TB Are:_x000D_
? History Of Prior TB Drug Treatment; Prior TB Treatment Failure_x000D_
? Contact With A Known Case Of Drug-Resistant TB_x000D_
? Birth In A Foreign Country, Particularly High-Incidence Countries_x000D_
? HIV Infection_x000D_
? Residence In London_x000D_
? Age Profile, With Highest Rates Between Ages 25 And 44, and_x000D_
? Male Gender. -
Question 112 of 123
112. Question
A 52-year-old gentleman is admitted to the Emergency Department about six days after becoming unwell with influenza.
_x000D_
Over the past 48 hours he has become progressively more short of breath with a cough productive of purulent and bloody sputum.
_x000D_
On examination, he is pyrexial 38.6 °C, his Blood Pressure is 100/60 mmHg, Pulse Rate is 97/Minute and Regular. His Saturations are 93% on Room Air. There are signs of Extensive Right Sided Consolidation.
_x000D_
Investigations show:
_x000D_
_x000D_ _x000D_
_x000D_ _x000D_ Haemoglobin
_x000D_
_x000D_
_x000D_ 130 g/L
_x000D_
_x000D_
_x000D_ (135-177)
_x000D_
_x000D_
_x000D_
_x000D_ _x000D_ Total Leucocyte Count
_x000D_
_x000D_
_x000D_ 16.1 ×109/L
_x000D_
_x000D_
_x000D_ (4-11)
_x000D_
_x000D_
_x000D_
_x000D_ _x000D_ Platelet Count
_x000D_
_x000D_
_x000D_ 194 ×109/L
_x000D_
_x000D_
_x000D_ (150-400)
_x000D_
_x000D_
_x000D_
_x000D_ _x000D_ Serum Sodium
_x000D_
_x000D_
_x000D_ 136 mmol/L
_x000D_
_x000D_
_x000D_ (135-146)
_x000D_
_x000D_
_x000D_
_x000D_ _x000D_ Serum Potassium
_x000D_
_x000D_
_x000D_ 4.4 mmol/L
_x000D_
_x000D_
_x000D_ (3.5-5)
_x000D_
_x000D_
_x000D_
_x000D_ _x000D_ Serum Creatinine
_x000D_
_x000D_
_x000D_ 128 µmol/L
_x000D_
_x000D_
_x000D_ (79-118)
_x000D_
_x000D_
_x000D_
_x000D_
_x000D_
Chest X-Ray shows Right Sided Consolidation with evidence of Cavitation.
_x000D_
Which of the following would be the most appropriate therapy?
CorrectIncorrectHint
This gentleman has Community-Acquired Pneumonia (CAP). The recent history of influenza may lead you to consider Staphylococcus aureus as the possible underlying organism, although this is an uncommon cause of Community-Acquired Pneumonia (CAP) in the United Kingdom. It is more common in the winter months and coincident influenza-type symptoms in approximately 40%. Pneumonia complications approximately 3% of cases of influenza, 10% of those admitted have been confirmed to be due to Staphyloccus aureus._x000D_
In the majority of patients, CAP should be confirmed by Chest Radiography before the commencement of Antibiotics. However, if patients are critically unwell they should be treated for the presumptive diagnosis. Antibiotic treatment should always be initiated within four hours of presentation._x000D_
Community-Acquired Pneumonia (CAP) caused by Staphyloccus aureus is more likely to present with multilobar shadowing, cavitation, pneumatoceles and spontaneous pneumothorax than other organisms. However, there are no characteristic features of chest radiographs that allow a confident prediction of the likely pathogen. Therefore, the general guidelines for treatment of CAP should be followed until an organism is identified. Staphylococcus aureus carries a high mortality, and therefore if suspected treatment should initially be for a severe CAP (see below for details)._x000D_
Low severity Community-Acquired Pneumonia (CAP) (CURB 0-1) can be treated with Amoxicillin 500 mg TDS PO. CURB 2 CAP should be treated with Amoxicillin 500 mg ? 1 gm TDS and Clarithromycin 500 mg TDS. Alternatives are available if patients are allergic to any of the above combinations. High severity CAP (CURB 3-5) should be treated as soon as possible with Co-Amoxiclav 1.2 gm TDS and Clarithromycin 500 mg BD._x000D_
The oral route is recommended in those with low and moderate severity CAP. Patients treated with parenteral antibiotics initially should be switched to an oral regimen once clinical improvement is seen and the patients has been afebrile for at least 24 hours. For most patients with uncomplicated CAP 7 days of antibiotic treatment is recommended. For those with high severity pneumonia where an organism has not been identified, 7-10 days treatment is indicated and extended to 14-21 days where clinically needed._x000D_
If Staphylococcus aureus is identified, treatment should be altered. Non-MRSA organisms should be treatment with Flucloxacillin and/or Rifampicin; an alternative for penicillin-allergic patients is Teicoplanin and Rifampicin. MRSA should be treated with Vancomycin. A prolonged antibiotic course is indicated._x000D_
_x000D_
**CURB: Confusion, Urea, Respiratory Rate, Blood Pressure. -
Question 113 of 123
113. Question
A 52-year-old gentleman is admitted to the Emergency Department about six days after becoming unwell with influenza.
_x000D_
Over the past 48 hours he has become progressively more short of breath with a cough productive of purulent and bloody sputum.
_x000D_
On examination, he is pyrexial 101.5°F, his blood pressure is 100/60 mmHg, pulse rate is 97 beats per minute and regular. His saturations are 93% on room air. There are signs of extensive right sided consolidation.
_x000D_
Investigations Show:
_x000D_
_x000D_ _x000D_
_x000D_ _x000D_ Haemoglobin
_x000D_
_x000D_
_x000D_ 130 g/L
_x000D_
_x000D_
_x000D_ (135-177)
_x000D_
_x000D_
_x000D_
_x000D_ _x000D_ Total Leucocyte Count
_x000D_
_x000D_
_x000D_ 16.1 ×109/L
_x000D_
_x000D_
_x000D_ (4-11)
_x000D_
_x000D_
_x000D_
_x000D_ _x000D_ Platelet Count
_x000D_
_x000D_
_x000D_ 194 ×109/L
_x000D_
_x000D_
_x000D_ (150-400)
_x000D_
_x000D_
_x000D_
_x000D_ _x000D_ Serum Sodium
_x000D_
_x000D_
_x000D_ 136 mmol/L
_x000D_
_x000D_
_x000D_ (135-146)
_x000D_
_x000D_
_x000D_
_x000D_ _x000D_ Serum Potassium
_x000D_
_x000D_
_x000D_ 4.4 mmol/L
_x000D_
_x000D_
_x000D_ (3.5-5)
_x000D_
_x000D_
_x000D_
_x000D_ _x000D_ Serum Creatinine
_x000D_
_x000D_
_x000D_ 128 µmol/L
_x000D_
_x000D_
_x000D_ (79-118)
_x000D_
_x000D_
_x000D_
_x000D_
_x000D_
Chest x-ray shows right sided consolidation with evidence of cavitation.
_x000D_
Which one of the following would be the most appropriate therapy for this gentleman?
CorrectIncorrectHint
The most appropriate therapy for this gentleman is B. Clarithromycin and co-amoxiclav. This is because he has signs of a bacterial pneumonia with cavitation, which suggests a necrotizing infection that may be caused by anaerobes, Staphylococcus aureus, or Klebsiella pneumoniae. Clarithromycin and co-amoxiclav are broad-spectrum antibiotics that cover these organisms and have good penetration into lung tissue.
_x000D_
The other options are less appropriate, for the following reasons:
_x000D_
- _x000D_
- A. Ciprofloxacin is a fluoroquinolone that has activity against gram-negative bacteria, but it is not very effective against anaerobes or Staphylococcus aureus. It may also have more side effects than clarithromycin and co-amoxiclav, such as tendon rupture, QT prolongation, and Clostridioides difficile infection.
- C. Flucloxacillin is a penicillin that is active against Staphylococcus aureus, but it does not cover anaerobes or gram-negative bacteria. It may also cause hypersensitivity reactions, hepatotoxicity, or interstitial nephritis.
- D. Benzylpenicillin is a penicillin that is active against some gram-positive bacteria, but it does not cover anaerobes, Staphylococcus aureus, or gram-negative bacteria. It may also cause hypersensitivity reactions, neurotoxicity, or electrolyte disturbances.
- E. Doxycycline is a tetracycline that has activity against some gram-positive and gram-negative bacteria, but it is not very effective against anaerobes or Staphylococcus aureus. It may also cause photosensitivity, oesophagitis, or tooth discolouration.
_x000D_
_x000D_
_x000D_
_x000D_
-
Question 114 of 123
114. Question
A 58-year-old pipe fitter presented with a dry nocturnal cough and increasing exertional breathlessness.
_x000D_
On examination, he had early finger clubbing, cyanosis and bilateral basal crackles. A chest x-ray showed bilateral lower zone shadowing.
_x000D_
Investigations Revealed:
_x000D_
_x000D_ _x000D_
_x000D_ _x000D_ PaO2 (Breathing Air)
_x000D_
_x000D_
_x000D_ 8.5 kPa
_x000D_
_x000D_
_x000D_ (11.3-12.6)
_x000D_
_x000D_
_x000D_
_x000D_ _x000D_ FEV1/FVC Ratio
_x000D_
_x000D_
_x000D_ 86%
_x000D_
_x000D_
_x000D_ –
_x000D_
_x000D_
_x000D_
_x000D_
_x000D_
Which one of the following investigations is most likely to establish the diagnosis?
CorrectIncorrectHint
The most likely diagnosis is interstitial lung disease (ILD), which is a group of disorders that cause inflammation and scarring of the lung tissue. Interstitial lung disease (ILD) can have various causes, such as occupational exposure, autoimmune diseases, infections, or unknown factors.
_x000D_
The most appropriate investigation to establish the diagnosis of interstitial lung disease (ILD) is D. High resolution computed tomography (HRCT) scan of chest. This is because high-resolution computed tomography (HRCT) can show detailed images of the lung tissue and reveal the extent and pattern of fibrosis, which can help narrow down the specific type of interstitial lung disease (ILD) and guide treatment decisions. High-resolution computed tomography (HRCT) is also more sensitive and specific than chest x-ray, which can be normal or nonspecific in some cases of interstitial lung disease (ILD).
_x000D_
The other investigations are not as useful or reliable for diagnosing interstitial lung disease (ILD). Serum angiotensin-converting enzyme (ACE) level can be elevated in some cases of sarcoidosis, which is a type of interstitial lung disease (ILD), but it is not specific and can be normal in up to 50% of patients. Measurement of diffusion capacity can indicate the impairment of gas exchange in the lungs, but it is not diagnostic of interstitial lung disease (ILD) and can be affected by other factors, such as anaemia or pulmonary hypertension. Echocardiography can assess the function and pressure of the heart, but it is not helpful for identifying the cause of interstitial lung disease (ILD). Transbronchial lung biopsy can obtain small samples of lung tissue, but it is not adequate for diagnosing most types of interstitial lung disease (ILD), as it can miss the areas of fibrosis or show nonspecific changes. A surgical lung biopsy may be needed in some cases where high-resolution computed tomography (HRCT) is inconclusive or the diagnosis is uncertain.
-
Question 115 of 123
115. Question
A 58-year-old lady with multiple comorbidities attends the respiratory clinic with increasing shortness of breath.
_x000D_
Medication of note includes dosulepin, omeprazole, amlodipine, ramipril, and atorvastatin.
_x000D_
On examination, her blood pressure is 130/80 mmHg, pulse rate is 72 beats per minute and regular. There are scattered inspiratory crackles throughout both lung fields. Pulmonary function testing reveals a restrictive defect.
_x000D_
Which one of the following agents is most likely to be responsible for her presenting clinical condition?
CorrectIncorrectHint
Pulmonary fibrosis is an uncommon side effect associated with the use of dosulepin. Although rare, cases of both subacute interstitial pneumonia and acute alveolitis have been reported in patients receiving dosulepin therapy.
_x000D_
Hence, the most likely agent to be responsible for the clinical condition of this lady is dosulepin (option B). Dosulepin is a tricyclic antidepressant that can cause interstitial lung disease, a type of intrinsic restrictive lung disease. Interstitial lung disease is characterized by inflammation and scarring of the lung tissue, which reduces the lung volume and impairs gas exchange. Interstitial lung disease can present with shortness of breath, crackles, and reduced lung function.
_x000D_
The other agents are less likely to cause restrictive lung disease. Ramipril (option A) is an angiotensin-converting enzyme inhibitor that can cause cough and angioedema, but not restrictive lung disease. Omeprazole (option C) is a proton pump inhibitor that can cause hypersensitivity pneumonitis, a type of extrinsic restrictive lung disease, but this is very rare. Amlodipine (option D) is a calcium channel blocker that can cause peripheral oedema, but not pulmonary oedema or restrictive lung disease. Penicillamine (option E) is a chelating agent that can cause pulmonary fibrosis, a type of intrinsic restrictive lung disease, but this is also very rare.
-
Question 116 of 123
116. Question
A 58-year-old lady with multiple comorbidities attends the Respiratory Clinic with increasing shortness of breath.
_x000D_
Medication of note includes Dosulepin, Omeprazole, Amlodipine, Ramipril, and Atorvastatin.
_x000D_
On examination, her Blood Pressure is 130/80 mmHg, Pulse Rate is 72 Beats Per Minute and Regular. There are scattered inspiratory crackles throughout both lung fields. Pulmonary Function Testing reveals a Restrictive Defect.
_x000D_
Which of the following agents is most likely to be responsible for her presenting clinical condition?
CorrectIncorrectHint
Dosulepin is recognised as a very rare cause of Pulmonary Fibrosis. Both Subacute Interstitial Pneumonia and Acute Alveolitis have been reported in association with Dosulepin Therapy._x000D_
None of the other agents listed are recognised as causes of Pulmonary Fibrosis._x000D_
More commonly recognised drug causes of Pulmonary Fibrosis include:_x000D_
? Amiodarone_x000D_
? Bleomycin_x000D_
? Methotrexate_x000D_
? Busulphan_x000D_
? Gold, and_x000D_
? Nitrofurantoin._x000D_
Penicillamine can be used in the treatment of Pulmonary Fibrosis._x000D_
Ref?. -
Question 117 of 123
117. Question
A 58-year-old lady is recently diagnosed with Small Cell Carcinoma of the Lung.
_x000D_
Which of the following Non-Metastatic Manifestations is she most likely to develop?
CorrectIncorrectHint
Non-Metastatic Paramalignant Manifestations For Small Cell Carcinoma Include:_x000D_
? Inappropriate ADH and Ectopic ACTH Secretion_x000D_
? Polymyositis_x000D_
? Dementia_x000D_
? Cerebellar Syndrome, and_x000D_
? Peripheral Neuropathy._x000D_
Eaton-Lambert Syndrome is also a Non-Metastatic Paramalignant Manifestation. Seventy percent occur in association with Small Cell Carcinoma. It is an Autoimmune Disorder affecting release of Acetylcholine at Neuromuscular Junction causing proximal muscle weakness, fatiguability, and muscle wasting. Often, power is increased initially by exercise (reversed myasthenia effect). Weakness and fatiguability can be improved with Guanidine Hydrochloride._x000D_
Cutaneous Lesions (Dermatomyositis, Thrombophlebitis Migrans, Acanthosis Nigricans, and Erythema Gyratum Repens) are rare._x000D_
Hypertrophic Pulmonary Osteoarthropathy [HPOA] and Ectopic Parathyroid Hormone (PTH)-Related Peptide Secretion relates particularly to Squamous Cell Carcinoma. -
Question 118 of 123
118. Question
A 58-year-old lady is on treatment for Rheumatoid Arthritis and she has recently become Dyspnoeic on Mild Exertion and developed a Dry Cough.
_x000D_
The Oxygen Saturation was found to be 82% on room air. The Chest X-Ray showed a Bilateral Interstitial Infiltrate. Infection Screen was negative and her symptoms were thought to be Drug-Induced.
_x000D_
Which drug is most likely to have caused this adverse effect?